Atomikello, avaruusmatkaaja, painovoima ja nopeus

Anonyymi-ap

> Mikä on nopein mahdollinen nopeus jonka ajan kulku voi saavuttaa?

Ajan kulku ei nopeudu koskaan, mutta ajan kulku voi hidastua.

Aika on fysiikan perussuure, jonka yksikkö on sekunti. Kun Cesium-isotooppi 133 viritetään kahden energiatason verran, se alkaa värähdellä tietyllä nopeudella. Kun tämä värähtelynopeus kerrotaan 9 192 635 770, saadaan sekunnin määrittelemä aika. Aikaa mitataan kellolla, tarkasti atomikellolla.

Avaruuden ja Maan välillä painovoima saa ajan kulumaan hieman eri tahtiin. Samoin vuoristossa ja alangoilla aika kuluu hieman eri tahtiin. Kyseessä on aikadilataatio. Venäläinen Gennadi Padalki vietti 879 vuorokautta Maata kiertävällä radalla. Hänelle aika kului hitaammin kuin maapallolla oleville ihmisille 0,02 sekunnin verran. Liikkuvassa koordinaatistossa kellot käyvät hitaammin, mutta liike on vain suhteellista. Koska aika ja avaruus ovat naimisissa keskenään, avaruuden käyristyminen vaikuttaa myös kellojen käyntiin. Suhteellinen aika muuttuu sen mukaan, millaisessa gravitaatiokentässä havaitsija liikkuu.

--

Yksi sekunti on siis sidottu Cs-133-atomin värähtelyyn.

Herää kysymys siitä, että jos avaruudessa matkataan todella suurella nopeudella - sovitaan 90 % valonnopeudesta - tai todella suuressa painovoimakentässä, myös Cs-133-atomin värähtelytaajuuden on muututtava tai määritelmää värähtelylle on avaruudessa muutettava. Avaruusmatkailija ja maahan jäänyt kollega tuntevat omassa koordinaatistossaan ajan kuluvan samalla tavoin, vaikka todellisuudessa näin ei ole: kaksi identtistä atomikelloa kykenevät näyttämään todellisen eron.

Koska aika on sidottu atomikellon värähtelyyn, mutta ko. avaruuden olosuhteissa aika kuluu hitaammin, atomikellon värähtelytaajuuden on myös muututtava, tai Maassa määritelty kerroin on väärin.

(1) Mihin fysiikan lakiin perustuu se, että Cs-133-atomin värähtely muuttuu kellon matkatessa - avaruudessa - suurta nopeutta (tai suuressa painovoimakentässä)?

(2) Jos avaruusmatkaaja haluaisi pysyä aina yhdessä ja samassa Maan tosiajassa, minkä kaavan mukaisesti hänen tulisi etsiä kerroin, jolla atomikellon värähtelytaajuus kerrotaan yhden sekunnin määritelmän mukaiseksi?

136

1530

    Vastaukset

    Anonyymi (Kirjaudu / Rekisteröidy)
    5000
    • Anonyymi

      Kohta (1) meni jo pieleen. Sekunti on paikalliselle havainnoijalle sekunnin mittainen. Sen saman, minkä me "paikallamme olevat" aistimme.

      • Anonyymi

        Tekstisi oli kaikkia mahdollisia perusteluja vaille varmasti lähes totta.

        Vastaa siihen kysymykseen, miten sekunti voi olla havainnoijalle sekunnin mittainen, jos hänellä ei ole tiedossa, mikä on tosiaika ja reaalisekunti. Ihmisen aika, siis sekunti, on sidottu atomin värähtelyyn, eikä se voi olla fysiikassa subjektiivinen käsite, vaikka ihmiselle olisikin. Sekunti on sekunti joka paikassa, jos vain atomin värähtelyllä on merkitystä.

        Muuttuuko siis Cs-133-atomin värähtely kellon matkatessa - avaruudessa - suurta nopeutta (tai suuressa painovoimakentässä)? Mihin fysiikan lakiin muutos perustuu?


      • Anonyymi
        Anonyymi kirjoitti:

        Tekstisi oli kaikkia mahdollisia perusteluja vaille varmasti lähes totta.

        Vastaa siihen kysymykseen, miten sekunti voi olla havainnoijalle sekunnin mittainen, jos hänellä ei ole tiedossa, mikä on tosiaika ja reaalisekunti. Ihmisen aika, siis sekunti, on sidottu atomin värähtelyyn, eikä se voi olla fysiikassa subjektiivinen käsite, vaikka ihmiselle olisikin. Sekunti on sekunti joka paikassa, jos vain atomin värähtelyllä on merkitystä.

        Muuttuuko siis Cs-133-atomin värähtely kellon matkatessa - avaruudessa - suurta nopeutta (tai suuressa painovoimakentässä)? Mihin fysiikan lakiin muutos perustuu?

        Entä jos se atomikello on koko ajan maassa, ja vain kellon näyttö välitetään radioteitse avaruusmatkaajalle?


      • Anonyymi
        Anonyymi kirjoitti:

        Entä jos se atomikello on koko ajan maassa, ja vain kellon näyttö välitetään radioteitse avaruusmatkaajalle?

        Hyvä kysymys!

        Olisi siis Maassa oleva "maakello", avaruudessa olevan maakellon "näytön kopio" sekä avaruudessa mukana oleva "avaruuskello".

        Olisiko niin, että maakellon kopiosta katsottuna avaruusmatkaajan aika tuntuu ikuisuudelta, mutta mukana olevan avaruuskellon mukaan katsottuna aika kuluu matkaajan kannalta normaalisti? Siis olettaen, että matkaajalla itsellään on jonkinlainen tuntemus ajan kulumisesta.

        Lienee selvää, että maakellon näyttö voitaisiin kopioida avaruuteen sellaisenaan. Tuskin aika muuttuu ajan (näytön kopion) välittämisessä avaruuskellon näytölle. Lienee selvää, että mukana oleva avaruuskello ja maakellon näytön kopio näyttäisivät aivan eri aikaa. Tosiaika olisi Maan aika.

        Edelleenkään ei ole selvää, muuttuuko mukana olevan atomikellon värähdystaajuus, vai pitääkö kellon kerrointa vain säätää, jotta tosiaika ja avaruusaika vastaisivat toisiaan.


      • Anonyymi
        Anonyymi kirjoitti:

        Tekstisi oli kaikkia mahdollisia perusteluja vaille varmasti lähes totta.

        Vastaa siihen kysymykseen, miten sekunti voi olla havainnoijalle sekunnin mittainen, jos hänellä ei ole tiedossa, mikä on tosiaika ja reaalisekunti. Ihmisen aika, siis sekunti, on sidottu atomin värähtelyyn, eikä se voi olla fysiikassa subjektiivinen käsite, vaikka ihmiselle olisikin. Sekunti on sekunti joka paikassa, jos vain atomin värähtelyllä on merkitystä.

        Muuttuuko siis Cs-133-atomin värähtely kellon matkatessa - avaruudessa - suurta nopeutta (tai suuressa painovoimakentässä)? Mihin fysiikan lakiin muutos perustuu?

        Atomin värähtelytaajuus ei tavallaan muutu, vaan itse aika. Perustuu yleiseen suhteellisuusteoriaan, ja sen pohjalta tehtyihin havaintoihin.


      • Anonyymi
        Anonyymi kirjoitti:

        Atomin värähtelytaajuus ei tavallaan muutu, vaan itse aika. Perustuu yleiseen suhteellisuusteoriaan, ja sen pohjalta tehtyihin havaintoihin.

        Ajan määritelmä (eli samalla sekunnin) määritelmä perustuu atomikellon Cs-133-värähtelyyn.

        Jos värähtely ei "tavallaan muutu", sekunti on sekunti kaikilla mahdollisilla tavoilla arvioituna ja kaikissa mahdollisissa olosuhteissa. Aika on siis aina sama. Näin ei kuitenkaan tunnetusti ole: yksi sekunti ei ole aina tasan yksi sekunti. Joko kertoimen on muututtava tai värähtelynopuden on muututtava.

        Joko ajan (ja sekunnin) määritelmä ei päde kaikissa mahdollisissa värähtelyolosuhteissa, ja määritelmä ei ole yleispätevä, tai värähtely ei ole samanlaista eri painovoimakentissä tai suurissa nopeuksissa.

        Aika ei muutu, jos sekunnin määritelmä ei muutu.


      • Anonyymi
        Anonyymi kirjoitti:

        Hyvä kysymys!

        Olisi siis Maassa oleva "maakello", avaruudessa olevan maakellon "näytön kopio" sekä avaruudessa mukana oleva "avaruuskello".

        Olisiko niin, että maakellon kopiosta katsottuna avaruusmatkaajan aika tuntuu ikuisuudelta, mutta mukana olevan avaruuskellon mukaan katsottuna aika kuluu matkaajan kannalta normaalisti? Siis olettaen, että matkaajalla itsellään on jonkinlainen tuntemus ajan kulumisesta.

        Lienee selvää, että maakellon näyttö voitaisiin kopioida avaruuteen sellaisenaan. Tuskin aika muuttuu ajan (näytön kopion) välittämisessä avaruuskellon näytölle. Lienee selvää, että mukana oleva avaruuskello ja maakellon näytön kopio näyttäisivät aivan eri aikaa. Tosiaika olisi Maan aika.

        Edelleenkään ei ole selvää, muuttuuko mukana olevan atomikellon värähdystaajuus, vai pitääkö kellon kerrointa vain säätää, jotta tosiaika ja avaruusaika vastaisivat toisiaan.

        Tietenkin radioaalloilla menee tietty aika sen maakellon näytön välittämiseen avaruusmatkaajalle.
        Pitääkö sen matkaajan välttämättä edetä kauemmaksi maa kellosta, jos vaikka kiertäisi avaruudessa ympyrää, etäisyys maahan lukittuna. Silloin radioaalloilla olisi aina sama matka kulkea näyttökopiolle ja aikaviive vakio.


      • Anonyymi
        Anonyymi kirjoitti:

        Ajan määritelmä (eli samalla sekunnin) määritelmä perustuu atomikellon Cs-133-värähtelyyn.

        Jos värähtely ei "tavallaan muutu", sekunti on sekunti kaikilla mahdollisilla tavoilla arvioituna ja kaikissa mahdollisissa olosuhteissa. Aika on siis aina sama. Näin ei kuitenkaan tunnetusti ole: yksi sekunti ei ole aina tasan yksi sekunti. Joko kertoimen on muututtava tai värähtelynopuden on muututtava.

        Joko ajan (ja sekunnin) määritelmä ei päde kaikissa mahdollisissa värähtelyolosuhteissa, ja määritelmä ei ole yleispätevä, tai värähtely ei ole samanlaista eri painovoimakentissä tai suurissa nopeuksissa.

        Aika ei muutu, jos sekunnin määritelmä ei muutu.

        Anon.
        "Aika ei muutu, jos sekunnin määritelmä ei muutu."

        Ajanmittauksen historiassa on ollut eripituisia sekunteja, tuskin se
        todellisuuden aika muuttuu mihinkään vaikka vaihdellaan sekunnin
        määritelmiä.

        Määritelmät ovat aina käsitteisiin liittyvää toimintaa.

        Einsteinille ajan ja kellon suhde oli vaikea ongelma, hän tiesi että luonnossa
        ei ole "todellista" kelloa, joten vasta 1936 hän sai aikaan jonkinlaisen
        määritelmän kellon käsitteestä. Eikä sekään ollut hyvä määritelmä.

        Sekunnin määritelmä on aina aikajärjestelmään kytkeytyvä määritelmä,
        ei aikaan. Nykyään on ainakin 6-7 erilaista sekunnin määritelmää eri
        aikajärjestelmissä.

        Atomikello on aikajärjestelmässä ihmisen mukavuuden ja helpon käsiteltävyyden
        takia, todellisuudelle atomikello ei ole mitenkään tärkeä.
        Atomikellon tärkein saavutus on samanpituiset sekunnit 10-1 nanosekunnin
        tasolla.

        Atomikello tarkkuutensa takia aiheuttaa ongelmia aikajärjestelmissä,
        tutkimuksissa atomikellosta on kyllä suurta apua.


      • Anonyymi
        Anonyymi kirjoitti:

        Atomin värähtelytaajuus ei tavallaan muutu, vaan itse aika. Perustuu yleiseen suhteellisuusteoriaan, ja sen pohjalta tehtyihin havaintoihin.

        Vaikuttaako ihmisen vanhenemiseen painovoimakenttä? Jos ihminen lentää esim avaruusaluksella paikkaan, johon painovoima ei vaikuta. Vanheneeko hän nopeammin, kuin maassa oleva jossain ajassa.


      • Anonyymi
        Anonyymi kirjoitti:

        Vaikuttaako ihmisen vanhenemiseen painovoimakenttä? Jos ihminen lentää esim avaruusaluksella paikkaan, johon painovoima ei vaikuta. Vanheneeko hän nopeammin, kuin maassa oleva jossain ajassa.

        Kyllä. Varpaat vanhenevat eri tahtia kuin pää kun ihminen seisoo. Syynä se, että varpaat ovat syvemmällä Maapallon massan aiheuttamassa gravitaatiokuopassa kuin pää. Gravitaatiopunasiirtymän vuoksi kun katsot alaspäin kädessäsi olevaa kelloa niin se tikittää hitaammin kuin miten kävisi silmiesi korkeudella oleva kello

        Mittauksin nähdään jo nyt se, miten yhden senttimetrin ero korkeudessa vaikuttaa gravitaation vuoksi kellon käyntinopeuteen.

        https://physicsworld.com/a/gravitational-time-dilation-measured-on-centimetre-and-millimetre-scales-in-atomic-clocks/

        Atomikello mittaa paikallista aikaa. Se ei ota kantaa siihen millä nopeudella aika kulkee jossakin muussa paikassa. Ajan kulun vertaamisessa joutuu tekemään hieman työtä ja ottamaan huomioon suhteellisuusteoreettiset korjaukset.

        https://en.wikipedia.org/wiki/Einstein_synchronisation


      • Anonyymi
        Anonyymi kirjoitti:

        Kyllä. Varpaat vanhenevat eri tahtia kuin pää kun ihminen seisoo. Syynä se, että varpaat ovat syvemmällä Maapallon massan aiheuttamassa gravitaatiokuopassa kuin pää. Gravitaatiopunasiirtymän vuoksi kun katsot alaspäin kädessäsi olevaa kelloa niin se tikittää hitaammin kuin miten kävisi silmiesi korkeudella oleva kello

        Mittauksin nähdään jo nyt se, miten yhden senttimetrin ero korkeudessa vaikuttaa gravitaation vuoksi kellon käyntinopeuteen.

        https://physicsworld.com/a/gravitational-time-dilation-measured-on-centimetre-and-millimetre-scales-in-atomic-clocks/

        Atomikello mittaa paikallista aikaa. Se ei ota kantaa siihen millä nopeudella aika kulkee jossakin muussa paikassa. Ajan kulun vertaamisessa joutuu tekemään hieman työtä ja ottamaan huomioon suhteellisuusteoreettiset korjaukset.

        https://en.wikipedia.org/wiki/Einstein_synchronisation

        Meinaat että kun laitetaan kaksi atomikelloa kelloa seinälle, lattianrajaan ja 2 metrin korkeudelle. Tarkkailija asettuu vastakkaisen seinän vieseen istumaan tuolille, siten että silmänsä on yhden metrin korkedella. Tästä hyvästä tarkkailuasemasta hän voi heti huomata että ylempi kello alkaa edistää verrattuna alempaan kelloon. 😂🤣


      • Anonyymi
        Anonyymi kirjoitti:

        Meinaat että kun laitetaan kaksi atomikelloa kelloa seinälle, lattianrajaan ja 2 metrin korkeudelle. Tarkkailija asettuu vastakkaisen seinän vieseen istumaan tuolille, siten että silmänsä on yhden metrin korkedella. Tästä hyvästä tarkkailuasemasta hän voi heti huomata että ylempi kello alkaa edistää verrattuna alempaan kelloon. 😂🤣

        Kyllä. Katsojan havaintojen mukaan ylempänä oleva kello edistää alempana olevaan kelloon nähden.

        Näitä mittauksia on tehty. Ihan samalla lailla on jo 1970-luvulla verrattu paikallaan ollutta atomikellon aikaa sellaiseen, jota lennätettiin Maapallon ympäri lentokoneella. Ero oli suhteellisuusyeorian ennusteen mukainen.

        Jos puhelimessasi on GPS navigaattori niin sen vastaanottimessa huomioidaan gravitaatiopunasiirtymän aiheuttama signaalin taajuuden nousu satelliittien kiertoradan korkeudella verrattuna Maapallon pinnalla olevaan vastaanottimeen. Tämä on siis nykyään varsin arkipäiväinen asia.


      • Anonyymi
        Anonyymi kirjoitti:

        Kyllä. Katsojan havaintojen mukaan ylempänä oleva kello edistää alempana olevaan kelloon nähden.

        Näitä mittauksia on tehty. Ihan samalla lailla on jo 1970-luvulla verrattu paikallaan ollutta atomikellon aikaa sellaiseen, jota lennätettiin Maapallon ympäri lentokoneella. Ero oli suhteellisuusyeorian ennusteen mukainen.

        Jos puhelimessasi on GPS navigaattori niin sen vastaanottimessa huomioidaan gravitaatiopunasiirtymän aiheuttama signaalin taajuuden nousu satelliittien kiertoradan korkeudella verrattuna Maapallon pinnalla olevaan vastaanottimeen. Tämä on siis nykyään varsin arkipäiväinen asia.

        Anon.
        "Näitä mittauksia on tehty. Ihan samalla lailla on jo 1970-luvulla verrattu..."

        Tuo lentokonekoe ei onnistunut aivan noin, suht.teorian ennuste osui
        väärin ja pienen "korjailun" jälkeen saatiin tyydyttävä tulos.

        GPS perustuu vain siihen että sateliittien atomikellot käyvät samaa
        tahtia kuin maa-aseman kellot, sen takia niitä tahdistetaan aikataulun
        mukaan, vaikka suhtis kieltää samanaikaisuuden tapahtumien välillä,
        GPS-kellot käyvät samanaikaisesti.


      • Anonyymi
        Anonyymi kirjoitti:

        Ajan määritelmä (eli samalla sekunnin) määritelmä perustuu atomikellon Cs-133-värähtelyyn.

        Jos värähtely ei "tavallaan muutu", sekunti on sekunti kaikilla mahdollisilla tavoilla arvioituna ja kaikissa mahdollisissa olosuhteissa. Aika on siis aina sama. Näin ei kuitenkaan tunnetusti ole: yksi sekunti ei ole aina tasan yksi sekunti. Joko kertoimen on muututtava tai värähtelynopuden on muututtava.

        Joko ajan (ja sekunnin) määritelmä ei päde kaikissa mahdollisissa värähtelyolosuhteissa, ja määritelmä ei ole yleispätevä, tai värähtely ei ole samanlaista eri painovoimakentissä tai suurissa nopeuksissa.

        Aika ei muutu, jos sekunnin määritelmä ei muutu.

        Pitää muistaa että että aika oli olemassa jo ennenkuin joku keksi sitoa ajan Cs - 133 värähtelyyn. Eli joku on tutkinut että se on lähimpänä sekunnin määritettä. Voisihan se olla vaikkapa vesipisaran tippuminen tietyltä matkalta. Ihan yhtä tarkkoja. Riippuu vain mittaus olosuhteista. Sekuntti niinkään ei ole mikään vakio. Sekuntti nuorilla ihmisillä on paljon pidempi kuin ruuhkavuosiaan viettävä aikuinen. Meidän ymmärryksemme ajasta on puutteellinen. Ihan yhtälailla, onko aika sidoksissa vain ihmisiin. Mistäs tiedämme mitenkä muu maailma kokee ajan??? Eläimet, kasvit, avaruus, henkimaailma, yms... Toki me huomataan niiden vanhentuminen mutta, miten ne kokee ajan? Koskeeko aika vain meitä. Onko olemassa ajan ulkopuolista tilaa, välitilaa, taikkapa niiden välissä manipulaatiota. Siis tarkoitan voiko joku välilllä olla ajassa ja ajan ulkopuolella. Lukeehan raamatussa: ‘yksi päivä on Herran edessä niinkuin tuhat vuotta ja tuhat vuotta niinkuin yksi päivä.’ Eli tämän Cs - 133 värähtely pätee vain jossain parametreissä. Ihan yhtälailla kun katsoo lentokoneesta joka lentää 1000km/h autoja motarilla, ne suorastaan matelee vaikka vauhti on 100-200km tunnissa. Jos tuon vauhdin kertoo 10 -5 niin ne matelisi kuitenkin kun etäältä katsot, vaikka vauhti oisi sama missä itse olet lentokoneessa.


      • Anonyymi
        Anonyymi kirjoitti:

        Pitää muistaa että että aika oli olemassa jo ennenkuin joku keksi sitoa ajan Cs - 133 värähtelyyn. Eli joku on tutkinut että se on lähimpänä sekunnin määritettä. Voisihan se olla vaikkapa vesipisaran tippuminen tietyltä matkalta. Ihan yhtä tarkkoja. Riippuu vain mittaus olosuhteista. Sekuntti niinkään ei ole mikään vakio. Sekuntti nuorilla ihmisillä on paljon pidempi kuin ruuhkavuosiaan viettävä aikuinen. Meidän ymmärryksemme ajasta on puutteellinen. Ihan yhtälailla, onko aika sidoksissa vain ihmisiin. Mistäs tiedämme mitenkä muu maailma kokee ajan??? Eläimet, kasvit, avaruus, henkimaailma, yms... Toki me huomataan niiden vanhentuminen mutta, miten ne kokee ajan? Koskeeko aika vain meitä. Onko olemassa ajan ulkopuolista tilaa, välitilaa, taikkapa niiden välissä manipulaatiota. Siis tarkoitan voiko joku välilllä olla ajassa ja ajan ulkopuolella. Lukeehan raamatussa: ‘yksi päivä on Herran edessä niinkuin tuhat vuotta ja tuhat vuotta niinkuin yksi päivä.’ Eli tämän Cs - 133 värähtely pätee vain jossain parametreissä. Ihan yhtälailla kun katsoo lentokoneesta joka lentää 1000km/h autoja motarilla, ne suorastaan matelee vaikka vauhti on 100-200km tunnissa. Jos tuon vauhdin kertoo 10 -5 niin ne matelisi kuitenkin kun etäältä katsot, vaikka vauhti oisi sama missä itse olet lentokoneessa.

        Anon.
        "Pitää muistaa että että aika oli olemassa jo ennenkuin joku keksi sitoa ajan Cs -
        133 värähtelyyn. Eli joku on tutkinut että se on lähimpänä sekunnin määritettä."

        Sekunti on määritelty aikaisemmin useita kertoja. Tämä cs-133 määritelmä
        on SI-järjestelmän mukainen määritelmä. Sen lisäksi on erilaisia aikajärjestelmiä
        joissa sekunti on määritelty, suhteessa muihin sekunteihin, on olemassa
        6-7 eri sekunnin määritelmää.

        Sekunnilla ei tietenkään ole todellista merkitystä todellisuudessa, se on vain
        ihmisille tarkoitettu, että aikayksiköitä käytetään samalla tavalla. Aikoinaan
        roomalaiset pitivät sekunnin mittana sisäänhengityksen kestoa.

        "Sekuntti niinkään ei ole mikään vakio. Sekuntti nuorilla ihmisillä..."
        aika voi tuntua nuorilla eri pituiselta kuin vanhoilla, se ei kuitenkaan muuta
        sekunnin pituutta.

        Aika on tietyllä tavalla sidoksissa ihmiseen, sillä meillä ainoastaan on
        aikakäsite, vaikka eläimillä on ajallisen hahmotuksen osia käytössään,
        aikakäsitettä niillä ei ole.

        Jumalasta ja henkimaailmasta en sano mitään.

        Aika on kuitenkin ollut olemassa ennen ihmistä, me emme ole tehneet aikaa,
        vaikka jotkut väittävät näin.


      • Anonyymi

        🍒🍑🍒🍑🍒🍑🍒🍑🍒

        😋 ­­N­y­m­f­­­o­­­m­­­a­a­n­­i -> https://l24.im/ecC7ux#kissagirl21

        🔞❤️💋❤️💋❤️🔞❤️💋❤️💋❤️🔞


    • Anonyymi

      Typerä aihe täällä inttämiseen.

      • Anonyymi

        Kun taito ja osaaminen loppuu, vastauksesi on 100 % oikein.


      • Anonyymi
        Anonyymi kirjoitti:

        Kun taito ja osaaminen loppuu, vastauksesi on 100 % oikein.

        Hmm.Ajatus lensikin sitten aikaakin nopeammin vaikka aiheuttikin mennessään kitkaa , vastusta sekä lämpeämistä maan painovoimakentän keskuuteen.


      • Anonyymi
        Anonyymi kirjoitti:

        Hmm.Ajatus lensikin sitten aikaakin nopeammin vaikka aiheuttikin mennessään kitkaa , vastusta sekä lämpeämistä maan painovoimakentän keskuuteen.

        Mitä YRITIT sanoa meille muille?


    • Anonyymi

      Anon.
      "Ajan kulku ei nopeudu koskaan, mutta ajan kulku voi hidastua."

      Aloittajalta hyvä kysymys, en anna kaavoja, mutta taustatietoa lisää.

      Kukaan ei ole vielä matkannut 90% valon nopeudesta, mutta muita
      esimerkkejä on, luotaimet jotka ovat kaukana ja liikkuvat kohtuullisen
      nopeasti, planeettojen kuvausluotaimet olleet siellä jo vuosia ja tietysti
      satelliittinavigointijärjestelmät.

      Kaikissa näissä on yhteinen piirre, niiden toimintaa valvotaan maasta,
      vaikka itse luotaimissa on atomikellot, eivät tosin niin tarkkoja kuin maassa.
      Maavalvomot elävät tietenkin UTC-aikajärjestelmässä ja kaikki käskyt
      suoritetaan sen mukaan.

      Nyt UTC-järjestelmästä on poistettu karkaussekunnit usean vuoden ajaksi,
      joten UTC-sekunti on SI-sekunnin pituinen, atomiaikaa ei sellaisenaan ole
      vaan TAI-järjestelmä joka suhteuttaa sen muihin järjestelmiin.
      UTC-aika ja TAI-aika käyvät samaan tahtiin, niillä on 37 sekunnin off-set,
      joten TAI on 37 sekuntia edellä UTC:tä.

      Esimerkiksi Plutoa kuvannut luotain toimii UTC-ajassa maavalvomon tahtiin,
      luotaimen alajärjestelmillä voi olla omia aikajärjestelmiä, jätetään ne pois.
      Luotaimen kellot ajautuvat pois valvomon kellojen tahdista, joten niitä
      tahdistetaan silloin tällöin.

      Kellojen erot voidaan laskea suoraan kaavasta jossa valon nopeus c on
      maksiminopeus, mikä löytyy muualta, tai luotain voisi lähettää oman
      aikamerkkinsä, jolloin valvomo tietäisi aikaeron ja voisi resetoida luotaimen
      kellon samaan tahtiin. Tai ehkä valvomo lähettää resetointisignaalia
      jatkuvasti tietyin aikavälein.

      Kellon resetointi suoritetaan digitaalisen informaation muodossa, joka ei
      välitä mitään koordinaatiovaihdoista, joten luotaimen kello ja valvomon
      kello saadaan samaan aikaan jollakin tarkkuudella.

      Näin toimivat ainakin MRO-kuvausluotain, LRO-luotain kuun radalla,
      GPS-navigaatiosateliitit, samoin euroopan ja kiinan navigointisysteemit
      käyttävät GPS-aikajäjestelmää, venäläinen systeemi UTC-järjestelmää.

      Koska luotaimet ja GPS-sateliitit ovat insinöörien tekemiä, heillä on omat
      ratkaisunsa tahdistuksen tekemiseen. Eikä firmat, jotka rakentavat sateliitit
      kerro edes fyysikolle miten se tehdään, sehän on liikesalaisuus.

      Mutta rivien välistä voisi tulkita, että insinöörit katsovat kelloerojen syntyvän
      kellon taajuuden muuttumisesta, joka olisi helppo resetoida, kun taas fyysikot
      tulkitsevat että aika muuttuu.

      Mutta mitään Einsteinilaista tahdistusta kellojen välillä ei tehdä.

      Loppuhuomautuksena, atomikello ei tunnista kiihtyvyyttä, ainakaan noin
      10^19 G:n tasolle.

      Ajasta.

      • Anonyymi

        No olihan siinä vuodatusta!.

        Esim. GPS-paikannuksessa suoritetaan monenlaisia korjauksia. Mutta niissä on mukana myös Einsteinin EST:sta ja YST:sta johtuvat korjaukset eli "einsteinilaisa tahdistuksia". Ei
        pidä sekoittaa korjauksissa käytettävää tekniikkaaniiden tarpeen fysikaalisiin perussyihin.

        Kaikkiaan GPS-systeemi on varsin monimutkainen, Jonkinlaista kuvaa tästä monimutkaisuudesta antaa Wikipedia-artikkeli "Global Positioning System" ja siinä annetut linkit muihin jutuihin. Kyllä sielläkin nuo relativistiset korjauksetkin mainitaan kohdassa "Timekeeping/Relativistic corrections".

        Esim. Amazonin luettelosta löytyy asiasta kirjoja, monien satojen sivujen juttuja kaikki.

        Järjestelmän nvarsinainen yekninen dikumentaatio lienee aikamoinen läjä.

        Kun puhutaan teoriassa tuosta aloittajan kysymyksestä ajan muuttumisesta ei GPS ole kovin hyvä esimerkki juuri tuon monimutkaisuutensa vuoksi.

        EST::n kaksosparadoksista on kyllä kirjoitettu tosi suuri määrä kirjoja ja artikkeleita.Aika paljon myös tuosta gravitaatioefektistä, esim. juuri mustien aukkojen tapauksissa. Koska nämä aikaefektit ovat aika lailla "arkijärjen" vastaisia, ei niitä kunnolla ymmärrä muuta kuin perehtymällä niiden matemaattisiin esityksiin. Populaariesitykset eivät oikein toimi.


      • Anonyymi
        Anonyymi kirjoitti:

        No olihan siinä vuodatusta!.

        Esim. GPS-paikannuksessa suoritetaan monenlaisia korjauksia. Mutta niissä on mukana myös Einsteinin EST:sta ja YST:sta johtuvat korjaukset eli "einsteinilaisa tahdistuksia". Ei
        pidä sekoittaa korjauksissa käytettävää tekniikkaaniiden tarpeen fysikaalisiin perussyihin.

        Kaikkiaan GPS-systeemi on varsin monimutkainen, Jonkinlaista kuvaa tästä monimutkaisuudesta antaa Wikipedia-artikkeli "Global Positioning System" ja siinä annetut linkit muihin jutuihin. Kyllä sielläkin nuo relativistiset korjauksetkin mainitaan kohdassa "Timekeeping/Relativistic corrections".

        Esim. Amazonin luettelosta löytyy asiasta kirjoja, monien satojen sivujen juttuja kaikki.

        Järjestelmän nvarsinainen yekninen dikumentaatio lienee aikamoinen läjä.

        Kun puhutaan teoriassa tuosta aloittajan kysymyksestä ajan muuttumisesta ei GPS ole kovin hyvä esimerkki juuri tuon monimutkaisuutensa vuoksi.

        EST::n kaksosparadoksista on kyllä kirjoitettu tosi suuri määrä kirjoja ja artikkeleita.Aika paljon myös tuosta gravitaatioefektistä, esim. juuri mustien aukkojen tapauksissa. Koska nämä aikaefektit ovat aika lailla "arkijärjen" vastaisia, ei niitä kunnolla ymmärrä muuta kuin perehtymällä niiden matemaattisiin esityksiin. Populaariesitykset eivät oikein toimi.

        Anon. kirjoitti: "Koska nämä aikaefektit ovat aika lailla "arkijärjen" vastaisia, ei niitä kunnolla ymmärrä muuta kuin perehtymällä niiden matemaattisiin esityksiin."

        Tiedämme, että yksi sekunti ei ole kaikissa olosuhteissa aina tasan yksi sekunti. Kuitenkin sekunnin määritelmä on aina yksi ja sama.

        Koska aika ei ole aina sama, joko sekunnin määritelmässä käytettävän kertoimen on muututtava olosuhteita vastaavaksi tai värähtelynopeuden on muututtava. Joko ajan (ja sekunnin) määritelmä ei päde kaikissa mahdollisissa värähtelyolosuhteissa, ja määritelmä ei ole yleispätevä, tai värähtely ei ole samanlaista eri painovoimakentissä tai suurissa nopeuksissa.

        Olisi helppo ajatella, että kappaleen nopeuden kasvaessa lähelle teoreettista maksimia tai painovoiman lähestyessä äärettömyyttä, atomin värähtelytaajuus muuttuisi. Onko tämä arkijärjellä ajateltu asia matemaattisesti aivan liian yksinkertainen ja pelkkä oikotie? Pysyykö atomin värähtelytaajuus siis aina samana?


      • Anonyymi
        Anonyymi kirjoitti:

        Anon. kirjoitti: "Koska nämä aikaefektit ovat aika lailla "arkijärjen" vastaisia, ei niitä kunnolla ymmärrä muuta kuin perehtymällä niiden matemaattisiin esityksiin."

        Tiedämme, että yksi sekunti ei ole kaikissa olosuhteissa aina tasan yksi sekunti. Kuitenkin sekunnin määritelmä on aina yksi ja sama.

        Koska aika ei ole aina sama, joko sekunnin määritelmässä käytettävän kertoimen on muututtava olosuhteita vastaavaksi tai värähtelynopeuden on muututtava. Joko ajan (ja sekunnin) määritelmä ei päde kaikissa mahdollisissa värähtelyolosuhteissa, ja määritelmä ei ole yleispätevä, tai värähtely ei ole samanlaista eri painovoimakentissä tai suurissa nopeuksissa.

        Olisi helppo ajatella, että kappaleen nopeuden kasvaessa lähelle teoreettista maksimia tai painovoiman lähestyessä äärettömyyttä, atomin värähtelytaajuus muuttuisi. Onko tämä arkijärjellä ajateltu asia matemaattisesti aivan liian yksinkertainen ja pelkkä oikotie? Pysyykö atomin värähtelytaajuus siis aina samana?

        Kun matkaajakaksonen matkaa hirmuvauhtia kohti Proxima Centauria niin hänen sydämensä lyö noin 60 kertaa minuutissa h ä n e n r sa a m i s s a a n (koordinaatistossaan)..

        Maassa olevan kaksosen sydän lyö noin 60 kertaa minuutissa h ä n e n r a a m i s s a a n.
        Mutta kun matkaaja tulee takaisin, maakaksosen sydän on lyönyt kymmenien vuosien ajan ja matkaajan sydän vain muutaman vuoden ajan.

        Nämä kaksoset eivät elä Newtonin absoluuttisessa avaruudessa jossa on absoluuttinen aika, vaan heillä on Minkowskin avaruudessa eri maailmanviivat ja maakaksosen viiva on paljon pitempi kuin matkaajan viima Lorentzin metriikassa.

        Koko jutun selittäminen maallikolle vaatisi oppikirjan verran tekstiä jota kukaan tuskin tälle palstalle viitsii ruveta kirjoittamaan. Löytyyhän niitä juttuja valmiina.

        Lyhyempi matemaattinen esitys taas on sellainen, ettei asiaan perehtymätön saa siitä selvää ja perehtyneelle se on tarpeeton.


      • Anonyymi
        Anonyymi kirjoitti:

        Kun matkaajakaksonen matkaa hirmuvauhtia kohti Proxima Centauria niin hänen sydämensä lyö noin 60 kertaa minuutissa h ä n e n r sa a m i s s a a n (koordinaatistossaan)..

        Maassa olevan kaksosen sydän lyö noin 60 kertaa minuutissa h ä n e n r a a m i s s a a n.
        Mutta kun matkaaja tulee takaisin, maakaksosen sydän on lyönyt kymmenien vuosien ajan ja matkaajan sydän vain muutaman vuoden ajan.

        Nämä kaksoset eivät elä Newtonin absoluuttisessa avaruudessa jossa on absoluuttinen aika, vaan heillä on Minkowskin avaruudessa eri maailmanviivat ja maakaksosen viiva on paljon pitempi kuin matkaajan viima Lorentzin metriikassa.

        Koko jutun selittäminen maallikolle vaatisi oppikirjan verran tekstiä jota kukaan tuskin tälle palstalle viitsii ruveta kirjoittamaan. Löytyyhän niitä juttuja valmiina.

        Lyhyempi matemaattinen esitys taas on sellainen, ettei asiaan perehtymätön saa siitä selvää ja perehtyneelle se on tarpeeton.

        Anon.
        "Kun matkaajakaksonen matkaa hirmuvauhtia kohti Proxima Centauria..."

        Hyvin lyhyesti sanottuna.

        Matkaaja vaihtaa koordinaatistoaan useasti matkan aikana,
        jokaisen vaihdon aikana maapallolla oleva kaksonen vanhenee "äkkiä".

        Jos matkaaja jäisikin kiertämään kaukaista tähteä, joka kierroksella
        maapallolla oleva vanhenisi rutkasti lisää.

        Näin asia on myös selitetty.

        Mitä jos kummankin kaksosen luona on atomikello ja yhteydenpitolaitteet,
        he lähettäisivät digitaalisesti koodatun viestin toisilleen kellojensa lukemista.

        Tuo digitaalinen viesti ei välitä mitään koordinaatiovaihdoista, mitä
        kaksoset näkisivät toistensa kellojen lukemista?

        Ja vielä, atomikello ei tunnista kiihtyvyyttä.


      • Anonyymi
        Anonyymi kirjoitti:

        Anon. kirjoitti: "Koska nämä aikaefektit ovat aika lailla "arkijärjen" vastaisia, ei niitä kunnolla ymmärrä muuta kuin perehtymällä niiden matemaattisiin esityksiin."

        Tiedämme, että yksi sekunti ei ole kaikissa olosuhteissa aina tasan yksi sekunti. Kuitenkin sekunnin määritelmä on aina yksi ja sama.

        Koska aika ei ole aina sama, joko sekunnin määritelmässä käytettävän kertoimen on muututtava olosuhteita vastaavaksi tai värähtelynopeuden on muututtava. Joko ajan (ja sekunnin) määritelmä ei päde kaikissa mahdollisissa värähtelyolosuhteissa, ja määritelmä ei ole yleispätevä, tai värähtely ei ole samanlaista eri painovoimakentissä tai suurissa nopeuksissa.

        Olisi helppo ajatella, että kappaleen nopeuden kasvaessa lähelle teoreettista maksimia tai painovoiman lähestyessä äärettömyyttä, atomin värähtelytaajuus muuttuisi. Onko tämä arkijärjellä ajateltu asia matemaattisesti aivan liian yksinkertainen ja pelkkä oikotie? Pysyykö atomin värähtelytaajuus siis aina samana?

        "Olisi helppo ajatella, että kappaleen nopeuden kasvaessa lähelle teoreettista maksimia tai painovoiman lähestyessä äärettömyyttä, atomin värähtelytaajuus muuttuisi. Onko tämä arkijärjellä ajateltu asia matemaattisesti aivan liian yksinkertainen ja pelkkä oikotie? Pysyykö atomin värähtelytaajuus siis aina samana?"

        Tässä ja edeltävissä saman oloisissa ihmettelyissä on se vika että oletetaan olemassaolevaksi jokin globaali koordinaatisto vaikka sellaista ei ole. Kaikki on kiinni siitä mistä käsin kelloa katsotaan eikä ole olemassa mitään muita tärkeämpää katsomispaikkaa, josta käsin kellojen käyntiä voisi absoluuttisesti arvostella.

        Jos nyt ajatellaan vaikka sitä kovaa vauhtia kulkevaa avaruusmatkailijaa niin hänen atomikellonsa käy täysin normaalisti, eli niin paljon värähdyksiä sekunnissa kuin kuuluukin. Saman havainnon tekee maassa olija omasta kellostaan. Mutta jos maassa olija millä hyvänsä systeemillä saa katsotuksi avaruusmatkustajan kelloa, hän havaitsee sen kävän hitaammin kuin oma kellonsa. Ja vastavuoroisesti avaruusmatkailija näkee maassa olevan kellon käyvän hitaammin kuin oman kellonsa.


    • Anonyymi

      Atomin viritystilan ja perustilan välinen energian määrä on jotain, mitä eri nopeuksilla liikkuvat havaitsijat näkevät eri tavalla. Aluksi ongelmaa ajatellessa näkee sen, että kentät ytimen ja elektronin tekeminä muuttuvat:

      https://physics.stackexchange.com/questions/616114/do-ionization-energies-of-atoms-change-when-they-are-in-a-moving-reference-frame

      Tästä ei saa oikeaa ratkaisua. Tosiasiassa atomin liike muuttaisi tarvitun teorian relativistiseksi QM:ksi, eikä QM:ksi jossa on nämä uudet potentiaalit.

      On paljon helpompi antaa asioiden, kuten fotoninen syntyä siinä koordinaatistossa, joka on atomiin nähden paikallaan, ja muuttaa pelkkä fotoni toisesta koordinaatistosta nähdyksi.

      • Anonyymi

        "Tästä ei saa oikeaa ratkaisua."

        Niin.

        Joko atomin värähtelytaajuus muuttuu nopeuden tai painovoiman mukaan tai sekunnin määritelmä ei ole vakio. Siis kun lähtökohtana on tosiaika.


      • Anonyymi
        Anonyymi kirjoitti:

        "Tästä ei saa oikeaa ratkaisua."

        Niin.

        Joko atomin värähtelytaajuus muuttuu nopeuden tai painovoiman mukaan tai sekunnin määritelmä ei ole vakio. Siis kun lähtökohtana on tosiaika.

        Jos sekunnin määritelmässä lukee 'pidä cesium atomeja reilusti alle valonnopeudessa verrattuna itseesi', silloin sitä ei voi verrata mihinkään, mitä tapahtuu eri nopeudella liikkuvaan kelloon, tyyliin kumpi on oikeampi tai väärempi. Sekunnin määritelmässä lukee ainakin, että on nähty cesium atomiin liittyvä ns. taajuus. Jos tässä mittauksessa lukee, että cesium atomi oli paikallaan, se on sekunnin määritelmässä sisällä. Teorian eli siis atomin energian teorian mukaan, jos kokeilijat alkavat katsoa liikkuvia atomeja, he näkevät kaikki taajuudet nollan ja äärettömän välillä. Sellaisen kokeen tuloksessa voisi lukea ettei tullut sekunnin määritelmää tänäänkään.


      • Anonyymi
        Anonyymi kirjoitti:

        "Tästä ei saa oikeaa ratkaisua."

        Niin.

        Joko atomin värähtelytaajuus muuttuu nopeuden tai painovoiman mukaan tai sekunnin määritelmä ei ole vakio. Siis kun lähtökohtana on tosiaika.

        "Tosiaika" on nykytiedon mukaan kiinteä, eli sekunti on aina sekunnin mittainen sille, joka havannoi sitä kellon luona.


      • Anonyymi
        Anonyymi kirjoitti:

        "Tosiaika" on nykytiedon mukaan kiinteä, eli sekunti on aina sekunnin mittainen sille, joka havannoi sitä kellon luona.

        Näin juuri eli "tosiaika" on aina paikallista. Kaikkien muissa paikoissa sijaitsevien kellojen käyntinopeus sen sijaan vaihtelee sen mukaan, millaisessa gravitaatiopotentiaalissa, liiketilassa ja millä etäysyydellä ne ovat käyntinopeutta havainnoivaan nähden.

        Gravitaation vaikutuksesta tiedetään jo se, että Maapallon pinnalla jo yhden senttimetrin korkeusero riittää tuottamaan havaittavan eron kellon käyntinopeudessa.

        Sitten jos joku joskus onnistuu havainnoin osoittamaan virheen suhteellisuusteorioista aiheutuvissa muutoksissa kellojen käyntinopeuksissa niin tulee tarvetta korjata teorioita. Sitä odoteltaessa mennään nykyisillä teorioilla, kun ne näyttävät kestävän hyvinkin rajua testaamista.


    • Anonyymi

      Ajan perusyksikkö on sekunti, joka on absoluuttisesti sidottu Cs-133-atomin värähtelytaajuuteen. Näin on sovittu. Voimme puhua vaikkapa Maan tosiajasta ja aika tarkastetaan atomikelloista, joiden tarkkuus on ylivoimaisen hyvä. Meillä on tiedossa, että aika kuluu eri tavoin eri painovoimakentissä tai nopeuden muuttuessa.

      Jos ja kun ajan mittayksikkö on sekunti, aika on sidottu suoraan sekunnin määritelmään. Haluamme pitää kiinni tuosta yhdestä ja ainoasta ajan ja sekunnin määritelmästä, unohdamme kaikki muut mahdolliset ajan määritelmät. Sidomme sen sekuntiin.

      On kaksi identtistä atomikelloa, joista ensimmäinen käy Maan tosiaikaa, ja sovitaan sen näyttämä aika vertailuajaksi. Toinen kello kiertää huimaa nopeutta (0.90 x c) valtavassa painovoimakentässä (= avaruuskello). Haluamme näiden kahden kellon näyttävän samaa aikaa ja siten aivan samoja sekuntilukuja.

      Muuttuuko avaruuskellon atomin häiritsemättömän perustilan ylihienorakennesiirtymän taajuus, kun atomiin kohdistuu suuri nopeus ja suuri nopeus?

      (A) Jos ko. taajuus muuttuu, avaruuskellossa voidaan käyttää matemaattista korjauskerrointa siten, että molemmat kellot näyttäjät tarkastelijalle yhtä-ja-samaa Maan tosiaikaa. Korjauskerroin saadaan mittaamalla Cs-atomin värähtelytaajuus ko. olosuhteissa.

      (B) Jos ko. taajuus pysyy samana, ja avaruuskellon aika poikkeaa Maan tosiajasta, sekunnin määritelmä ei ole pätevä ko. avaruuskellon olosuhteissa: sekunnin määritelmä olisi pätevä vain maan olosuhteissa. Millä kaavalla avaruuskellon näyttämää lukemaa olisi korjattava, että avaruuskello näyttäisi Maan tosiaikaa?

      Kaikille lienee joka tapauksessa selvää, että aika on sopimuskysymys, ja vain imistä kiinnostavat sekunnit. Luonto ei tunne sekunteja. Ihminen tarkastelee aikaa aina omalta kannaltaan. On kuitenkin mahdollista, että avaruuteen siirtyvä Star Trek -sukupolvi syntyy sadan vuoden sisällä. Silloin saattaa olla merkitystä sillä, että kaksi kelloa eri paikoissa näyttäisi aina samaa aikaa.

    • Anonyymi

      Gravitaatio ei vaikuta aikaan mitenkään, vaan laitteisiin. Laitteisiin kohdistuneet voimat häiritsee ajanmittausta teknisesti. Tämä ei ole muutos ajassa, mutta se tulkitaan sellaiseksi, koska taustalla vaikuttaa sitkeä oletus ajan olemassaolosta. Aikadilaatio = tekninen toiminnan muutos.

      Gravitaatio vaikuttaa myös fotoneihin. Kun fotonien liike hidastuu se vaikuttaa havaitsijassa tai laitteessa informaation saannin nopeuteen. Tämäkin koetaan muutoksena ajassa, vaikka kyse on muutoksesta informaation kulussa.

      Havainnollistus: pöydällä olevan kellon valonäyttö näyttää 12:00:00. Kun siitä lähtevät fotonit havaitaan valominuutin päässä, eli nähdään "12:00:00", näyttää kyseinen kello havaintohetkellä 12:01:00. Ts. informaation kulku vie "aikaa". Kun kellon ja havaitsijan välillä tapahtuu voimakas gravitaatio, muuttuu fotonien liike, esim. hidastuu. Jos tämä hidastuminen havaitaan, se koetaan ajan hidastumisena. Kyse ei kuitenkaan ole ajan hidastumisesta, vaan informaation kulun hidastumisesta.

      Aikaa ei ole olemassa itsenäisenä elementtinä tai voimana. Se on erillinen "päälle liimattu" sovellus.

      • Anonyymi

        Eli millä tavoin kaksi tarkkaa atomikelloa saadaan näyttämään Maan tosiaikaa, kun yksi kelloista on Maassa ja toinen suuressa nopeudessa ja suuressa painovoimassa avaruudessa?

        Jos kyseessä olisi Star Trekin Enterprise -alus, kello pitäisi tietenkin kytkeä kiihtyvyys-, painovoima- ja nopeusantureihin. Mutta millä tavoin kello korjaa omaa avaruusaikaansa näyttääkseen Maan tosiaikaa - mikä on korjauskerroin tai kaava. On ymmärrettävä että sekunti Maan päällä ei ole sekunti avaruudessa.


      • Anonyymi
        Anonyymi kirjoitti:

        Eli millä tavoin kaksi tarkkaa atomikelloa saadaan näyttämään Maan tosiaikaa, kun yksi kelloista on Maassa ja toinen suuressa nopeudessa ja suuressa painovoimassa avaruudessa?

        Jos kyseessä olisi Star Trekin Enterprise -alus, kello pitäisi tietenkin kytkeä kiihtyvyys-, painovoima- ja nopeusantureihin. Mutta millä tavoin kello korjaa omaa avaruusaikaansa näyttääkseen Maan tosiaikaa - mikä on korjauskerroin tai kaava. On ymmärrettävä että sekunti Maan päällä ei ole sekunti avaruudessa.

        Monessa puheenvuorossa näkyy kirjoitetun atomikelloista,sekunnin määritelmästä, sekunnin "pituudesta" j.n.e.

        Se osoittaa, että kirjoittajilla ei ole mitään käsitystä EST:sta tai YST:sta vaan he pohtivat asiaa arkijärkensä ja newtonilaisen maailmankuvan pohjalta.

        Tuollaista aprikointua voi jatkaa loputtomasti ilman ettö se johtaa mihinkään järkevään tulokseen. Aina kysytään lisää: "Mutta entä jos...?" ja tarina jatkuu.

        Emme me tiedä minkälainen maailma todellisuudessa on. Mitä tuo kysymys mahtaa edes tarkoittaa? Sen sijaan luodaan teorioita asiasta ja niitä pyritään kokeilla ja havainnoilla joko vahvistamaan tai kumoamaan. Tähän mennessä tehdyt kokeet nuo Einsteinin teoriat ovat kestäneet hyvin, jopa eittäin hyvin. Sen sijaan monet kilpailevat teoriat nuo kokeet ja havainnot ovat osoittaneet vääriksi.

        Tietenkin tulevaisuudessa ihmiskunta voi keksiä jotain vielä parempaa kuin Einstein, mutta uusien teorioidenkin on annettava ainakin tietyillä parametrien alueilla hyvin samanlaisia tuloksia kuin EST tai YST siksi, että nuo kokeet ovat osoittaneet mitä tuloksia on nsaatava.


      • Anonyymi
        Anonyymi kirjoitti:

        Monessa puheenvuorossa näkyy kirjoitetun atomikelloista,sekunnin määritelmästä, sekunnin "pituudesta" j.n.e.

        Se osoittaa, että kirjoittajilla ei ole mitään käsitystä EST:sta tai YST:sta vaan he pohtivat asiaa arkijärkensä ja newtonilaisen maailmankuvan pohjalta.

        Tuollaista aprikointua voi jatkaa loputtomasti ilman ettö se johtaa mihinkään järkevään tulokseen. Aina kysytään lisää: "Mutta entä jos...?" ja tarina jatkuu.

        Emme me tiedä minkälainen maailma todellisuudessa on. Mitä tuo kysymys mahtaa edes tarkoittaa? Sen sijaan luodaan teorioita asiasta ja niitä pyritään kokeilla ja havainnoilla joko vahvistamaan tai kumoamaan. Tähän mennessä tehdyt kokeet nuo Einsteinin teoriat ovat kestäneet hyvin, jopa eittäin hyvin. Sen sijaan monet kilpailevat teoriat nuo kokeet ja havainnot ovat osoittaneet vääriksi.

        Tietenkin tulevaisuudessa ihmiskunta voi keksiä jotain vielä parempaa kuin Einstein, mutta uusien teorioidenkin on annettava ainakin tietyillä parametrien alueilla hyvin samanlaisia tuloksia kuin EST tai YST siksi, että nuo kokeet ovat osoittaneet mitä tuloksia on nsaatava.

        Oletko yksinkertainen?

        Kysymys ei ole siitä, että tässä yhteydessä suhteellisuus- tai kvanttiteorioista epäiltäisiin mitään. Ne pitävät paikkansa! Jos olet niin viisas, kuin kuvittelet olevasi, kerro millä korjauskertoimella tai kaavalla kuvatut kaksi kellot saadaan näyttämään samaa aikaa!

        Etkö ymmärrä, että ne kellot ns. kellot käyvät eri aikaa - sekunti Maassa ei ole sama kuin sekunti avaruudessa ko. olosuhteissa, vaikka sekunnin määritelmä on sama? Sekunti puolestaan on ihmisen keksintöä, ymmärrätkö? Luonto tai fysiikka ei ole luonut aikaa tai sekuntia, ihminen on tosin luonut paljon mittayksiköitä omiin tarpeisiinsa. Sekunnille on ihmisen asettama tarve.

        Mistä löydämme sen kaavan, jolla avaruuskellon näyttö korjataan Maan tosiaikaan? Mitä kohtaa sinä et kysymyksessä ymmärrä?


      • Anonyymi
        Anonyymi kirjoitti:

        Oletko yksinkertainen?

        Kysymys ei ole siitä, että tässä yhteydessä suhteellisuus- tai kvanttiteorioista epäiltäisiin mitään. Ne pitävät paikkansa! Jos olet niin viisas, kuin kuvittelet olevasi, kerro millä korjauskertoimella tai kaavalla kuvatut kaksi kellot saadaan näyttämään samaa aikaa!

        Etkö ymmärrä, että ne kellot ns. kellot käyvät eri aikaa - sekunti Maassa ei ole sama kuin sekunti avaruudessa ko. olosuhteissa, vaikka sekunnin määritelmä on sama? Sekunti puolestaan on ihmisen keksintöä, ymmärrätkö? Luonto tai fysiikka ei ole luonut aikaa tai sekuntia, ihminen on tosin luonut paljon mittayksiköitä omiin tarpeisiinsa. Sekunnille on ihmisen asettama tarve.

        Mistä löydämme sen kaavan, jolla avaruuskellon näyttö korjataan Maan tosiaikaan? Mitä kohtaa sinä et kysymyksessä ymmärrä?

        Tuo löytyy GPS-satelliitin käyttöohjeesta. Katso sieltä. Tänne sitä on turha kopioida.


      • Anonyymi
        Anonyymi kirjoitti:

        Tuo löytyy GPS-satelliitin käyttöohjeesta. Katso sieltä. Tänne sitä on turha kopioida.

        Vai että GPS-satelliitin käyttöohje ottaa huomioon 0.9-kertaisen valonnopeuden ja valtavan painovoiman. Älä naurata.


      • Anonyymi
        Anonyymi kirjoitti:

        Vai että GPS-satelliitin käyttöohje ottaa huomioon 0.9-kertaisen valonnopeuden ja valtavan painovoiman. Älä naurata.

        0.9-kertainen valonnopeus ja valtava painovoima vaikuttaa ainoastaan sinun märkien sahajauhojen täyttämään pääkoppaasi. Ei mihinkään muualle.


      • Anonyymi
        Anonyymi kirjoitti:

        0.9-kertainen valonnopeus ja valtava painovoima vaikuttaa ainoastaan sinun märkien sahajauhojen täyttämään pääkoppaasi. Ei mihinkään muualle.

        Luulit tietäväsi paljon, mutta olit väärässä. Sinä et vain osaa.


      • Anonyymi
        Anonyymi kirjoitti:

        0.9-kertainen valonnopeus ja valtava painovoima vaikuttaa ainoastaan sinun märkien sahajauhojen täyttämään pääkoppaasi. Ei mihinkään muualle.

        Mikäs nyt meni ihon alle?


      • Anonyymi
        Anonyymi kirjoitti:

        Oletko yksinkertainen?

        Kysymys ei ole siitä, että tässä yhteydessä suhteellisuus- tai kvanttiteorioista epäiltäisiin mitään. Ne pitävät paikkansa! Jos olet niin viisas, kuin kuvittelet olevasi, kerro millä korjauskertoimella tai kaavalla kuvatut kaksi kellot saadaan näyttämään samaa aikaa!

        Etkö ymmärrä, että ne kellot ns. kellot käyvät eri aikaa - sekunti Maassa ei ole sama kuin sekunti avaruudessa ko. olosuhteissa, vaikka sekunnin määritelmä on sama? Sekunti puolestaan on ihmisen keksintöä, ymmärrätkö? Luonto tai fysiikka ei ole luonut aikaa tai sekuntia, ihminen on tosin luonut paljon mittayksiköitä omiin tarpeisiinsa. Sekunnille on ihmisen asettama tarve.

        Mistä löydämme sen kaavan, jolla avaruuskellon näyttö korjataan Maan tosiaikaan? Mitä kohtaa sinä et kysymyksessä ymmärrä?

        Kaavat löytyvät Minkowskilta (erityinen suhteellisuusteoria) ja einsteinilaisesta painovoimateoriasta. Haasteena on se, että painovoimateorian dimensoiden välisiä ei-lineaarisia riippuvuussuhteita on kuusi, mutta tunnemme vain viisi dimensiota. Siten ratkaisut perustuvat aina myös empiriaan käyttötilanteesta ja niiden todellisesta tarkkuudesta on epävrmuutta. Tosin Minkowskilaista eli painovoimatonta avaruutta lähellä olevissa ympäristöissä kuten GPS-satelliiteissä päästään käytännön sovelluksiin riittävään tarkkuuteen.


      • Anonyymi
        Anonyymi kirjoitti:

        Oletko yksinkertainen?

        Kysymys ei ole siitä, että tässä yhteydessä suhteellisuus- tai kvanttiteorioista epäiltäisiin mitään. Ne pitävät paikkansa! Jos olet niin viisas, kuin kuvittelet olevasi, kerro millä korjauskertoimella tai kaavalla kuvatut kaksi kellot saadaan näyttämään samaa aikaa!

        Etkö ymmärrä, että ne kellot ns. kellot käyvät eri aikaa - sekunti Maassa ei ole sama kuin sekunti avaruudessa ko. olosuhteissa, vaikka sekunnin määritelmä on sama? Sekunti puolestaan on ihmisen keksintöä, ymmärrätkö? Luonto tai fysiikka ei ole luonut aikaa tai sekuntia, ihminen on tosin luonut paljon mittayksiköitä omiin tarpeisiinsa. Sekunnille on ihmisen asettama tarve.

        Mistä löydämme sen kaavan, jolla avaruuskellon näyttö korjataan Maan tosiaikaan? Mitä kohtaa sinä et kysymyksessä ymmärrä?

        Olen kyllä yksinkertainen mutta silti jonkin verran perehtynyt EST:aan ja YST:aan.

        Sinun kysymyksistäsi näkyy, ettet ymmärrä mitä esim. EST sanoo vaan ajattelet newtonilaisittain.

        Kun maakaksosen raami on R ja matkaajan raami on R' ja R' liikkuu raamissa R nopeudella v, niin maakaksonen jakaa 4-ulotteisen avaruusajan (spacetime) avaruus- ja aikakomponentteihin eri tavalla kuin matkaaja R'-raamissaan.Näin syntyvät pituuskontraktio ja aikadilataatio.

        Tapahtumien simultaanisuus on myös eri R- ja R'-raameissa. Kun maakaksonen mittaa sauvan pituuden hetkellä t, molemmat päät samaan aikaan, niin nuo hetket eivät ole simultaaniset matkaajan ajassa. He mittaavat eri asioita.

        Mutta tämä pitää ymmärtää perehtymällä EST:n matematiikkaan. Tällaiset sanalliset selitykset ovat aina vain kalpea kuva asiasta ja jättävät monasti mahdollisuuden kaikenlaisiin "entä jos"-kysymyksiin ja selostuksiin milloin minkinlaisista härveleistä ja kokeista joita kysyjälle pitäisi selitellä.


      • Anonyymi
        Anonyymi kirjoitti:

        Olen kyllä yksinkertainen mutta silti jonkin verran perehtynyt EST:aan ja YST:aan.

        Sinun kysymyksistäsi näkyy, ettet ymmärrä mitä esim. EST sanoo vaan ajattelet newtonilaisittain.

        Kun maakaksosen raami on R ja matkaajan raami on R' ja R' liikkuu raamissa R nopeudella v, niin maakaksonen jakaa 4-ulotteisen avaruusajan (spacetime) avaruus- ja aikakomponentteihin eri tavalla kuin matkaaja R'-raamissaan.Näin syntyvät pituuskontraktio ja aikadilataatio.

        Tapahtumien simultaanisuus on myös eri R- ja R'-raameissa. Kun maakaksonen mittaa sauvan pituuden hetkellä t, molemmat päät samaan aikaan, niin nuo hetket eivät ole simultaaniset matkaajan ajassa. He mittaavat eri asioita.

        Mutta tämä pitää ymmärtää perehtymällä EST:n matematiikkaan. Tällaiset sanalliset selitykset ovat aina vain kalpea kuva asiasta ja jättävät monasti mahdollisuuden kaikenlaisiin "entä jos"-kysymyksiin ja selostuksiin milloin minkinlaisista härveleistä ja kokeista joita kysyjälle pitäisi selitellä.

        Kysymys ei ollut mistään kaksosista, ne eivät kysyttyä asiaa selitä.


      • Anonyymi
        Anonyymi kirjoitti:

        Kaavat löytyvät Minkowskilta (erityinen suhteellisuusteoria) ja einsteinilaisesta painovoimateoriasta. Haasteena on se, että painovoimateorian dimensoiden välisiä ei-lineaarisia riippuvuussuhteita on kuusi, mutta tunnemme vain viisi dimensiota. Siten ratkaisut perustuvat aina myös empiriaan käyttötilanteesta ja niiden todellisesta tarkkuudesta on epävrmuutta. Tosin Minkowskilaista eli painovoimatonta avaruutta lähellä olevissa ympäristöissä kuten GPS-satelliiteissä päästään käytännön sovelluksiin riittävään tarkkuuteen.

        > Haasteena on se, että painovoimateorian dimensoiden välisiä ei-lineaarisia riippuvuussuhteita on kuusi, mutta tunnemme vain viisi dimensiota.

        OK. Osaatko kertoa nuo dimensiot ja riippuvuussuhteet suoraan tai haluaisitko antaa numerointiin linkin?


      • Anonyymi kirjoitti:

        Vai että GPS-satelliitin käyttöohje ottaa huomioon 0.9-kertaisen valonnopeuden ja valtavan painovoiman. Älä naurata.

        " GPS-satelliitin käyttöohje ottaa huomioon 0.9-kertaisen valonnopeuden ja valtavan painovoiman. "

        Ei tarvita 0.9 kertaista valonnopeutta ja valtavaa painovoimaa, vaan jo ne pienemmät erot kun verrataan satelliittia vs. ihminen maapallolla aiheuttaa tarpeeksi suuren eroavaisuuden että se otetaan huomioon GPS-koordinaatteja laskiessa kun tämänkaltaisessa triangulaatiossa pitää tietää täsmälleen miten kauan sitten eri signaalit lähetettiin.


      • Anonyymi
        Anonyymi kirjoitti:

        > Haasteena on se, että painovoimateorian dimensoiden välisiä ei-lineaarisia riippuvuussuhteita on kuusi, mutta tunnemme vain viisi dimensiota.

        OK. Osaatko kertoa nuo dimensiot ja riippuvuussuhteet suoraan tai haluaisitko antaa numerointiin linkin?

        Minkowskilaisessa metriikassa dimensiot ovat joko xyz-koordinaatisto tai napakoordinaatisto, ct ja massa. Riippuvuussuhteet klassisessa einsteinilaisessa painovoimateorissa määrittävät Cristoferssenin symbolit.

        Jos olet oikeasti kiinnostunut asiasta, aluksi kannattaa lukea:

        https://arxiv.org/pdf/1503.02172.pdf

        Sen ymmärtämiseen riittää lukion pitkä matematiikka ja hieman googlausta käytetyistä symboleista.


      • Anonyymi
        Anonyymi kirjoitti:

        Minkowskilaisessa metriikassa dimensiot ovat joko xyz-koordinaatisto tai napakoordinaatisto, ct ja massa. Riippuvuussuhteet klassisessa einsteinilaisessa painovoimateorissa määrittävät Cristoferssenin symbolit.

        Jos olet oikeasti kiinnostunut asiasta, aluksi kannattaa lukea:

        https://arxiv.org/pdf/1503.02172.pdf

        Sen ymmärtämiseen riittää lukion pitkä matematiikka ja hieman googlausta käytetyistä symboleista.

        Siis Cristoffel-symbolit, ei Cristoferssen.


    • Avaaja: Avauksestasi puuttuu yksi olennainen kysymys tämän ymmärtämiseen,
      "Mikä aika on?"

      Ja vastauksena siihen, se on sekä temporaalinen ulottuvuus että laskennallinen suure jota käytetään kun tahdotaan katsoa miten paljon aikaa tietyn asian tapahtuminen ottaa.
      Se, onko kyseessä Cesium-atomin viritystasojen purkautuminen tai esim .tietyn matkan käveleminen tietyllä nopeudella ei ole huomattavaa eroa. Sekunti on aina sekunti, vaikka jos verrataan eri paikkoja avaruudessa, saattaa niiden sekuntien pituus erota toisistaan.

      "Yksi sekunti on siis sidottu Cs-133-atomin värähtelyyn."

      Tämä on vain hyvin helppo ja luotettava tapa mitata ja havainnollistaa sekunti. Teoriassa voisit käyttää vaikka kalibroidun pituuden ja omaavaa heiluria ja se toimisi yhtä hyvin, vaikka vähemmän tarkasti.

      "Herää kysymys siitä, että jos avaruudessa matkataan todella suurella nopeudella - sovitaan 90 % valonnopeudesta - tai todella suuressa painovoimakentässä, myös Cs-133-atomin värähtelytaajuuden on muututtava tai määritelmää värähtelylle on avaruudessa muutettava. "

      Ei ole mitään järkeä muuttaa värähtelytaajuutta koska se ottaa saman määrän sekunteja myös kun verrataan mihin tahansa muuhun reaktioon joka ottaa tunnetun määrän aikaa.
      Eli siis vaikka nopeus muuttuu ja ulkopuolisen kannalta näyttäisi että ajankulku on muuttunut... Avaruusmatkaajana itse kun mittaat niin sekunti on aina sekunnin pituinen ja siinä tapahtuu oletettu määrä asioita.

      Tämä siis tarkoittaa että vaikka aika pysyy aina samana kokijalle, ei ole olemassa absoluuttista aikaa vaan se muuttuu riippuen näkijästä.

      "Avaruusmatkailija ja maahan jäänyt kollega tuntevat omassa koordinaatistossaan ajan kuluvan samalla tavoin,..."

      Tähän mennessä menee aivan oikein! :)

      "... vaikka todellisuudessa näin ei ole:"

      Mutta tässä taas menee hakoteille. Ei ole olemassa "oikeaa" tai absoluuttista aikaa jota pitäisi noudattaa koska havainnoitsijalle kaikki tapahtuu suhteessa omaan aikaansa.

      "kaksi identtistä atomikelloa kykenevät näyttämään todellisen eron."

      Nämä pystyvät kertomaan miten suuri ero ajoissa on, mutta niiden vertaileminenkin tuo omat haasteensa. Mutta se tärkeämpi juttu muistaa on että "maan aika" ei ole yhtään oikeampi kuin "avaruusmatkustajan aika", tästä siis tulee nimitys "suhteellisuusteoria".

      "Koska aika on sidottu atomikellon värähtelyyn, mutta ko. avaruuden olosuhteissa aika kuluu hitaammin, atomikellon värähtelytaajuuden on myös muututtava, tai Maassa määritelty kerroin on väärin."

      90% valonnopeudessa kulkevalle aika kulkee silti samaa nopeutta kuin muillekin, eli yksi sekunti per sekunti. Tämän määritelmän mukaan atomin värähtely pysyy myös samana kuin mikä mitattiin maassa. Eli siis kerroin on absoluuttinen, vaikka aika itsessään ei ole absoluuttisesti sama kaikkialla.

      "(1) Mihin fysiikan lakiin perustuu se, että Cs-133-atomin värähtely muuttuu kellon matkatessa - avaruudessa - suurta nopeutta (tai suuressa painovoimakentässä)?"

      Atomin värähtely pysyy molemmissa tapauksissa samana oli nopeutena tai kiihtyvyytenä mikä tahansa.
      Se mitä varmaan tarkoitit on "Minkä fysiikan lakien mukaan nopeus tai painovoimakenttä aiheuttavat että staattisesta katsojasta katsoen näyttää kuin aika kulkisi eri nopeutta?" niin vastaan tähän kysymykseen:

      Nopeasti matkatessa: Erityinen suhteellisuusteoria
      Suuressa kiihtyvyydessä / painovoimakentässä: Yleinen suhteellisuusteoria

      "(2) Jos avaruusmatkaaja haluaisi pysyä aina yhdessä ja samassa Maan tosiajassa,"

      Tämä ei ole mahdollista, sillä fysikaalista 'tosiaikaa' ei ole olemassa. Jos tahdot voit tietenkin yrittää laskea ja seurata mitä aika olisi esim. Maapallolla, mutta sillä ei olisi mitään merkitystä matkaajan kannalta.

      "minkä kaavan mukaisesti hänen tulisi etsiä kerroin, jolla atomikellon värähtelytaajuus kerrotaan yhden sekunnin määritelmän mukaiseksi?"

      Ole hyvä :)
      https://en.wikipedia.org/wiki/Time_dilation

      • Anonyymi

        "Ole hyvä :)
        https://en.wikipedia.org/wiki/Time_dilation"

        Näitä kaavoja ei pidä käyttää aloittajan kysymyksessä numero (2), koska kysymys koskee sitä, mikä on minkä tahansa avaruusmatkaajan, jonka nopeus on 0.9c, tekemä kellojen tahdistus. Siinä pitää selvästi tietää hänen reittinsä kaikkien tähtien ja mustienaukkojen ('voimakkaat gravitaatiokentät') ohi tähän mennessä. Pelkkä maan gravitaatiokenttä ja riippuvuus etäisyydeestä maahan ei riitä.


      • Anonyymi
        Anonyymi kirjoitti:

        "Ole hyvä :)
        https://en.wikipedia.org/wiki/Time_dilation"

        Näitä kaavoja ei pidä käyttää aloittajan kysymyksessä numero (2), koska kysymys koskee sitä, mikä on minkä tahansa avaruusmatkaajan, jonka nopeus on 0.9c, tekemä kellojen tahdistus. Siinä pitää selvästi tietää hänen reittinsä kaikkien tähtien ja mustienaukkojen ('voimakkaat gravitaatiokentät') ohi tähän mennessä. Pelkkä maan gravitaatiokenttä ja riippuvuus etäisyydeestä maahan ei riitä.

        Ihanko varmasti pitää tietää reitti selvästi? Ettet vain narraa.


      • Anonyymi
        Anonyymi kirjoitti:

        Ihanko varmasti pitää tietää reitti selvästi? Ettet vain narraa.

        Kiihtyvyys ja gravitaatio vaikuttavat kellon käyntiin joten koko reitti on tunnettava. Muutenhan reittiin voisi kuulua esimerkiksi ulkopuolelta havainnoivan mielestä vuoden pituinen sukellus lähelle mustan aukon tapahtumahorisonttia, johon sukeltajan itsensä mielestä kului vain vuorokauden verran aikaa.


    • Anonyymi

      >>> Siinä pitää selvästi tietää hänen reittinsä kaikkien tähtien ja mustienaukkojen ('voimakkaat gravitaatiokentät') ohi tähän mennessä. Pelkkä maan gravitaatiokenttä ja riippuvuus etäisyydeestä maahan ei riitä.

      Juuri näin. Edellä on ehdotettu, että avaruuskello saataisiin näyttämään Maan tosiaikaa, kun Star Trekin Enterprise -aluksessa laskennassa otetaan huomioon ainakin aluksen kiihtyvyys, nopeus ja painovoima. Näillä voitaisiin tulkita aikaan tunnetusti vaikuttavat tekijät, kuten aluksen nopeus ja reitti suhteessa painovoimaan. Voimakkaat gravitaatiokentät olisi siis huomioitava.

      Kaksi kelloa eli "Maan tosiaikakello" ja avaruuskello ns. käyvät eri aikaa - sekunti Maassa ei ole sama kuin sekunti avaruudessa ko. olosuhteissa, vaikka sekunnin määritelmä on sama. Kellon näytöllä on eri lukema, eikä merkitystä ole sillä, miten "kaksoset" ihmisinä tuntevat ajan. Sekunti on vain ja ainoastaan sidottu atomin värähtelyyn. Sekunti on ihmisen keksintöä, kuten on myös vaikkapa metri. Luonto tai fysiikka ei ole luonut aikaa tai sekuntia, ihminen on joutunut luomaan paljon mittayksiköitä omiin tarpeisiinsa. Sekunnille, samoin kuin metrille tai jaardille, on ihmisen asettama yhteismitallinen tarve. Luonto ja fysiikka toimivat ihmisen tahdosta riippumatta, ihmisellä on ainoastaan tarve ymmärtää.

      Kysymys avauksessa on vain siitä, että kahden identtisen atomikellon (Maan tosiaika ja avaruuskello) näytöt eivät näytä identtistä aikaa, vaikka sekunti on yksiselitteisesti määritelty. Haussa on siis ollut kaava, jolla Star Trekin Enterprisen avaruuskello ja Maan tosiaika -kello kykenisivät molemmat näyttämään yhtä ja samaa aikaa, samaa sekuntinäyttöä: Maan tosiaikaa.

      Kaikki kirjoittavat tuntuvat olevan yhtä mieltä siitä, että Cs-133-atomin värähtelytaajuus on yksi ja sama riippumatta atomiin kohdistuvasta nopeudesta tai painovoimasta. Aikadilataatio toimii siis mysteerisesti, vaikka puhumme atomikellon osalta yhdestä ja samasta asiasta, värähtelystä.

      --

      Kiitos henkilölle, joka kertoi tämän: "Tästä eteenpäin avaruus itsessään ja aika itsessään ovat tuomittuja haalistumaan pelkiksi varjoiksi ja vain näiden kahden eräänlainen liitto voi säilyttää riippumattoman todellisuuden." (Minkowski 1908)

      • Anonyymi

        "Kaikki kirjoittavat tuntuvat olevan yhtä mieltä siitä, että Cs-133-atomin värähtelytaajuus on yksi ja sama riippumatta atomiin kohdistuvasta nopeudesta tai painovoimasta. Aikadilataatio toimii siis mysteerisesti"

        Sävellä lisää huuhaajuttuja niin voit pienen harjoittelun jälkee ruveta ammattimaiseksi hörhöjuttujen säveltäjäksi.


      • Anonyymi
        Anonyymi kirjoitti:

        "Kaikki kirjoittavat tuntuvat olevan yhtä mieltä siitä, että Cs-133-atomin värähtelytaajuus on yksi ja sama riippumatta atomiin kohdistuvasta nopeudesta tai painovoimasta. Aikadilataatio toimii siis mysteerisesti"

        Sävellä lisää huuhaajuttuja niin voit pienen harjoittelun jälkee ruveta ammattimaiseksi hörhöjuttujen säveltäjäksi.

        Mihin fysiikan lakiin perustuu se, että Cs-133-atomin värähtelytaajuus muuttuu atomikellon matkatessa - avaruudessa - suurta nopeutta tai suuressa painovoimakentässä? Olisiko sinulla tuohon kaavaa antaa, miten värähtely muuttuu? Jos näin on, se selittäisi tosi monta asiaa tässä säikeessä.


      • Anonyymi
        Anonyymi kirjoitti:

        Mihin fysiikan lakiin perustuu se, että Cs-133-atomin värähtelytaajuus muuttuu atomikellon matkatessa - avaruudessa - suurta nopeutta tai suuressa painovoimakentässä? Olisiko sinulla tuohon kaavaa antaa, miten värähtely muuttuu? Jos näin on, se selittäisi tosi monta asiaa tässä säikeessä.

        Tuskinpa tuohon kenenkään lainlaatijan tarvitsee ruveta mitään lakeja säätämään. Fysiikan lakeja tarvitsevat ne, jotka eivät asiasta oikeasti mitään ymmärrä ja tarvitsevat tentin läpäisemiseen jonkun helpon laskentakaavan.

        Atomikellojen käyntinopeuden on mittaamalla havaittu muuttuvan painovoimakentässä ja kellon liikkuessa. Suhteellisuusteoriasta on selitystä yritetty löytää ja usein se on onnistunut melko hyvällä menestyksellä.

        Suosittelen tutustumista suhteellisuusteoriaan. Aloittaa voi vaikkapa Feynmanin opuksesta "Suhteellisen helppoa". Löytynee kirjastosta.


      • Anonyymi
        Anonyymi kirjoitti:

        Tuskinpa tuohon kenenkään lainlaatijan tarvitsee ruveta mitään lakeja säätämään. Fysiikan lakeja tarvitsevat ne, jotka eivät asiasta oikeasti mitään ymmärrä ja tarvitsevat tentin läpäisemiseen jonkun helpon laskentakaavan.

        Atomikellojen käyntinopeuden on mittaamalla havaittu muuttuvan painovoimakentässä ja kellon liikkuessa. Suhteellisuusteoriasta on selitystä yritetty löytää ja usein se on onnistunut melko hyvällä menestyksellä.

        Suosittelen tutustumista suhteellisuusteoriaan. Aloittaa voi vaikkapa Feynmanin opuksesta "Suhteellisen helppoa". Löytynee kirjastosta.

        Olipa tuossa vaatimaton vastaus. Kirjoittajan täysin turha ylimielisyys osoittaa, että tieto ei ole saavuttanut vielä hiusten alapuolista kerrosta.

        "Atomikellojen käyntinopeuden on mittaamalla havaittu muuttuvan painovoimakentässä ja kellon liikkuessa."

        Mitä sitten - tämä on ollut jo kaikille jo 100 % selvää avauksesta saakka? Kysymys oli viimeksi siitä, muuttuuko Cs-133-atomin värähtelytaajuus atomiin kohdistuvasta nopeudesta tai painovoimasta johtuen.

        Miksi siis puhut asian vierestä? Aikadilataatiossa ei ole tähän saakka mitään epäselvää ollut kenelläkään. Ellet ymmärrä kysymystä tai et vaivaudu sitä lukemaan, voit todellakin ja toiveesta pysyä poissa!

        --

        Tässä muiden näkemyksiä värähtelystä.

        Lainaus: "Atomin värähtelytaajuus ei tavallaan muutu, vaan itse aika. Perustuu yleiseen suhteellisuusteoriaan, ja sen pohjalta tehtyihin havaintoihin."

        Lainaus: "Mutta rivien välistä voisi tulkita, että insinöörit katsovat kelloerojen syntyvän
        kellon taajuuden muuttumisesta, joka olisi helppo resetoida, kun taas fyysikot
        tulkitsevat että aika muuttuu."

        Lainaus: "Ei ole mitään järkeä muuttaa värähtelytaajuutta koska se ottaa saman määrän sekunteja myös kun verrataan mihin tahansa muuhun reaktioon joka ottaa tunnetun määrän aikaa."

        Lainaus: "90% valonnopeudessa kulkevalle aika kulkee silti samaa nopeutta kuin muillekin, eli yksi sekunti per sekunti. Tämän määritelmän mukaan atomin värähtely pysyy myös samana kuin mikä mitattiin maassa."


      • Anonyymi
        Anonyymi kirjoitti:

        Olipa tuossa vaatimaton vastaus. Kirjoittajan täysin turha ylimielisyys osoittaa, että tieto ei ole saavuttanut vielä hiusten alapuolista kerrosta.

        "Atomikellojen käyntinopeuden on mittaamalla havaittu muuttuvan painovoimakentässä ja kellon liikkuessa."

        Mitä sitten - tämä on ollut jo kaikille jo 100 % selvää avauksesta saakka? Kysymys oli viimeksi siitä, muuttuuko Cs-133-atomin värähtelytaajuus atomiin kohdistuvasta nopeudesta tai painovoimasta johtuen.

        Miksi siis puhut asian vierestä? Aikadilataatiossa ei ole tähän saakka mitään epäselvää ollut kenelläkään. Ellet ymmärrä kysymystä tai et vaivaudu sitä lukemaan, voit todellakin ja toiveesta pysyä poissa!

        --

        Tässä muiden näkemyksiä värähtelystä.

        Lainaus: "Atomin värähtelytaajuus ei tavallaan muutu, vaan itse aika. Perustuu yleiseen suhteellisuusteoriaan, ja sen pohjalta tehtyihin havaintoihin."

        Lainaus: "Mutta rivien välistä voisi tulkita, että insinöörit katsovat kelloerojen syntyvän
        kellon taajuuden muuttumisesta, joka olisi helppo resetoida, kun taas fyysikot
        tulkitsevat että aika muuttuu."

        Lainaus: "Ei ole mitään järkeä muuttaa värähtelytaajuutta koska se ottaa saman määrän sekunteja myös kun verrataan mihin tahansa muuhun reaktioon joka ottaa tunnetun määrän aikaa."

        Lainaus: "90% valonnopeudessa kulkevalle aika kulkee silti samaa nopeutta kuin muillekin, eli yksi sekunti per sekunti. Tämän määritelmän mukaan atomin värähtely pysyy myös samana kuin mikä mitattiin maassa."

        "Kysymys oli viimeksi siitä, muuttuuko Cs-133-atomin värähtelytaajuus atomiin kohdistuvasta nopeudesta tai painovoimasta johtuen."

        Miksi jankutat paskaa kun tuo nopeuden ja painovoiman vaikutus on jo moneen kertaan todettu. Onko korviesi välissä joku ongelma?


      • Anonyymi
        Anonyymi kirjoitti:

        "Kysymys oli viimeksi siitä, muuttuuko Cs-133-atomin värähtelytaajuus atomiin kohdistuvasta nopeudesta tai painovoimasta johtuen."

        Miksi jankutat paskaa kun tuo nopeuden ja painovoiman vaikutus on jo moneen kertaan todettu. Onko korviesi välissä joku ongelma?

        "Atomikellojen käyntinopeuden on mittaamalla havaittu muuttuvan painovoimakentässä ja kellon liikkuessa - mitään muuta et osaa sanoa."

        Tuo asia ei ole ollut kenellekään epäselvää, miten vaikeaa sinun on se ymmärtää? Jankutat samaa asiaa koko ajan, vaikka se on selvä.

        Sinulla on kovasti huonot hermot, et ilmeisesti ole opiskellut lainkaan yliopistolla, vaan olet lukenut asioita Tiede-lehdestä. Jos vastaisit kysymykseen, sinun ei tarvitsisi välittää minun korvien välistä, vaan omastasi.


      • Anonyymi
        Anonyymi kirjoitti:

        "Kysymys oli viimeksi siitä, muuttuuko Cs-133-atomin värähtelytaajuus atomiin kohdistuvasta nopeudesta tai painovoimasta johtuen."

        Miksi jankutat paskaa kun tuo nopeuden ja painovoiman vaikutus on jo moneen kertaan todettu. Onko korviesi välissä joku ongelma?

        Vastauksesi oli typerä ja jatkaa täällä inttämistä. Vastausta ei ole olemassa täällä foorumilla, koska osaamistakaan ei ole, vaan se pitäisi etsiä englanniksi tai saksaksi.


      • Anonyymi

        Siis kuten sinusta vai? En usko, se vaatii jo luontaisia kykyjä.


      • Anonyymi
        Anonyymi kirjoitti:

        Olipa tuossa vaatimaton vastaus. Kirjoittajan täysin turha ylimielisyys osoittaa, että tieto ei ole saavuttanut vielä hiusten alapuolista kerrosta.

        "Atomikellojen käyntinopeuden on mittaamalla havaittu muuttuvan painovoimakentässä ja kellon liikkuessa."

        Mitä sitten - tämä on ollut jo kaikille jo 100 % selvää avauksesta saakka? Kysymys oli viimeksi siitä, muuttuuko Cs-133-atomin värähtelytaajuus atomiin kohdistuvasta nopeudesta tai painovoimasta johtuen.

        Miksi siis puhut asian vierestä? Aikadilataatiossa ei ole tähän saakka mitään epäselvää ollut kenelläkään. Ellet ymmärrä kysymystä tai et vaivaudu sitä lukemaan, voit todellakin ja toiveesta pysyä poissa!

        --

        Tässä muiden näkemyksiä värähtelystä.

        Lainaus: "Atomin värähtelytaajuus ei tavallaan muutu, vaan itse aika. Perustuu yleiseen suhteellisuusteoriaan, ja sen pohjalta tehtyihin havaintoihin."

        Lainaus: "Mutta rivien välistä voisi tulkita, että insinöörit katsovat kelloerojen syntyvän
        kellon taajuuden muuttumisesta, joka olisi helppo resetoida, kun taas fyysikot
        tulkitsevat että aika muuttuu."

        Lainaus: "Ei ole mitään järkeä muuttaa värähtelytaajuutta koska se ottaa saman määrän sekunteja myös kun verrataan mihin tahansa muuhun reaktioon joka ottaa tunnetun määrän aikaa."

        Lainaus: "90% valonnopeudessa kulkevalle aika kulkee silti samaa nopeutta kuin muillekin, eli yksi sekunti per sekunti. Tämän määritelmän mukaan atomin värähtely pysyy myös samana kuin mikä mitattiin maassa."

        Anon.

        "Lainaus: "Mutta rivien välistä voisi tulkita, että insinöörit katsovat kelloerojen
        syntyvän
        kellon taajuuden muuttumisesta, joka olisi helppo resetoida, kun taas fyysikot
        tulkitsevat että aika muuttuu."

        GPS-kellojen tahdistuksesta vielä, kelloja ei tahdisteta Einsteinin tahdistuksen
        tapaan, vaan yhdellä "rysähdyksellä" resetoimalla kellojen taajuus.

        Tällä on tärkeä ero Einsteinin tahdistukseen, siinä siirretään ajallisia suhteita
        kellojen välillä, nimenomaan kahta suhdetta, näitä Einstein piti tärkeinä.

        Kello resetoinnissa ajalliset suhteet ovat erilaisia, jos ollenkaan aikaan
        liittyviä, vain kelloihin liittyviä.


      • Anonyymi
        Anonyymi kirjoitti:

        Anon.

        "Lainaus: "Mutta rivien välistä voisi tulkita, että insinöörit katsovat kelloerojen
        syntyvän
        kellon taajuuden muuttumisesta, joka olisi helppo resetoida, kun taas fyysikot
        tulkitsevat että aika muuttuu."

        GPS-kellojen tahdistuksesta vielä, kelloja ei tahdisteta Einsteinin tahdistuksen
        tapaan, vaan yhdellä "rysähdyksellä" resetoimalla kellojen taajuus.

        Tällä on tärkeä ero Einsteinin tahdistukseen, siinä siirretään ajallisia suhteita
        kellojen välillä, nimenomaan kahta suhdetta, näitä Einstein piti tärkeinä.

        Kello resetoinnissa ajalliset suhteet ovat erilaisia, jos ollenkaan aikaan
        liittyviä, vain kelloihin liittyviä.

        Lainaus: "Atomin värähtelytaajuus ei tavallaan muutu, vaan itse aika. Perustuu yleiseen suhteellisuusteoriaan, ja sen pohjalta tehtyihin havaintoihin."

        Lainaus: "Mutta rivien välistä voisi tulkita, että insinöörit katsovat kelloerojen syntyvän kellon taajuuden muuttumisesta, joka olisi helppo resetoida, kun taas fyysikot tulkitsevat että aika muuttuu."

        Lainaus: "Ei ole mitään järkeä muuttaa värähtelytaajuutta koska se ottaa saman määrän sekunteja myös kun verrataan mihin tahansa muuhun reaktioon joka ottaa tunnetun määrän aikaa."

        Lainaus: "90% valonnopeudessa kulkevalle aika kulkee silti samaa nopeutta kuin muillekin, eli yksi sekunti per sekunti. Tämän määritelmän mukaan atomin värähtely pysyy myös samana kuin mikä mitattiin maassa."

        --

        Lainaus: "GPS-kellojen tahdistuksesta vielä, kelloja ei tahdisteta Einsteinin tahdistuksen
        tapaan, vaan yhdellä "rysähdyksellä" resetoimalla kellojen taajuus...... Tällä on tärkeä ero Einsteinin tahdistukseen, siinä siirretään ajallisia suhteita kellojen välillä, nimenomaan kahta suhdetta, näitä Einstein piti tärkeinä."

        GPS-kellojen tapa tahdistaa olisi siis Star Trekin Enterprise-aluksella käyttökelvoton, koska alus haluaisi aina pysyä yhdessä ja samassa Maan tosiajassa. Siis jatkuvasti ja koko ajan. "Rysähdys" kerran tunnissa tai kerran vuorokaudessa oli valtavilla nopeuksilla tai painovoimakentissä liian harvoin.

        Kuten eräs anonyymi on todennut "siinä pitää selvästi tietää (aluksen) reitti kaikkien tähtien ja mustienaukkojen ('voimakkaat gravitaatiokentät') ohi tähän mennessä. Pelkkä maan gravitaatiokenttä ja riippuvuus etäisyydestä maahan ei riitä." Tuo toteamus on aivan totta, sillä yksikin poikkeaminen voimakkaassa gravitaatiokentässä riittäisi sekoittamaan avaruuskellon ajan totaalisesti Maan tosiajasta. Aluksen ja atomikellon tulisi siis jatkuvasti 24/7 analysoida kiihtyvyyttää, nopeuttaan sekä ympäristön painovoimaa.

        Jos lainattu anonyymin väite pitää paikkansa, Einsteinin tapa olisi oikea tapa, sillä siinä siirrettäisiin kellojen ajallisia suhteita.

        Lainaus: "Kello resetoinnissa ajalliset suhteet ovat erilaisia, jos ollenkaan aikaan
        liittyviä, vain kelloihin liittyviä."

        Enterprise-aluksen kohdalla kaikki ovat tietoisia siitä, että aika ei ole kiinnostava, koska se on avaruuden joka koordinaatistossa erilainen ja havainnoitsijasta riippuvainen. Vain aluksen kellonajan on haluttu vastaavan Maan tosiaikaa, ja avauksessa kiinnostavaa on siis vain kelloihin liittyvät suhteet.

        Edelleenkin jää eri kirjoittajien näkemyksistä epäselväksi, onko Cs-133-atomin värähtelytaajuus yksi ja sama riippumatta atomiin kohdistuvasta nopeudesta tai painovoimasta. Jos kellojen GPS-tahdistuksessa taajuus resetoidaan "insinöörien tapaan rysäyksinä", mikä estäisi tahdistamasta kelloja 24/7 Einsteinin tahdistuksen tapaan jatkuvasti? Ilmeisesti taajuus resetoidaan pelkästään käytännön syistä.

        --

        Jos eletään tähtiaikaa 8454.130, kapteeni Kirkillä on edessään lukuisia haasteita, jolloin Maan sekunnit tai tunnit eivät ole tärkeitä. Maan tosiaika on sivuseikka, kun käytössä on tähtiaika; hyvä että edes vuodet ovat selvillä. Poimuajo olisi mahdollista vain pimeän energian avulla. Riittävällä määrällä pimeää energiaa avaruusaluksen ympärille olisi mahdollista muodostaa ”kupla”, jossa on olemassa jonkinlainen ihmiskunnalle toistaiseksi tuntematon M-teoriassa ehdotettu ulottuvuus...... Kuplan sisällä olevan aluksen eteen muodostunut kutistuva aika-avaruuden vyöhyke yhdessä aluksen taakse muodostuneen laajenevan alueen kanssa ”työntäisi kuplaa ja sen sisällä olevaa alusta eteenpäin eräänlaisessa aika-avaruustuubissa ilman, että alus teknisesti ylittäisi valonnopeutta”. Kts. myös Casimirin ilmiö.

        https://www.businessinsider.com/warp-drive-study-department-defense-real-fake-2018-5?r=US&IR=T

        "Tästä eteenpäin avaruus itsessään ja aika itsessään ovat tuomittuja haalistumaan pelkiksi varjoiksi ja vain näiden kahden eräänlainen liitto voi säilyttää riippumattoman todellisuuden." (Minkowski 1908)


      • Anonyymi
        Anonyymi kirjoitti:

        Lainaus: "Atomin värähtelytaajuus ei tavallaan muutu, vaan itse aika. Perustuu yleiseen suhteellisuusteoriaan, ja sen pohjalta tehtyihin havaintoihin."

        Lainaus: "Mutta rivien välistä voisi tulkita, että insinöörit katsovat kelloerojen syntyvän kellon taajuuden muuttumisesta, joka olisi helppo resetoida, kun taas fyysikot tulkitsevat että aika muuttuu."

        Lainaus: "Ei ole mitään järkeä muuttaa värähtelytaajuutta koska se ottaa saman määrän sekunteja myös kun verrataan mihin tahansa muuhun reaktioon joka ottaa tunnetun määrän aikaa."

        Lainaus: "90% valonnopeudessa kulkevalle aika kulkee silti samaa nopeutta kuin muillekin, eli yksi sekunti per sekunti. Tämän määritelmän mukaan atomin värähtely pysyy myös samana kuin mikä mitattiin maassa."

        --

        Lainaus: "GPS-kellojen tahdistuksesta vielä, kelloja ei tahdisteta Einsteinin tahdistuksen
        tapaan, vaan yhdellä "rysähdyksellä" resetoimalla kellojen taajuus...... Tällä on tärkeä ero Einsteinin tahdistukseen, siinä siirretään ajallisia suhteita kellojen välillä, nimenomaan kahta suhdetta, näitä Einstein piti tärkeinä."

        GPS-kellojen tapa tahdistaa olisi siis Star Trekin Enterprise-aluksella käyttökelvoton, koska alus haluaisi aina pysyä yhdessä ja samassa Maan tosiajassa. Siis jatkuvasti ja koko ajan. "Rysähdys" kerran tunnissa tai kerran vuorokaudessa oli valtavilla nopeuksilla tai painovoimakentissä liian harvoin.

        Kuten eräs anonyymi on todennut "siinä pitää selvästi tietää (aluksen) reitti kaikkien tähtien ja mustienaukkojen ('voimakkaat gravitaatiokentät') ohi tähän mennessä. Pelkkä maan gravitaatiokenttä ja riippuvuus etäisyydestä maahan ei riitä." Tuo toteamus on aivan totta, sillä yksikin poikkeaminen voimakkaassa gravitaatiokentässä riittäisi sekoittamaan avaruuskellon ajan totaalisesti Maan tosiajasta. Aluksen ja atomikellon tulisi siis jatkuvasti 24/7 analysoida kiihtyvyyttää, nopeuttaan sekä ympäristön painovoimaa.

        Jos lainattu anonyymin väite pitää paikkansa, Einsteinin tapa olisi oikea tapa, sillä siinä siirrettäisiin kellojen ajallisia suhteita.

        Lainaus: "Kello resetoinnissa ajalliset suhteet ovat erilaisia, jos ollenkaan aikaan
        liittyviä, vain kelloihin liittyviä."

        Enterprise-aluksen kohdalla kaikki ovat tietoisia siitä, että aika ei ole kiinnostava, koska se on avaruuden joka koordinaatistossa erilainen ja havainnoitsijasta riippuvainen. Vain aluksen kellonajan on haluttu vastaavan Maan tosiaikaa, ja avauksessa kiinnostavaa on siis vain kelloihin liittyvät suhteet.

        Edelleenkin jää eri kirjoittajien näkemyksistä epäselväksi, onko Cs-133-atomin värähtelytaajuus yksi ja sama riippumatta atomiin kohdistuvasta nopeudesta tai painovoimasta. Jos kellojen GPS-tahdistuksessa taajuus resetoidaan "insinöörien tapaan rysäyksinä", mikä estäisi tahdistamasta kelloja 24/7 Einsteinin tahdistuksen tapaan jatkuvasti? Ilmeisesti taajuus resetoidaan pelkästään käytännön syistä.

        --

        Jos eletään tähtiaikaa 8454.130, kapteeni Kirkillä on edessään lukuisia haasteita, jolloin Maan sekunnit tai tunnit eivät ole tärkeitä. Maan tosiaika on sivuseikka, kun käytössä on tähtiaika; hyvä että edes vuodet ovat selvillä. Poimuajo olisi mahdollista vain pimeän energian avulla. Riittävällä määrällä pimeää energiaa avaruusaluksen ympärille olisi mahdollista muodostaa ”kupla”, jossa on olemassa jonkinlainen ihmiskunnalle toistaiseksi tuntematon M-teoriassa ehdotettu ulottuvuus...... Kuplan sisällä olevan aluksen eteen muodostunut kutistuva aika-avaruuden vyöhyke yhdessä aluksen taakse muodostuneen laajenevan alueen kanssa ”työntäisi kuplaa ja sen sisällä olevaa alusta eteenpäin eräänlaisessa aika-avaruustuubissa ilman, että alus teknisesti ylittäisi valonnopeutta”. Kts. myös Casimirin ilmiö.

        https://www.businessinsider.com/warp-drive-study-department-defense-real-fake-2018-5?r=US&IR=T

        "Tästä eteenpäin avaruus itsessään ja aika itsessään ovat tuomittuja haalistumaan pelkiksi varjoiksi ja vain näiden kahden eräänlainen liitto voi säilyttää riippumattoman todellisuuden." (Minkowski 1908)

        """Jos lainattu anonyymin väite pitää paikkansa, Einsteinin tapa olisi oikea tapa, sillä siinä siirrettäisiin kellojen ajallisia suhteita."""

        Myös anonyymin kommentoima wikipedia-kaava on tehty Einsteinin tavalla. Eli siinä ollaan pysyvällä semi-nopella radalla yhden gravitaatiolähteen ympärillä, ja Einsteinin efektit lasketaan verrattuna toiseen henkilöön.


      • Anonyymi
        Anonyymi kirjoitti:

        """Jos lainattu anonyymin väite pitää paikkansa, Einsteinin tapa olisi oikea tapa, sillä siinä siirrettäisiin kellojen ajallisia suhteita."""

        Myös anonyymin kommentoima wikipedia-kaava on tehty Einsteinin tavalla. Eli siinä ollaan pysyvällä semi-nopella radalla yhden gravitaatiolähteen ympärillä, ja Einsteinin efektit lasketaan verrattuna toiseen henkilöön.

        Einsteinin kellon tahdistus ei siis toimi mainituissa avaruuden olosuhteissa.

        Nasan Voyager 1 -operaatio käynnistettiin vuonna 1977. Luotain siirtyi tähtienväliseen tilaan vuonna 2012 ja jatkoi edelleen matkaansa. Tällä hetkellä Voyager on lähes 25 miljardin kilometrin päässä Maasta.

        Maapallolta tulevalla signaalilla kestää lähes vuorokausi saavuttaa luotain samoin kuin toiseenkin suuntaan kulkevalla signaalilla. Kestäisi siis parisen vuorokautta, jotta kellot saataisiin samaan tahdistettuun tosiaikaan.

        Voyager 1 on hidas eikä siihen kohdistu tähtien välisessä avaruudessa mitään erityisiä gravitaatiovoimia. Sen nopeus on vain noin reilut 60 000 km/h. Avaruuden mittakaavassa nopeus ja etäisyys on kuin kärpäsen pissa Atlantissa.


      • Anonyymi
        Anonyymi kirjoitti:

        Lainaus: "Atomin värähtelytaajuus ei tavallaan muutu, vaan itse aika. Perustuu yleiseen suhteellisuusteoriaan, ja sen pohjalta tehtyihin havaintoihin."

        Lainaus: "Mutta rivien välistä voisi tulkita, että insinöörit katsovat kelloerojen syntyvän kellon taajuuden muuttumisesta, joka olisi helppo resetoida, kun taas fyysikot tulkitsevat että aika muuttuu."

        Lainaus: "Ei ole mitään järkeä muuttaa värähtelytaajuutta koska se ottaa saman määrän sekunteja myös kun verrataan mihin tahansa muuhun reaktioon joka ottaa tunnetun määrän aikaa."

        Lainaus: "90% valonnopeudessa kulkevalle aika kulkee silti samaa nopeutta kuin muillekin, eli yksi sekunti per sekunti. Tämän määritelmän mukaan atomin värähtely pysyy myös samana kuin mikä mitattiin maassa."

        --

        Lainaus: "GPS-kellojen tahdistuksesta vielä, kelloja ei tahdisteta Einsteinin tahdistuksen
        tapaan, vaan yhdellä "rysähdyksellä" resetoimalla kellojen taajuus...... Tällä on tärkeä ero Einsteinin tahdistukseen, siinä siirretään ajallisia suhteita kellojen välillä, nimenomaan kahta suhdetta, näitä Einstein piti tärkeinä."

        GPS-kellojen tapa tahdistaa olisi siis Star Trekin Enterprise-aluksella käyttökelvoton, koska alus haluaisi aina pysyä yhdessä ja samassa Maan tosiajassa. Siis jatkuvasti ja koko ajan. "Rysähdys" kerran tunnissa tai kerran vuorokaudessa oli valtavilla nopeuksilla tai painovoimakentissä liian harvoin.

        Kuten eräs anonyymi on todennut "siinä pitää selvästi tietää (aluksen) reitti kaikkien tähtien ja mustienaukkojen ('voimakkaat gravitaatiokentät') ohi tähän mennessä. Pelkkä maan gravitaatiokenttä ja riippuvuus etäisyydestä maahan ei riitä." Tuo toteamus on aivan totta, sillä yksikin poikkeaminen voimakkaassa gravitaatiokentässä riittäisi sekoittamaan avaruuskellon ajan totaalisesti Maan tosiajasta. Aluksen ja atomikellon tulisi siis jatkuvasti 24/7 analysoida kiihtyvyyttää, nopeuttaan sekä ympäristön painovoimaa.

        Jos lainattu anonyymin väite pitää paikkansa, Einsteinin tapa olisi oikea tapa, sillä siinä siirrettäisiin kellojen ajallisia suhteita.

        Lainaus: "Kello resetoinnissa ajalliset suhteet ovat erilaisia, jos ollenkaan aikaan
        liittyviä, vain kelloihin liittyviä."

        Enterprise-aluksen kohdalla kaikki ovat tietoisia siitä, että aika ei ole kiinnostava, koska se on avaruuden joka koordinaatistossa erilainen ja havainnoitsijasta riippuvainen. Vain aluksen kellonajan on haluttu vastaavan Maan tosiaikaa, ja avauksessa kiinnostavaa on siis vain kelloihin liittyvät suhteet.

        Edelleenkin jää eri kirjoittajien näkemyksistä epäselväksi, onko Cs-133-atomin värähtelytaajuus yksi ja sama riippumatta atomiin kohdistuvasta nopeudesta tai painovoimasta. Jos kellojen GPS-tahdistuksessa taajuus resetoidaan "insinöörien tapaan rysäyksinä", mikä estäisi tahdistamasta kelloja 24/7 Einsteinin tahdistuksen tapaan jatkuvasti? Ilmeisesti taajuus resetoidaan pelkästään käytännön syistä.

        --

        Jos eletään tähtiaikaa 8454.130, kapteeni Kirkillä on edessään lukuisia haasteita, jolloin Maan sekunnit tai tunnit eivät ole tärkeitä. Maan tosiaika on sivuseikka, kun käytössä on tähtiaika; hyvä että edes vuodet ovat selvillä. Poimuajo olisi mahdollista vain pimeän energian avulla. Riittävällä määrällä pimeää energiaa avaruusaluksen ympärille olisi mahdollista muodostaa ”kupla”, jossa on olemassa jonkinlainen ihmiskunnalle toistaiseksi tuntematon M-teoriassa ehdotettu ulottuvuus...... Kuplan sisällä olevan aluksen eteen muodostunut kutistuva aika-avaruuden vyöhyke yhdessä aluksen taakse muodostuneen laajenevan alueen kanssa ”työntäisi kuplaa ja sen sisällä olevaa alusta eteenpäin eräänlaisessa aika-avaruustuubissa ilman, että alus teknisesti ylittäisi valonnopeutta”. Kts. myös Casimirin ilmiö.

        https://www.businessinsider.com/warp-drive-study-department-defense-real-fake-2018-5?r=US&IR=T

        "Tästä eteenpäin avaruus itsessään ja aika itsessään ovat tuomittuja haalistumaan pelkiksi varjoiksi ja vain näiden kahden eräänlainen liitto voi säilyttää riippumattoman todellisuuden." (Minkowski 1908)

        Mutu on olettamuksen mukkaa kala


    • Anonyymi

      Sanoin jo aiemmin mutta ei näytä menevän perille:

      Jos raami R' liikkuu raamissa R nopeudella v, niin R:n havaitsija ja R':n havaitsija jakavat avaruusajan aika- ja avaruuskomponentteihin eri lailla.Nelivektorin S aika- ja avaruuskomponentit ovat erilaiset näissä raameissa. Seuraa aikadilataatio ja pituuskontraktio.Noiden komponenttien muunnokset kertoo Lorentz-transformaatio.

      Ihan turhaa jankuttaa noista cesium-kelloista!

      Kun kuljetaan vaikkapa edestakaisin johonkin tähteen niin on tutkittava kulkijan maailmanviiva. Tämän pituus Lorentz-metriikassa (Minkowski-metriikka, jos haluatte) kertoo matkan pituuden 4-avaruudessa. Se on kulkijan itseisaika.
      Maan pinnalla havaittava itseisaika on pitempi kuin matkaajan tuossa metriikassa.

      Dixi

      • Anonyymi

        > Ihan turhaa jankuttaa noista cesium-kelloista!

        Sinä et ole lainkaan siinä asemassa, että voisit kertoa, mistä täällä keskustellaan. Et ole ymmärtänyt avauksen luonnetta, kuvaamasi raamit ovat täysin huuhaata ja olet omissa harhoissasi.

        On jo selvinnyt, että paras tapa edistää asiaa olisi Einsteinin tahdistus.


      • Anonyymi
        Anonyymi kirjoitti:

        > Ihan turhaa jankuttaa noista cesium-kelloista!

        Sinä et ole lainkaan siinä asemassa, että voisit kertoa, mistä täällä keskustellaan. Et ole ymmärtänyt avauksen luonnetta, kuvaamasi raamit ovat täysin huuhaata ja olet omissa harhoissasi.

        On jo selvinnyt, että paras tapa edistää asiaa olisi Einsteinin tahdistus.

        "Myös anonyymin kommentoima wikipedia-kaava on tehty Einsteinin tavalla. Eli siinä ollaan pysyvällä semi-nopella radalla yhden gravitaatiolähteen ympärillä, ja Einsteinin efektit lasketaan verrattuna toiseen henkilöön."

        Einsteinin tahdistus ei tietenkään toimisi ko. olosuhteissa, sillä se edellyttäisi signaalien lähettämistä ja vastaanottamista. Oletettavasti avaruudessa oltaisiin usein sellaisilla etäisyyksillä, että tiedon lähettäminen ja vastaanottaminen kestäisi ns. ikuisuuden.


      • Anonyymi
        Anonyymi kirjoitti:

        "Myös anonyymin kommentoima wikipedia-kaava on tehty Einsteinin tavalla. Eli siinä ollaan pysyvällä semi-nopella radalla yhden gravitaatiolähteen ympärillä, ja Einsteinin efektit lasketaan verrattuna toiseen henkilöön."

        Einsteinin tahdistus ei tietenkään toimisi ko. olosuhteissa, sillä se edellyttäisi signaalien lähettämistä ja vastaanottamista. Oletettavasti avaruudessa oltaisiin usein sellaisilla etäisyyksillä, että tiedon lähettäminen ja vastaanottaminen kestäisi ns. ikuisuuden.

        Tahdistetaan nyt jollain ihmikonstilla matkaajakaksosen kello käymään samaan tahtiin kuin maamaakaksosen kello. Matkaaja palatessa hänen kellonsa ja maakaksosen kello näyttävät, että aikaa on kulunut 50 vuotta. Maakaksonen on 50 vuotta vanhempi kuin alkuhetkellä, matkaaja on vanhentunut vain esim. 5 vuotta. Matkaajan sydän on kuitenkin lyönyt vain tietyn määrän eli kymmenesosan maakaksosen sydämen lyönneistä.

        Otetaan nyt rautalankaesimerkki. Olkoon meillä 2-ulotteisessa avaruudessa ortonormaalinen x,t-koordinaatisto. Otetaan käyttöön vino x',t'-koordinaatisto jossa esim. x'-akseli on kulmassa pii/6 x-akselia vastaan positiivisessa kiertosuunnassa ja t'-akseli tästä samaan suuntaan kulmassa pii/6 eli tekee t-akselin kanssa kulman pii/6.
        Otetaan nyt 2-vektori S. Sillä on tietyt komponentit x,t-koordinaatistossa ja toiset komponentit x',t'-koordinaatistossa. Molemmat koordinaatistot käyttävät samoja mittayksiköitä.Vektorin S pituus on koordinaatistosta riippumaton.Komponenttien välillä toimii tavallinen euklidinen muunnos.

        .Samoin 4-vektorien pituus EST:ssa on absoluuttinen mutta eri koordinaatistoissa se lasketaan eri lailla:
        c^2 t^2 - dx^2 - dy^2 - dz^2 = c^2 dt'^2 - dx'^2 - dy'^2 - dz^'^2.
        Aika- ja abaruuskomponenteilla on eri arvot näissä koordinaatistoissa mutta vektori on absoluuttinen suure.
        Jos matkaajalla on mukanaan mittakeppi jonka pituudeksi (avaruuskomponentiksi) hän saa arvon 1 m, niin maakaksonen saa kepin pituudeksi arvon, jonka voi laskea Lorentz-muunnoksella. Tämä on maakaksosen mielestä kepin todellinen pituus!

        Kun asiota mietitään EST:ssa täytyy käyttää sen ajatusmaailmaa. Newtonilainen ajattelu ei kelpaa, mutta monet ajattelevat newtonilaisittain vaikka ovat puhuvinaan EST:sta


      • Anonyymi
        Anonyymi kirjoitti:

        Tahdistetaan nyt jollain ihmikonstilla matkaajakaksosen kello käymään samaan tahtiin kuin maamaakaksosen kello. Matkaaja palatessa hänen kellonsa ja maakaksosen kello näyttävät, että aikaa on kulunut 50 vuotta. Maakaksonen on 50 vuotta vanhempi kuin alkuhetkellä, matkaaja on vanhentunut vain esim. 5 vuotta. Matkaajan sydän on kuitenkin lyönyt vain tietyn määrän eli kymmenesosan maakaksosen sydämen lyönneistä.

        Otetaan nyt rautalankaesimerkki. Olkoon meillä 2-ulotteisessa avaruudessa ortonormaalinen x,t-koordinaatisto. Otetaan käyttöön vino x',t'-koordinaatisto jossa esim. x'-akseli on kulmassa pii/6 x-akselia vastaan positiivisessa kiertosuunnassa ja t'-akseli tästä samaan suuntaan kulmassa pii/6 eli tekee t-akselin kanssa kulman pii/6.
        Otetaan nyt 2-vektori S. Sillä on tietyt komponentit x,t-koordinaatistossa ja toiset komponentit x',t'-koordinaatistossa. Molemmat koordinaatistot käyttävät samoja mittayksiköitä.Vektorin S pituus on koordinaatistosta riippumaton.Komponenttien välillä toimii tavallinen euklidinen muunnos.

        .Samoin 4-vektorien pituus EST:ssa on absoluuttinen mutta eri koordinaatistoissa se lasketaan eri lailla:
        c^2 t^2 - dx^2 - dy^2 - dz^2 = c^2 dt'^2 - dx'^2 - dy'^2 - dz^'^2.
        Aika- ja abaruuskomponenteilla on eri arvot näissä koordinaatistoissa mutta vektori on absoluuttinen suure.
        Jos matkaajalla on mukanaan mittakeppi jonka pituudeksi (avaruuskomponentiksi) hän saa arvon 1 m, niin maakaksonen saa kepin pituudeksi arvon, jonka voi laskea Lorentz-muunnoksella. Tämä on maakaksosen mielestä kepin todellinen pituus!

        Kun asiota mietitään EST:ssa täytyy käyttää sen ajatusmaailmaa. Newtonilainen ajattelu ei kelpaa, mutta monet ajattelevat newtonilaisittain vaikka ovat puhuvinaan EST:sta

        Lisään nyt vielä, että se, mitä tuossa sanoin siitä kepin avaruuskomponentista pätee myös kun mitataan jotain ajanjaksoa eli 4-vektorin aikakomponentteja. Ne ovat molemmille havaitsijoille t o d e l l i s i a, vaikka niillä onkin eri arvot. Tässä EST:n maailmankuva eroaa tuosta Newtonin maailmasta.


      • Anonyymi
        Anonyymi kirjoitti:

        "Myös anonyymin kommentoima wikipedia-kaava on tehty Einsteinin tavalla. Eli siinä ollaan pysyvällä semi-nopella radalla yhden gravitaatiolähteen ympärillä, ja Einsteinin efektit lasketaan verrattuna toiseen henkilöön."

        Einsteinin tahdistus ei tietenkään toimisi ko. olosuhteissa, sillä se edellyttäisi signaalien lähettämistä ja vastaanottamista. Oletettavasti avaruudessa oltaisiin usein sellaisilla etäisyyksillä, että tiedon lähettäminen ja vastaanottaminen kestäisi ns. ikuisuuden.

        >>>> Einsteinin tahdistus ei tietenkään toimisi ko. olosuhteissa, sillä se edellyttäisi signaalien lähettämistä ja vastaanottamista. Oletettavasti avaruudessa oltaisiin usein sellaisilla etäisyyksillä, että tiedon lähettäminen ja vastaanottaminen kestäisi ns. ikuisuuden.

        On mainittu, että Maan tosiajan ja avaruusajan sykronomiseksi pitäisi tietää ainakin avaruusaluksen kulloinenkin nopeus, painovoima sekä tapauskohtaisesti myös kiihtyvyys. Erityinen suhteellisuusteoria käsittelee vain koordinaatistoja, jotka liikkuvat toistensa suhteen vakionopeuksilla, ja kiihtyvyydet kuuluvat yleisen suhteellisuusteorian puolelle.

        Jos siis kahden kellon etäisyys sekä muuttuu vakionopeudella että muuttuvalla nopeudella kiihtyvyys huomioiden, pitäisi ottaa myös huomioon se, onko käytössä erityinen vaiko yleinen suhteellisuuteoria. Suhteellisuusteorian mukaan sekä aika että etäisyydet ovat suhteellisia. Etäisyyksiä on totuttu tietenkin tarkastelemaan ainoastaan Maan lepokoordinaatistossa! Lorentz-muunnokset voivat kertoa, että sovitun referenssipisteen (pituuden) suhteen liikkuva havaitsija näkee pisteen eri tavalla kuin sen suhteen levossa oleva havaitsija. Ilmiö on nimeltään Lorentzin kontraktio. Liikkuvan tarkastelijan kannalta matka-aika etäiselle planeetalle ei ole sama kuin Maan lepokoordinaatistosta tarkasteltuna.

        Avaruuskellon suhteen olisi siis oleellista tietää, mitä inertiaalikoordinaatistoa käytetään ja montako kertaa tai missä avaruusaluksen inertiaalikoordinaatisto on matkan kestäessä vaihtanut. Jos Star Trekin Enterprise on luvannut käydä tarkistamassa läheisen eksoplaneetan asuttamiskelpoisuuden, ajallisesti tulisi sopia, missä inertiaalikoordinaatistossa matkan pituudesta sovitaan. Luontaista olisi sopia että kaikki tapahtuu Maan intertiaalikoordinaatistossa, koska halutaan myös puhua Maan tosiajasta.

        Maan tosiaika ja Maan inertiaalikoordinaatisto. Koska Star Trekin Enterprisen nopeus ei Maan intertiaalikoordinaatiston suhteen vakio, tulisi huomioida sekä erityinen että yleinen suhteellisuusteoria. Kiihtyvyys vaikuttaa asiassa kellon käytökseen. Koska Enterprisen nopeus on suuri ja se saattaa kohdata suuria painovoimakohteita, kiihtyvyyden lisäksi laskennassa tulee huomioida sekä nopeus- ja painovoimasensorien lukemat sekä tietenkin Lorentzin muunnokset sekä tietenkin se, vaihtaako Enterprise intertiaalikoordinaatistoaan.

        Mitä tulee atomien värähtelyyn, myös se lienee suhteellista. Enterprisen koordinaatistossa värähtelytaajuus on sama kuin se on sekunnille määriteltykin, mutta Maan inertiaalikoordinaatistossa värähtely samoin kuin sekunti ovat täysin erilaisia. Ajan ja värähtelyn suhteen kyse on vain siitä, kuka ja missä kysyy.

        Olisi erikoista, että joku kykenisi edellisen mitenkään laskemaan. Asia on aivan liian vaikea. Äkkiä ajateltuna on turha puhua sekunneista, hyvä kun alus pystyisi palaamaan maahan edes oikeana vuonna.


      • Anonyymi
        Anonyymi kirjoitti:

        >>>> Einsteinin tahdistus ei tietenkään toimisi ko. olosuhteissa, sillä se edellyttäisi signaalien lähettämistä ja vastaanottamista. Oletettavasti avaruudessa oltaisiin usein sellaisilla etäisyyksillä, että tiedon lähettäminen ja vastaanottaminen kestäisi ns. ikuisuuden.

        On mainittu, että Maan tosiajan ja avaruusajan sykronomiseksi pitäisi tietää ainakin avaruusaluksen kulloinenkin nopeus, painovoima sekä tapauskohtaisesti myös kiihtyvyys. Erityinen suhteellisuusteoria käsittelee vain koordinaatistoja, jotka liikkuvat toistensa suhteen vakionopeuksilla, ja kiihtyvyydet kuuluvat yleisen suhteellisuusteorian puolelle.

        Jos siis kahden kellon etäisyys sekä muuttuu vakionopeudella että muuttuvalla nopeudella kiihtyvyys huomioiden, pitäisi ottaa myös huomioon se, onko käytössä erityinen vaiko yleinen suhteellisuuteoria. Suhteellisuusteorian mukaan sekä aika että etäisyydet ovat suhteellisia. Etäisyyksiä on totuttu tietenkin tarkastelemaan ainoastaan Maan lepokoordinaatistossa! Lorentz-muunnokset voivat kertoa, että sovitun referenssipisteen (pituuden) suhteen liikkuva havaitsija näkee pisteen eri tavalla kuin sen suhteen levossa oleva havaitsija. Ilmiö on nimeltään Lorentzin kontraktio. Liikkuvan tarkastelijan kannalta matka-aika etäiselle planeetalle ei ole sama kuin Maan lepokoordinaatistosta tarkasteltuna.

        Avaruuskellon suhteen olisi siis oleellista tietää, mitä inertiaalikoordinaatistoa käytetään ja montako kertaa tai missä avaruusaluksen inertiaalikoordinaatisto on matkan kestäessä vaihtanut. Jos Star Trekin Enterprise on luvannut käydä tarkistamassa läheisen eksoplaneetan asuttamiskelpoisuuden, ajallisesti tulisi sopia, missä inertiaalikoordinaatistossa matkan pituudesta sovitaan. Luontaista olisi sopia että kaikki tapahtuu Maan intertiaalikoordinaatistossa, koska halutaan myös puhua Maan tosiajasta.

        Maan tosiaika ja Maan inertiaalikoordinaatisto. Koska Star Trekin Enterprisen nopeus ei Maan intertiaalikoordinaatiston suhteen vakio, tulisi huomioida sekä erityinen että yleinen suhteellisuusteoria. Kiihtyvyys vaikuttaa asiassa kellon käytökseen. Koska Enterprisen nopeus on suuri ja se saattaa kohdata suuria painovoimakohteita, kiihtyvyyden lisäksi laskennassa tulee huomioida sekä nopeus- ja painovoimasensorien lukemat sekä tietenkin Lorentzin muunnokset sekä tietenkin se, vaihtaako Enterprise intertiaalikoordinaatistoaan.

        Mitä tulee atomien värähtelyyn, myös se lienee suhteellista. Enterprisen koordinaatistossa värähtelytaajuus on sama kuin se on sekunnille määriteltykin, mutta Maan inertiaalikoordinaatistossa värähtely samoin kuin sekunti ovat täysin erilaisia. Ajan ja värähtelyn suhteen kyse on vain siitä, kuka ja missä kysyy.

        Olisi erikoista, että joku kykenisi edellisen mitenkään laskemaan. Asia on aivan liian vaikea. Äkkiä ajateltuna on turha puhua sekunneista, hyvä kun alus pystyisi palaamaan maahan edes oikeana vuonna.

        Puutun nyt vain yhteen asiaan tuossa. Jos todellisia gravitaatiokenttiä ei ole niin kyllä kiihtyvänkin (ja hidastuvan) aluksen maailmanviivan pituus eli matkaajan itseisaika voidaan laskea ihan EST:ssa. YST:a ei siinä tarvita.


      • Anonyymi
        Anonyymi kirjoitti:

        "Myös anonyymin kommentoima wikipedia-kaava on tehty Einsteinin tavalla. Eli siinä ollaan pysyvällä semi-nopella radalla yhden gravitaatiolähteen ympärillä, ja Einsteinin efektit lasketaan verrattuna toiseen henkilöön."

        Einsteinin tahdistus ei tietenkään toimisi ko. olosuhteissa, sillä se edellyttäisi signaalien lähettämistä ja vastaanottamista. Oletettavasti avaruudessa oltaisiin usein sellaisilla etäisyyksillä, että tiedon lähettäminen ja vastaanottaminen kestäisi ns. ikuisuuden.

        Kaavoissa ei ole varsinaisesti kyse kellojen synkronisoinnista, joka on peräisin SR:stä sillä nimellä kuin Einsteinin synkronisaatio. Siinä lähetetään valoa A:sta B:hen ja B vastaa, tai ainakin käynnistää silloin kellonsa, kun saa valon.

        https://einsteinpapers.press.princeton.edu/vol2-trans/155

        Einsteinin luvussa I. §1 ja I. §2 synkronisoidaan vain kelloja, jotka ovat litteässä avaruudessa liikkumatta, mutta toisistaan kaukana. Hän saa määriteltyä tietävänsä ajan jossain kaukana ja voivansa kirjoittaa sen perusteella seuraavat luvut. Synkronisaatioon voi kuulua se, että laitetaan molemmat kellot näyttämään hetkeä 0:00 samaa fyysistä tapahtumaa varten. Tässä se on se, kun B ottaa valon vastaan. Ja A:ssa kello tietää, milloin se oli, koska olettaa sen olleen tasan puolessa välissä sitä aikaa kun ensimmäisestä lähetyksestä meni. Tässä tapauksessa kellot käyvät aina jatkossakin samaa tahtia ja näyttävät aina saman luvun m : ss. Heti kun jokin liikkuu, kuten toinen kello tai joku muu niiden havaitsija, Einstein sanoo, että kaikki menee ihan eri tavalla.

        Jos A katsoo liikkuvaa B:tä, voidaan asettaa jonkin A:n luona olevan tapahtuman nimeltä 'A 0:00' olemaan sama kuin jokin nähdyllä radalla oleva tapahtuma. Se voi olla se, kun valo oli B:ssä. Mutta tässä pitää tietää muualta, mikä B:n nopeus ym. on, koska valon toinen matka kestää kauemmin. Tätä voi edelleen sanoa synkronisaatioksi, jos osataan saada sama 0:00 -hetki, mutta aika käy eri nopeudella. Muut tapahtumat näyttävät eri m : ss -aikoina tapahtuneilta, kun kellojen luona katsoo kaikkea.

        Yhteinen 0:00 -tapahtuma on siksi, jotta voisi tehdä esim. kokeen, missä molemmat tekevät sen samaan aikaan. Se ei SR:n ulkopuolella aina toimi siten, että Einsteinin menetelmä, missä vaihdetaan valosignaaleja voisi tuottaa edes sen ensimmäisen 0:00 -hetken oikealla tavalla. Menetelmä, joka olisi tehty yhdelle harvinaiselle toimivalle GR-tapaukselle ei olisi enää nimeltään Einsteinin synkronisaatio, koska siihen liittyy yleisaihe samanaikaisen pinnan määritelmästä:

        https://physics.stackexchange.com/questions/187719/is-simultaneity-well-defined-in-general-relativity

        Vain Kokeessa joka vertaa kahta kelloa mutta ei niiden (loppuarvo - alkuarvo) -lukua, pitää olettaa jokin sama lähtöaika. Niissä ei ole siltikään kyseessä Einsteinin menetelmä, jos kokeen voi aloittaa helposti yhdestä pisteestä. Synkronisaation onnistuminen tai sen tietäminen ei anna tietoa siitä, mikä on ollut tai tulee olemaan kellojen käyntinopeus. Jatkuva uusi synkronisaatio ei kuulu alkuperäiseen määritelmään, mutta jos tekee A:n ja B:n välillä lisäksi pysyvän säännön siitä, millä toistolla lähetetään lisää valoa (täydellistä hiukkasta), silloin näiden vaihtuvien lukujen laittaminen muistiin, voi vastata sitä tietoa, miten paljon on ollut aikadilataatiota, mutta tästä ei voida esim. johtaa, mikä on sen syy.

        Mainituissa kaavoissa ei synkronoida mitään, vaan niissä tiedetään vain, mikä on toisen havaitsijan ajan tai kellon käyntinopeus. Kellonsa käynnin nopeuden voi lähettää tekstiviestinä koko avaruudelle, ja kaikki avaruudessa saavat siitä selvää silloin, jos on lähettänyt jotain kiintopisteitä, mistä voi määritellä nopeutensa samalla tavalla kuin muut. On myös tunnettava kaikki pisteet jotka tuottavat gravitaatiota itsensä luona. Silloin tietää tämän gravitaation aiheuttaman aikadilataation verrattuna johonkin täydelliseen tyhjiöön.

        GPS-systeemi on nykyään sellainen, missä tarvitaan synkronisoitu kello, joka on aina ajaltaan m : ss sama kuin gps-kännykän kello. Tämä johtuu siitä, että sen peruskäyttö toimii yhdensuuntaisella lähetyksellä ja vastaanotolla, jolloin ainoa tapa saada etäisyystieto satelliittiin on, että satelliitti lähettää sellaisen kellonajan, josta laskemalla saa signaalin matkaan käytetyn ajan. Jos GPS toimisi lähettämällä signaaleja edes takaisin, se ei tarvitsisi synkronoitua kelloa, vaan riittäisi tietää kellojen käyntinopeudet. Tämä edes takaisin signalointi miljoonista kännyköistä samaan satelliittiin vaatii aluksi jonkin lähettimen kiinni kännykkään, ja kaikki lähetys tukkii kaistan.

        Kellon synkronointi voi olla niin helppoa, kuin että se synkronoidaan käden puristuksella maassa. Tällaisen jälkeen on teoriassa mahdollista lähettää esim. GPS-satelliitti, paikkaan ja nopeuteen, joka tiedetään etukäteen. Koska sen tietää etukäteen, voi fyysisesti manipuloida satelliitin kellon käymään hitaammin kuin maassa olevan. Ja näin ne ovat manipuloitu. Sen seurauksena,kun ns. oikea fyysinen aika käy nopeasti sateliitissa, siitä tulee lopulta samanlainen kello maasta katsottuna, kuin mitä kellot maassa ovat. Edellisessä lähetettävän viestityn ajan on oltava nimittäin tätä aikaa. GPS-satelliitteja synkronoidaan tosin lisää, koska oikeaan järjestelmään kertyy virheitä laukaisussa ja aina kellon yrittäessä käydä. Normaali käynti ei riitä vaadittuun tarkkuuteen ilman siirtämistä maapallolla oleviin raskaampiin kellojoukkoihin, jotka antavat saman 0:00 -hetken sekä kännyköille ja satelliiteille.


      • Anonyymi
        Anonyymi kirjoitti:

        Tahdistetaan nyt jollain ihmikonstilla matkaajakaksosen kello käymään samaan tahtiin kuin maamaakaksosen kello. Matkaaja palatessa hänen kellonsa ja maakaksosen kello näyttävät, että aikaa on kulunut 50 vuotta. Maakaksonen on 50 vuotta vanhempi kuin alkuhetkellä, matkaaja on vanhentunut vain esim. 5 vuotta. Matkaajan sydän on kuitenkin lyönyt vain tietyn määrän eli kymmenesosan maakaksosen sydämen lyönneistä.

        Otetaan nyt rautalankaesimerkki. Olkoon meillä 2-ulotteisessa avaruudessa ortonormaalinen x,t-koordinaatisto. Otetaan käyttöön vino x',t'-koordinaatisto jossa esim. x'-akseli on kulmassa pii/6 x-akselia vastaan positiivisessa kiertosuunnassa ja t'-akseli tästä samaan suuntaan kulmassa pii/6 eli tekee t-akselin kanssa kulman pii/6.
        Otetaan nyt 2-vektori S. Sillä on tietyt komponentit x,t-koordinaatistossa ja toiset komponentit x',t'-koordinaatistossa. Molemmat koordinaatistot käyttävät samoja mittayksiköitä.Vektorin S pituus on koordinaatistosta riippumaton.Komponenttien välillä toimii tavallinen euklidinen muunnos.

        .Samoin 4-vektorien pituus EST:ssa on absoluuttinen mutta eri koordinaatistoissa se lasketaan eri lailla:
        c^2 t^2 - dx^2 - dy^2 - dz^2 = c^2 dt'^2 - dx'^2 - dy'^2 - dz^'^2.
        Aika- ja abaruuskomponenteilla on eri arvot näissä koordinaatistoissa mutta vektori on absoluuttinen suure.
        Jos matkaajalla on mukanaan mittakeppi jonka pituudeksi (avaruuskomponentiksi) hän saa arvon 1 m, niin maakaksonen saa kepin pituudeksi arvon, jonka voi laskea Lorentz-muunnoksella. Tämä on maakaksosen mielestä kepin todellinen pituus!

        Kun asiota mietitään EST:ssa täytyy käyttää sen ajatusmaailmaa. Newtonilainen ajattelu ei kelpaa, mutta monet ajattelevat newtonilaisittain vaikka ovat puhuvinaan EST:sta

        Tuli tuohon kirjoitusvirhe. P.o.: c^2 dt^2 -dx^2....


      • Anonyymi
        Anonyymi kirjoitti:

        "Myös anonyymin kommentoima wikipedia-kaava on tehty Einsteinin tavalla. Eli siinä ollaan pysyvällä semi-nopella radalla yhden gravitaatiolähteen ympärillä, ja Einsteinin efektit lasketaan verrattuna toiseen henkilöön."

        Einsteinin tahdistus ei tietenkään toimisi ko. olosuhteissa, sillä se edellyttäisi signaalien lähettämistä ja vastaanottamista. Oletettavasti avaruudessa oltaisiin usein sellaisilla etäisyyksillä, että tiedon lähettäminen ja vastaanottaminen kestäisi ns. ikuisuuden.

        Helppoa kuin heinänteko tuo kellojen vertailu.
        Lähetetään toinen kello edestakaiselle matkalle jonnekin Alderaaniin tai Timbuktuun ja verrataan kelloja kun ne ovat taas vierekkäin pöydällä.


      • Anonyymi
        Anonyymi kirjoitti:

        Helppoa kuin heinänteko tuo kellojen vertailu.
        Lähetetään toinen kello edestakaiselle matkalle jonnekin Alderaaniin tai Timbuktuun ja verrataan kelloja kun ne ovat taas vierekkäin pöydällä.

        Niin kauan kun tätä ei pystytä käytännössä tekemään, nuo höpön höpön kirjoittelijat saavat vapaasti mellastaa.
        Ei ne kellojen ajat poikkea toisitaan, eikä kellon kuljettelijakaan ole vanhentunut sen vähempää tai enempää kuin maassa odottelijakaan.


      • Anonyymi
        Anonyymi kirjoitti:

        Niin kauan kun tätä ei pystytä käytännössä tekemään, nuo höpön höpön kirjoittelijat saavat vapaasti mellastaa.
        Ei ne kellojen ajat poikkea toisitaan, eikä kellon kuljettelijakaan ole vanhentunut sen vähempää tai enempää kuin maassa odottelijakaan.

        Kyllä Timbuktuun pääsee lentokoneella. Tai kamelilla. Ei tuo ole mitään uutta ja ennenkuulumatonta.


      • Anonyymi
        Anonyymi kirjoitti:

        Niin kauan kun tätä ei pystytä käytännössä tekemään, nuo höpön höpön kirjoittelijat saavat vapaasti mellastaa.
        Ei ne kellojen ajat poikkea toisitaan, eikä kellon kuljettelijakaan ole vanhentunut sen vähempää tai enempää kuin maassa odottelijakaan.

        > On muodostunut kaksi koulukuntaa, sateliitteja ja luotaimia tekevät insinöörit
        ja "einsteiniläiset" fyysikot, jotka eivät edes suostu keskustelemaan keskenään.

        Kumpaanko koulukuntaan sinä kuulut?


      • Anonyymi
        Anonyymi kirjoitti:

        > On muodostunut kaksi koulukuntaa, sateliitteja ja luotaimia tekevät insinöörit
        ja "einsteiniläiset" fyysikot, jotka eivät edes suostu keskustelemaan keskenään.

        Kumpaanko koulukuntaan sinä kuulut?

        Mikä on "Einsteiniläinen" fyysikko? Liittyykö se jotenkin tämän palstan paskanjauhajiin?


      • Anonyymi
        Anonyymi kirjoitti:

        Mikä on "Einsteiniläinen" fyysikko? Liittyykö se jotenkin tämän palstan paskanjauhajiin?

        Anon.
        "Mikä on "Einsteiniläinen" fyysikko?"

        Tietäisit jos seuraisit mitä maailmassa tapahtuu, etkä vain pyörisi
        kakkaleikeissäsi.


      • Anonyymi
        Anonyymi kirjoitti:

        > On muodostunut kaksi koulukuntaa, sateliitteja ja luotaimia tekevät insinöörit
        ja "einsteiniläiset" fyysikot, jotka eivät edes suostu keskustelemaan keskenään.

        Kumpaanko koulukuntaan sinä kuulut?

        Anon.
        "> On muodostunut kaksi koulukuntaa,

        Kumpaanko koulukuntaan sinä kuulut?"

        En ole sidoksissa kumpaankaan koulukuntaan,
        kannatan sitä jonka logiikka on kunnossa.


      • Anonyymi
        Anonyymi kirjoitti:

        Niin kauan kun tätä ei pystytä käytännössä tekemään, nuo höpön höpön kirjoittelijat saavat vapaasti mellastaa.
        Ei ne kellojen ajat poikkea toisitaan, eikä kellon kuljettelijakaan ole vanhentunut sen vähempää tai enempää kuin maassa odottelijakaan.

        Kansan syvät rivit eli tyhjät tynnyrit ovat puhuneet. Kolisevat tyhjyyttään.


      • Anonyymi
        Anonyymi kirjoitti:

        Niin kauan kun tätä ei pystytä käytännössä tekemään, nuo höpön höpön kirjoittelijat saavat vapaasti mellastaa.
        Ei ne kellojen ajat poikkea toisitaan, eikä kellon kuljettelijakaan ole vanhentunut sen vähempää tai enempää kuin maassa odottelijakaan.

        https://en.wikipedia.org/wiki/Hafele-Keating_experiment

        Tuo koe siis tehtiin jo vuonna 1970 ja nähtiin siinä että kellot kävivät eri aikaa.


      • Anonyymi

      • Anonyymi

      • Anonyymi
        Anonyymi kirjoitti:

        Jo maailman sivu on ollut edistäviä ja jätättäviä kelloja, entä sitten?

        Nyt löysi "einsteiniläinen" fyysikko todellisen syyn.


    • Anonyymi
    • Anonyymi

      Kapteeni Kirk olisi ihmeissään tästä keskustelusta.

      Vaikka inertiaalikoordinaatisto, koordinaatistomuutokset, Lorentzin kontraktio, Lorentzin muunnokset, Maan lepokoordinaatisto, gravitaatiomuutokset ja aikadilataatio otettaisiin huomioon, kapteeni Kirk tuskin pystyisi lähettämään äidilleen viestiä siten, että se olisi äidin syntymäpäivänä oikea-aikaisesti perillä.

      Vielä jos aluksen nopeus olisi kuplassa siirtyvän poimuajon mukainen, aijan mittaamisesta ei nykykeinoin tulisi kerrassaan mitään.

      • Anonyymi

        Anon.
        "Kapteeni Kirk olisi ihmeissään tästä keskustelusta.. ...."

        Kirjoitteluissa on viitattu atomikellon sekunteihin jonakin ehkä tärkeänä asiana,
        sekunti on kuitenkin vain aikajärjestelmän käsite, samalla tavoin kuin metri
        mittajärjestelmän perusyksikkönä.
        Minkä takia sekunnilla on tuollainen atominvärähtelyyn liittyvä määritelmä,
        siksi että SI-sekunnista piti tehdä mahdollisimman saman pituinen sekunti kuin
        ennen SI-jäjestelmää, preSI-sekunti ja SI-sekunti ovat suunnilleen samanpituisia.

        Atomikellot ovat kuitenkin tuoneet uusia tekijöitä ajan ymmärtämiseen, mitä ei
        aikaisemmin ymmärretty, tai tarkemmin atomikellot ja digitaalinen tiedonsiirto.

        Atomikellon suhde aikaan on kuitenkin samanlainen kuin muillakin kelloilla, aika
        ei käytä atomikelloa, kuten ei muitakaan kelloja.

        Kellot ovat mekaanisia laitteita ja toisaalta aika on hyvin abstraktinen käsite
        ajattelussamme, sellaisenaan aineeton tekijä, tästä ei kuitenkaan kannata
        päätellä että aika olisi vain käsite.

        Nyt meillä ihmisillä on hankalahko tilanne, suhteellisuusteoriat ovat sata vuotta
        vanhoja, silloiset Einsteinin kehitelmät ovat hankalia, eikä niiden luonteesta aina
        voi olla varma.

        On muodostunut kaksi koulukuntaa, sateliitteja ja luotaimia tekevät insinöörit
        ja "einsteiniläiset" fyysikot, jotka eivät edes suostu keskustelemaan keskenään!

        Einsteinin eräät ajatukset on hankalia, saadakseen suhtiksen toimimaan, hän
        on postuloinut tiettyjä asioita, fyysikot eivät tunnu välittävän postulaateista
        mitään, he vain laskevat ne normaaleina mukaan, mutta postulaattien rooli
        loogisena paikkalappuna onkin toinen, ne hämärtää sen mitä voimme päätellä
        kyseisestä kehitelmästä.

        Postulaatteja käytetään aika yleisesti ja aina niillä on päättelyä heikentävä rooli.

        Atomikellot ovat tuoneet esille sen, ettei koordinaatistomuunnokset vaikuta
        atomikelloihin, eikä ne tunnista kiihtyvyyttä, ja siten ne erottavat kiihtyvyyden ja
        painovoiman toisistaan.

        Atomikellojen mukaan ne pitävät vertailukoordinaatistona paikallisia massakeskittymiä,
        maapallolla on oma koordinaatisto, aurikokunnalla omansa ja planeetoilla omansa.
        Maapallon koordinaatisto löydettiin vasta 1970-luvulla.

        Digitaalinen tiedonsiirto ei välitä mitään esim luotaimen nopeudesta, eikä sen
        koordinaatistosta tai sen vaihdosta, sen takia luotaimen ja maan kellot ovat
        helposti tahdistettavia.

        Fyysikkojen aikadilaatio onkin insinöörien kellodilaatio.

        Suomenkielisessä wikissä atomikelloista sanotaan hämärästi:

        "Kelloihin – kuten itse ajan kulumiseen – vaikuttavat...."

        Niinpä kaksosparadoksiin saadaan oikea vastaus, kun kummallakin kaksosella on
        omat atomikellot ja matkan loputtua verrataan niitä.

        StarTrek ongelma ratkeaa aluksella olevalla aliavaruuskanavan kautta, joka välittää
        reaaliaikaisen keskustelun ja ajanpäivityksen.


    • Anonyymi

      Inertiaalikoordinaatisto on koordinaatisto, jossa Newtonin mekaniikan lait pätevät. Havaintoympäristöt, jotka liikkuvat toistensa suhteen vakionopeudella, ovat inertiaalikoordinaatistoja. Suhteellisuusperiaatteen mukaisesti luonnonlait ovat samanlaisina voimassa kaikissa toistensa suhteen inertiaalisissa koordinaatistoissa.

      Yksinkertaisesti tulkiten, jos avauksessa Maahan jäävä atomikello ja ns. avaruuskello eivät ole toistensa suhteen kiihtyvässä liikkeessä, ne ovat samassa inertiaalikoordinaatistossa, ja luonnonlait ovat samanlaisina. Kappale kiihtyy, kun sen nopeus muuttuu.

      Jos kaksi atomikelloa ovat samassa inertiaalikoordinaatistossa, eikä niiden nopeus muutu, ne ovat aina samassa ajassa. Oletuksena, ellei niiden keskinäinen nopeussuhde muutu, kyseessä on yksi ja sama vakionopeus ja siten sama inertiaalikoordinaatisto.

      Onko mahdollista, että Maassa (G = 9,81) ja mustan aukon läheisyydessä (painovoima on suuri mutta rajallinen) olevat atomikellot ovat samassa inertiaalikoordinaatistossa? Jos näin olisi, Maan atomikello sekä mustan aukon läheisyydessä oleva atomikello näyttäisivät samaa aikaa. Oletuksena mustassa aukossa aika pysähtyy ja sen läheisyydessä aika hidastuu, jolloin Maan tosiaikaa tuossa ympäristössä ei voida tuntea.

      Aiheuttaako painovoima aina kiihtyvän liikkeen, vaikka kappale olisikin näennäisesti paikallaan? Määritelmällisesti tasainen liike ei voi aiheuttaa kiihtyvyyttä.

      • Anonyymi

        Kunn puhutaann aikadilataatiosta niin ei olla Newtonin mekaniikassa. Sen lait e i v ä t tällöin päde.


      • Anonyymi
        Anonyymi kirjoitti:

        Kunn puhutaann aikadilataatiosta niin ei olla Newtonin mekaniikassa. Sen lait e i v ä t tällöin päde.

        Suhteellisuusperiaatteen mukaisesti luonnonlait ovat samanlaisina voimassa kaikissa toistensa suhteen inertiaalisissa koordinaatistoissa.


      • Anonyymi
        Anonyymi kirjoitti:

        Kunn puhutaann aikadilataatiosta niin ei olla Newtonin mekaniikassa. Sen lait e i v ä t tällöin päde.

        Onko mahdollista, että Maassa (G = 9,81) ja mustan aukon läheisyydessä (painovoima on suuri mutta rajallinen) olevat atomikellot ovat samassa inertiaalikoordinaatistossa?


      • Anonyymi
        Anonyymi kirjoitti:

        Kunn puhutaann aikadilataatiosta niin ei olla Newtonin mekaniikassa. Sen lait e i v ä t tällöin päde.

        Päteekö liikkeen suhteellisuusperiaate suhteellisuusteoriassa?

        The Dynamic Universe (DU) eli dynaamisen universuminen mallissa (DU) kelloilla (atomeilla) on eri värähtelytaajuudet eri energiatiloissa; sen takia kellot näyttävät eri aikaa. Tiedeyhteisö on hylkinyt suomalaisen huippukeksijän, Tuomas Suntolan, teoriaa.

        https://physicsfoundations.org/fi/1_5_dynamic-universe.html


      • Anonyymi
        Anonyymi kirjoitti:

        Kunn puhutaann aikadilataatiosta niin ei olla Newtonin mekaniikassa. Sen lait e i v ä t tällöin päde.

        Kysymys siitä, vaihtuuko inertiaalikoordinaatisto kellojen välillä vai pysyykö se koko ajan samana? Onko koordinaatisto koskaan ollutkaan sama, vaikka koordinaatistojen (kappaleiden. massojen, aikojen jne.) välillä ei olekaan havaittavaa kiihtyvyyttä.

        Suhteellisuusperiaatteen mukaisesti luonnonlait ovat samanlaisina voimassa kaikissa toistensa suhteen inertiaalisissa koordinaatistoissa. Kysymys onkin siitä, miten aikadilaatio voi ilmionä olla olemassa kahden inertiaalisen koordinaatiston välillä: ellei asiaa päätetä kiihtyvyyden perusteella, niin millä perusteella?


      • Anonyymi

        Aikaa ei voi enää synkronoida mustan aukon tapahtumahorisontin takaa.


      • Anonyymi
        Anonyymi kirjoitti:

        Aikaa ei voi enää synkronoida mustan aukon tapahtumahorisontin takaa.

        Luetko koskaan sitä, mitä muuta kirjoittavat? Sanottiin, että "mustan aukon läheisyydessä (painovoima on suuri mutta rajallinen)".

        Missä sanottiin, että tapahtumat ovat tapahtumahorisontin takana? Keksit asian aivan itse.


      • Anonyymi

        "Inertiaalikoordinaatisto on koordinaatisto, jossa Newtonin mekaniikan lait pätevät."

        Väärin. Muunmuassa jotkut koordinaatistot ovat äärettömiä, joten jos sanoo X:n pätevän koordinaatistossa, niin pätemistä on tosi paljon. Kaikki mitä näet olisi Newtonin mukaista havaintoa.

        "Havaintoympäristöt, jotka liikkuvat toistensa suhteen vakionopeudella, ovat inertiaalikoordinaatistoja."

        Väärin. Koordinaatiston voi määritellä yhden origon avulla. Jos tuntuu epävarmalta, että onko mahdollista kiihdyttää samaan suuntaan samalla voimalla ja pysyä samassa vauhdissa, niin silloin pitäisi olla samaan aikaan epäilys siitä, onko origon ympärillä ympäristö joka tottelee aina origon liikettä. Inertiaalikoordinaatiston voi määritellä yhden koordinaatiston avulla eikä tarvitse kahta. Tässä voi ajatella työkalua nimeltä akselometri.

        "Suhteellisuusperiaatteen mukaisesti luonnonlait ovat samanlaisina voimassa kaikissa toistensa suhteen inertiaalisissa koordinaatistoissa."

        Väärin. Koordinaatistot eivät ole toistensa suhteen inertiaalisia. Inertiaalinen on aina inertiaalinen.

        Yksi suhteellisuusperiaatteista käsittelee vain inertiaalisia koordinaatistoja.

        "Yksinkertaisesti tulkiten, jos avauksessa Maahan jäävä atomikello ja ns. avaruuskello eivät ole toistensa suhteen kiihtyvässä liikkeessä, ne ovat samassa inertiaalikoordinaatistossa"

        Väärin. Toinen on yhdessä inertiaalikoordinaatistossa, ja toinen kello on toisessa. Inertiaalikoordinaatiston yksi määrittelevä muuttuja on kellon nopeusvektori.

        " ja luonnonlait ovat samanlaisina. Kappale kiihtyy, kun sen nopeus muuttuu."

        Tunnetko kyseessä olevan lain? Koska se ei ole Newtonin mekaniikka.

        "Jos kaksi atomikelloa ovat samassa inertiaalikoordinaatistossa, eikä niiden nopeus muutu, ne ovat aina samassa ajassa. "

        Määrittele ajassa oleminen.

        Jos on olemassa inertiaalikoordinaatisto, silloin koko universumi on yleensä jossakin tässä koordinaatistossa. Jos jonkun mielestä asian nopeus ei muutu, tämä ei todista välttämättä, että kyseinen objekti ei kiihdytä. Esineen pitäisi läpäistä ei-kiihdyttämisen testi. Sen jälkeen esine olisi inertiaalinen. Inertiaalisen esineen keskelle voisi laittaa yhden nopeusvektorinmukaisen inertiaalikoordinaatiston, missä kyseinen esine on aina yhdessä pisteessä, kuten origo. Inertiaalisiksi päässeillä esineillä on yleensä oikea havainto siitä, että toiset kiihdyttämättömät esineet ovat kiihdyttämättömiä myös, mutta kun testi on kerran mainittu siihen voitaisiin vedota uudelleen.

        Kiihdyttämättömät kappaleet litteässä avaruudessa käyvät kumpikin symmetrisesti toistensa mielestä liian nopeasti ajassa. Siten, että erinopeuksiset kappaleet eivät tulkitse näkemiään tapahtumia samoilla kelloilla samoiksi kellon ajoiksi. Mikä tarkoittaa mm. että jos ne näkevät toistensa kellot viisareina, ne ovat eri mieltä, miten nopeaa kierros niissä pitää olla. Esim. avaruusalus menee Maasta kohti Siriuksen maaliviivaa ja näkee matkan olevan viisi kellokierrosta. Jos maasta näkee aluksen, maaliviivan ja aluskellon, se näyttää siltä, että viisi kierrosta tulee täyteen, kun maaliviiva ylitetään. Kun Maa katsoo samalla omaa kelloaan, tämä kello on kuitenkin käynyt kuusi kierrosta. Aluksen mielestä, jos Maassa olisi mahdolista nähdä jokin paikka, mihin maa menee (samassa suunnassa kuin mihin nopeus on, eli periaatteessa aluksen pitää vetää tätä paikkaa perässään ja tuoda se Maalle), niin Maa kävisi vain viisi kierrosta aluksen kuudessa.

        "Onko mahdollista, että Maassa (G = 9,81) ja mustan aukon läheisyydessä (painovoima on suuri mutta rajallinen) olevat atomikellot ovat samassa inertiaalikoordinaatistossa? "

        Yllä olevista asiosta tulee vain mieleen, että kysyt tätä asiaa väärin eikä vastauksella ole vielä merkitystä.

        "Jos näin olisi, Maan atomikello sekä mustan aukon läheisyydessä oleva atomikello näyttäisivät samaa aikaa"

        Määrittele saman ajan näyttäminen. Mistään ns. faktasta, mikä pätee inertiaalikoordinaatistoissa ei seuraa mitään, mikä antaa suoraan jonkun vastauksen kaarevassa avaruudessa. Toisia kelloja yritetään käsittää litteässä avaruudessa Lorentzin muunnoksella, joka siirtyy yhdestä inertiaalikoordinaatistosta toiseen. Ei ole olemassa yhtään kaarevuutta omaavaa avaruutta, missä voisi siirtyä toisen kappaleen koordinaatistoon vain tekemällä tämän Lorentzin muunnoksen (homogeenisessa kentässä saa siirtyä sillä siihen suuntaan, mikä on poikittain vedon suuntaan, ja epähomogeenisyydessä ongelma ei ole joskus ajan nopeudessa vaan siinä, että tulee väärän verran huomioitua etäisyyttä ajan ja paikan suunnassa, jos yrittää litistää kaarevuuden pois). Tätä ei muuta se, että toinen kappale näyttää pysyvän paikoillaan. Kaarevassa avaruudessa eri kaarevuuspisteissä olevat kappaleet näkevät yleensä tällaisen paikallaan olemisen vain koska jompikumpi kappale käyttää jotain muuta kuin gravitaatiovoimaa kiihdyttämään itseään.

        1


      • Anonyymi
        Anonyymi kirjoitti:

        "Inertiaalikoordinaatisto on koordinaatisto, jossa Newtonin mekaniikan lait pätevät."

        Väärin. Muunmuassa jotkut koordinaatistot ovat äärettömiä, joten jos sanoo X:n pätevän koordinaatistossa, niin pätemistä on tosi paljon. Kaikki mitä näet olisi Newtonin mukaista havaintoa.

        "Havaintoympäristöt, jotka liikkuvat toistensa suhteen vakionopeudella, ovat inertiaalikoordinaatistoja."

        Väärin. Koordinaatiston voi määritellä yhden origon avulla. Jos tuntuu epävarmalta, että onko mahdollista kiihdyttää samaan suuntaan samalla voimalla ja pysyä samassa vauhdissa, niin silloin pitäisi olla samaan aikaan epäilys siitä, onko origon ympärillä ympäristö joka tottelee aina origon liikettä. Inertiaalikoordinaatiston voi määritellä yhden koordinaatiston avulla eikä tarvitse kahta. Tässä voi ajatella työkalua nimeltä akselometri.

        "Suhteellisuusperiaatteen mukaisesti luonnonlait ovat samanlaisina voimassa kaikissa toistensa suhteen inertiaalisissa koordinaatistoissa."

        Väärin. Koordinaatistot eivät ole toistensa suhteen inertiaalisia. Inertiaalinen on aina inertiaalinen.

        Yksi suhteellisuusperiaatteista käsittelee vain inertiaalisia koordinaatistoja.

        "Yksinkertaisesti tulkiten, jos avauksessa Maahan jäävä atomikello ja ns. avaruuskello eivät ole toistensa suhteen kiihtyvässä liikkeessä, ne ovat samassa inertiaalikoordinaatistossa"

        Väärin. Toinen on yhdessä inertiaalikoordinaatistossa, ja toinen kello on toisessa. Inertiaalikoordinaatiston yksi määrittelevä muuttuja on kellon nopeusvektori.

        " ja luonnonlait ovat samanlaisina. Kappale kiihtyy, kun sen nopeus muuttuu."

        Tunnetko kyseessä olevan lain? Koska se ei ole Newtonin mekaniikka.

        "Jos kaksi atomikelloa ovat samassa inertiaalikoordinaatistossa, eikä niiden nopeus muutu, ne ovat aina samassa ajassa. "

        Määrittele ajassa oleminen.

        Jos on olemassa inertiaalikoordinaatisto, silloin koko universumi on yleensä jossakin tässä koordinaatistossa. Jos jonkun mielestä asian nopeus ei muutu, tämä ei todista välttämättä, että kyseinen objekti ei kiihdytä. Esineen pitäisi läpäistä ei-kiihdyttämisen testi. Sen jälkeen esine olisi inertiaalinen. Inertiaalisen esineen keskelle voisi laittaa yhden nopeusvektorinmukaisen inertiaalikoordinaatiston, missä kyseinen esine on aina yhdessä pisteessä, kuten origo. Inertiaalisiksi päässeillä esineillä on yleensä oikea havainto siitä, että toiset kiihdyttämättömät esineet ovat kiihdyttämättömiä myös, mutta kun testi on kerran mainittu siihen voitaisiin vedota uudelleen.

        Kiihdyttämättömät kappaleet litteässä avaruudessa käyvät kumpikin symmetrisesti toistensa mielestä liian nopeasti ajassa. Siten, että erinopeuksiset kappaleet eivät tulkitse näkemiään tapahtumia samoilla kelloilla samoiksi kellon ajoiksi. Mikä tarkoittaa mm. että jos ne näkevät toistensa kellot viisareina, ne ovat eri mieltä, miten nopeaa kierros niissä pitää olla. Esim. avaruusalus menee Maasta kohti Siriuksen maaliviivaa ja näkee matkan olevan viisi kellokierrosta. Jos maasta näkee aluksen, maaliviivan ja aluskellon, se näyttää siltä, että viisi kierrosta tulee täyteen, kun maaliviiva ylitetään. Kun Maa katsoo samalla omaa kelloaan, tämä kello on kuitenkin käynyt kuusi kierrosta. Aluksen mielestä, jos Maassa olisi mahdolista nähdä jokin paikka, mihin maa menee (samassa suunnassa kuin mihin nopeus on, eli periaatteessa aluksen pitää vetää tätä paikkaa perässään ja tuoda se Maalle), niin Maa kävisi vain viisi kierrosta aluksen kuudessa.

        "Onko mahdollista, että Maassa (G = 9,81) ja mustan aukon läheisyydessä (painovoima on suuri mutta rajallinen) olevat atomikellot ovat samassa inertiaalikoordinaatistossa? "

        Yllä olevista asiosta tulee vain mieleen, että kysyt tätä asiaa väärin eikä vastauksella ole vielä merkitystä.

        "Jos näin olisi, Maan atomikello sekä mustan aukon läheisyydessä oleva atomikello näyttäisivät samaa aikaa"

        Määrittele saman ajan näyttäminen. Mistään ns. faktasta, mikä pätee inertiaalikoordinaatistoissa ei seuraa mitään, mikä antaa suoraan jonkun vastauksen kaarevassa avaruudessa. Toisia kelloja yritetään käsittää litteässä avaruudessa Lorentzin muunnoksella, joka siirtyy yhdestä inertiaalikoordinaatistosta toiseen. Ei ole olemassa yhtään kaarevuutta omaavaa avaruutta, missä voisi siirtyä toisen kappaleen koordinaatistoon vain tekemällä tämän Lorentzin muunnoksen (homogeenisessa kentässä saa siirtyä sillä siihen suuntaan, mikä on poikittain vedon suuntaan, ja epähomogeenisyydessä ongelma ei ole joskus ajan nopeudessa vaan siinä, että tulee väärän verran huomioitua etäisyyttä ajan ja paikan suunnassa, jos yrittää litistää kaarevuuden pois). Tätä ei muuta se, että toinen kappale näyttää pysyvän paikoillaan. Kaarevassa avaruudessa eri kaarevuuspisteissä olevat kappaleet näkevät yleensä tällaisen paikallaan olemisen vain koska jompikumpi kappale käyttää jotain muuta kuin gravitaatiovoimaa kiihdyttämään itseään.

        1

        "Maan tosiaikaa tuossa ympäristössä ei voida tuntea."

        Jos minä saan nyt määritellä tämän, niin jos sanoit, että litteässä avaruudessa sai tuntea kaiken kaikesta tekemällä Lorentzin-muunnoksen, niin silloin erittäin usein GR:ssä voi tuntea kaiken tekemällä yleisemmän koordinaattimuunnoksen tai jonkin joukon laskuja. Omasta paikasta mustassa aukossa pitää olla täydellinen tietämys, samoin kuin muiden paikasta, ja silloin ylipäänsä tietämys aukosta pitää olla täydellinen samoin kuin edellä tieto nopeuksista oli täydellistä.

        "Aiheuttaako painovoima aina kiihtyvän liikkeen, vaikka kappale olisikin näennäisesti paikallaan? Määritelmällisesti tasainen liike ei voi aiheuttaa kiihtyvyyttä."

        Tasaisuuden toinen määritelmä on painovoiman aiheuttama liike. Kiihtyvä liike, joka tasan kumoaa painovoiman, ja pitää kappaleen nopeuden pienenä verrattuna johokin muuhun kappaleeseen, ei aiheuta mitään omaa lisäkontribuutiota näiden kahden kellon vertailuun.

        https://physics.stackexchange.com/questions/226055/time-dilation-and-freefall-follow-up-and-simplification

        2


      • Anonyymi
        Anonyymi kirjoitti:

        Suhteellisuusperiaatteen mukaisesti luonnonlait ovat samanlaisina voimassa kaikissa toistensa suhteen inertiaalisissa koordinaatistoissa.

        Niinpä. Mutta ne eivät ole Newtonin lait vaan Einsteinin. Huomaa tuo pieni ero!


    • Anonyymi

      Tämäkin urbaanilegenda kellon kyydityksestä nousee täällä esiin toistuvasti...

      • Anonyymi

        Kerro lisää tuntemuksistasi, siis mielipiteestäsi.


      • Anonyymi

        Onko dinosaurukset luut istutettu paikoilleen sellaisinaan tasan 6000 vuotta sitten?


      • Anonyymi

        Luulisi että noilla hörhöillä alkakaisi hälytyskellot soida, kun ei ole missään esitetty edes valokuvaa empiirisestä kokeesta, josta näkisi kellojen olevan eri ajassa.😁
        Täyttä huuhaata kunnes toisin todistetaan.


    • Anonyymi

      Päteekö liikkeen suhteellisuusperiaate suhteellisuusteoriassa?

      The Dynamic Universe (DU) eli dynaamisen universuminen mallissa (DU) kelloilla (atomeilla) on eri värähtelytaajuudet eri energiatiloissa; sen takia kellot näyttävät eri aikaa. Tiedeyhteisö on hylkinyt suomalaisen huippukeksijän, Tuomas Suntolan, teoriaa.

      https://physicsfoundations.org/fi/1_5_dynamic-universe.html

      Täällä on sanottu, että taajuus on sama kaikissa inertiaalikoordinaatistoissa.

      Vääriä vastauksia tässä avauksessa on annettu niin paljon, että luottamus vastaajiin on mennyt. Viimeksi henkilö toisti Väärin.-sanaa lukuisia kertoja, vaikka väitteet todetaan fysiikan oppikirjoissa.

      Kuka tahansa voi höpöttää näköjään mitä tahansa.

      • Anonyymi

        "Täällä on sanottu, että taajuus on sama kaikissa inertiaalikoordinaatistoissa."

        (Täällä on siellä, missä Suntola on?)

        Määrittele kenellä on taajuus, ja missä on inertiaalikoordinaatisto, ja mikä on sama. Einsteinin piti piirtää monta valoviivaa moneen kertaan eri tavalla sanoakseen asioista kuten juuri taajuus yhtään mitään, mutta sinä luulet tietäväsi ja kommunikoivasi kaiken sanomalla 'se' ja 'sama'.

        Jos sanot olevan jonkun yhteisön ulkopuolisen ihmisen ja sanot sen jälkeen lauseen hänen sivultaan, niin voisit myös mainita, onko tämä kyseisen henkilön näkemys tästä yhteisön kannasta vai esittääkö hän omaansa..

        Voin sanoa, että lause kuten 'taajuus on sama _vain_ kaikissa inertiaalikoordinatistoissa,' olisi mahdollista lukea kuin se olisi yhteisön kannan kyseenalaistava kommentti. Silloin jos siinä taajuus tarkoittaisi sitä, minkä taajuuden henkilö näkee itsensä luona.

        ...

        Täytyy mennä esim. alimpaan linkkiin, missä lukee ensimmäisessä kappaleessa

        Suntola: GPS–järjestelmän teoreettisista perusteista:
        "Kellojen käyntitaajuuden osalta vaatimus toteutuu kun havaitsijaan näh-
        den liikkeessä olevan kelloon sovelletaan Lorentz muunnoksesta saatavaa aikadilaatiota, ts.
        liikkeessä olevan kellon käyntitaajuus (tai ajan kulku) havaitaan tekijällä 2 2
        1 v c− hidastu-
        neena, missä v on kellon nopeus havaitsijaan nähden."

        Joten hän on sanonut, että kun taajuus on A-koordinaatistossa f, voidaan Lorentzin muunnoksella mennä inertiaalikoordinaatistosta A toiseen inertiaalikoordinaatistoon B, joka eroaa toisesta v:llä. Silloin sama objekti, missä taajuus oli f, ei ole enää f tästä koordinaatistosta nähtynä. Suntola on siis tässä oppikirjan mukainen.


      • Anonyymi
        Anonyymi kirjoitti:

        "Täällä on sanottu, että taajuus on sama kaikissa inertiaalikoordinaatistoissa."

        (Täällä on siellä, missä Suntola on?)

        Määrittele kenellä on taajuus, ja missä on inertiaalikoordinaatisto, ja mikä on sama. Einsteinin piti piirtää monta valoviivaa moneen kertaan eri tavalla sanoakseen asioista kuten juuri taajuus yhtään mitään, mutta sinä luulet tietäväsi ja kommunikoivasi kaiken sanomalla 'se' ja 'sama'.

        Jos sanot olevan jonkun yhteisön ulkopuolisen ihmisen ja sanot sen jälkeen lauseen hänen sivultaan, niin voisit myös mainita, onko tämä kyseisen henkilön näkemys tästä yhteisön kannasta vai esittääkö hän omaansa..

        Voin sanoa, että lause kuten 'taajuus on sama _vain_ kaikissa inertiaalikoordinatistoissa,' olisi mahdollista lukea kuin se olisi yhteisön kannan kyseenalaistava kommentti. Silloin jos siinä taajuus tarkoittaisi sitä, minkä taajuuden henkilö näkee itsensä luona.

        ...

        Täytyy mennä esim. alimpaan linkkiin, missä lukee ensimmäisessä kappaleessa

        Suntola: GPS–järjestelmän teoreettisista perusteista:
        "Kellojen käyntitaajuuden osalta vaatimus toteutuu kun havaitsijaan näh-
        den liikkeessä olevan kelloon sovelletaan Lorentz muunnoksesta saatavaa aikadilaatiota, ts.
        liikkeessä olevan kellon käyntitaajuus (tai ajan kulku) havaitaan tekijällä 2 2
        1 v c− hidastu-
        neena, missä v on kellon nopeus havaitsijaan nähden."

        Joten hän on sanonut, että kun taajuus on A-koordinaatistossa f, voidaan Lorentzin muunnoksella mennä inertiaalikoordinaatistosta A toiseen inertiaalikoordinaatistoon B, joka eroaa toisesta v:llä. Silloin sama objekti, missä taajuus oli f, ei ole enää f tästä koordinaatistosta nähtynä. Suntola on siis tässä oppikirjan mukainen.

        Mikä tuli todistetuksi.

        Väärin.


      • Anonyymi

        Totta, koska eräillä on niin kova pätemisen tarve.


    • Anonyymi

      "Inertiaalikoordinaatisto on koordinaatisto, jossa Newtonin mekaniikan lait pätevät."

      https://www.jyu.fi/science/fi/fysiikka/opiskelu/tyoosasto/tyot/fysa2020/fysa2020_2_k2015.pdf

      "Väärin. Muunmuassa jotkut koordinaatistot ovat äärettömiä, joten jos sanoo X:n pätevän koordinaatistossa, niin pätemistä on tosi paljon. Kaikki mitä näet olisi Newtonin mukaista havaintoa."

      Jyväskylän yliopisto siis opettaa väärin.

      "Havaintoympäristöt, jotka liikkuvat toistensa suhteen vakionopeudella, ovat inertiaalikoordinaatistoja."

      https://matematiikkalehtisolmu.fi/2014/SR1_TL.pdf

      "Väärin. Koordinaatiston voi määritellä yhden origon avulla. Jos tuntuu epävarmalta, että onko mahdollista kiihdyttää samaan suuntaan samalla voimalla ja pysyä samassa vauhdissa, niin silloin pitäisi olla samaan aikaan epäilys siitä, onko origon ympärillä ympäristö joka tottelee aina origon liikettä. Inertiaalikoordinaatiston voi määritellä yhden koordinaatiston avulla eikä tarvitse kahta. Tässä voi ajatella työkalua nimeltä akselometri."

      Fysiikan opettajat opettavat siis väärin.

      "Suhteellisuusperiaatteen mukaisesti luonnonlait ovat samanlaisina voimassa kaikissa toistensa suhteen inertiaalisissa koordinaatistoissa."

      https://users.aalto.fi/~thunebe1/courses/monjst.pdf

      "Väärin. Koordinaatistot eivät ole toistensa suhteen inertiaalisia. Inertiaalinen on aina inertiaalinen."

      Oulun yliopisto siis opettaa väärin.

      "Yksinkertaisesti tulkiten, jos avauksessa Maahan jäävä atomikello ja ns. avaruuskello eivät ole toistensa suhteen kiihtyvässä liikkeessä, ne ovat samassa inertiaalikoordinaatistossa"

      Suppea suhteellisuusteoria: fysiikka sama kaikissa inertiaalikoordinaatistoissa. Yleinen suhteellisuusteoria: fysiikka sama kaikissa koordinaatistoissa. Vakionopeudella liikkuvat koordinaatistot ovat erityisasemassa eli Lorentz-muunnos tehdään samalla tavalla joka paikassa aika-avaruudessa. AINE MÄÄRÄÄ AIKA-AVARUUDEN GEOMETRIAN.

      http://www.courses.physics.helsinki.fi/teor/stp/stp_2014_2.pdf

      "Väärin. Toinen on yhdessä inertiaalikoordinaatistossa, ja toinen kello on toisessa. Inertiaalikoordinaatiston yksi määrittelevä muuttuja on kellon nopeusvektori."

      Myös Helsingin yliopisto opettaa siis väärin.

      --

      Väärin, väärin, väärin ja väärin.

      Seurassamme on henkilö, joka tietää kaiken paremmin.

      --

      Onneksi Wikipedia osaa vastata asiaan täysin yksiselitteisesti ja kiertelemättä:

      "Täysin muuttumattomalla nopeudella liikkuvan kappaleen sanotaan olevan inertiaalikoordinaatistossa. Jos vakionopeuksisia kappaleita on useita ja ne liikkuvat täysin samaan suuntaan, ovat ne samassa inertiaalikoordinaatistossa, vaikka kappaleiden välinen etäisyys olisi ääretön. Siten inertiaalikoordinaatistotkin ovat äärettömiä. Eri vakionopeudella tai eri suuntaan liikkuvat kappaleet ovat eri inertiaalikoordinaatistoissa. Inertiaalikoordinaatistot ovat ihanteellisia eli kuvitteellisia, eikä niitä oikeasti ole. Tämä johtuu siitä, että kaikkialla kappaleiden liikkeeseen vaikuttaa kaukaistenkin kappaleiden painovoima, jolloin minkään todellisen kappaleen nopeus ei voi olla täysin muuttumaton."

      Jos kappaleet ovat kuvitteellisesti samassa inertiaalikoordinaatistossa, niissä vaikuttava aika on aina sama.

      • Anonyymi

        "Toinen on yhdessä inertiaalikoordinaatistossa, ja toinen kello on toisessa. Inertiaalikoordinaatiston yksi määrittelevä muuttuja on kellon nopeusvektori."

        Tämä väite on totta.


      • Anonyymi

        "Eri vakionopeudella tai eri suuntaan liikkuvat kappaleet ovat eri inertiaalikoordinaatistoissa."

        Jos kappaleet eivät liiku tai liikkuvat samaan suuntaan samalla nopeudella, ne ovat samassa inertiaalikoordinaatistossa. Intertiaalikoordinaatisto voi olla ääretön, joten se "kerää" kaikki mahdolliset samaan ryhmään kuuluvat kappaleet samaan koordinaatistoon äärettömän laajalta alueelta. Rajoja siis ei ole olemassa.

        Samassa intertiaalikoordinaatistossa, olipa etäisyys miten pitkä tahansa, kuvitteellinen aika ja sen kulku on sama.

        "Inertiaalikoordinaatistot ovat ihanteellisia eli kuvitteellisia, eikä niitä oikeasti ole. Tämä johtuu siitä, että kaikkialla kappaleiden liikkeeseen vaikuttaa kaukaistenkin kappaleiden painovoima, jolloin minkään todellisen kappaleen nopeus ei voi olla täysin muuttumaton."

        Tämä puolestaan tarkoittaa käytännössä sitä, että sama inertiaalikoordinaatisto äärettömän kaukana olevien kappaleiden välillä olisi äärimmäisen harvinaista ja täysin puhdasta sattumaa. Universumi on kuitenkin suuri paikka.


      • Anonyymi

        Luonnonlakien suhteellisuusperiaate: mikä tahansa tasaisella nopeudella liikkuva havaitsija mittaa omassa koordinaatistossaan luonnonlait normaaleiksi, ja kaikki tasaisella nopeudella liikkuvat koordinaatistot ovat keskenään "samankaltaisia". Kun havaitsijan nopeus muuttuu suhteessa toiseen liikkuvaan koordinaatistoon, koordinaatistot eivät ole enää samankaltaisia, joten luonnonlait tulkitaan niissä eri tavoin. Sama koskee aikaa.


      • Anonyymi
        Anonyymi kirjoitti:

        Luonnonlakien suhteellisuusperiaate: mikä tahansa tasaisella nopeudella liikkuva havaitsija mittaa omassa koordinaatistossaan luonnonlait normaaleiksi, ja kaikki tasaisella nopeudella liikkuvat koordinaatistot ovat keskenään "samankaltaisia". Kun havaitsijan nopeus muuttuu suhteessa toiseen liikkuvaan koordinaatistoon, koordinaatistot eivät ole enää samankaltaisia, joten luonnonlait tulkitaan niissä eri tavoin. Sama koskee aikaa.

        Aika kulkee tunnetusti yhteen suuntaan. Syy ei ole tiedossa: tiedetään vain että syy pitää olla ennen seurausta (kausaalisuus).


      • Anonyymi
        Anonyymi kirjoitti:

        Aika kulkee tunnetusti yhteen suuntaan. Syy ei ole tiedossa: tiedetään vain että syy pitää olla ennen seurausta (kausaalisuus).

        Anon.
        "Aika kulkee tunnetusti yhteen suuntaan. Syy ei ole tiedossa:..."

        Ajalla on erilaisia suhteita, kun eräs suhde vallitsee, puhutaan symmetriasta.
        kyseinen symmetria pätee luonnonlakien tasolla, se on luonnonlakien laki.

        Todellisuuden makrotasolla on pätee toinen symmetriasuhde, puhutaan
        symmetriarikosta, silloin koemme "ajan kulun".


      • Anonyymi

        "
        "Inertiaalikoordinaatisto on koordinaatisto, jossa Newtonin mekaniikan lait pätevät."
        https://www.jyu.fi/science/fi/fysiikka/opiskelu/tyoosasto/tyot/fysa2020/fysa2020_2_k2015.pdf
        "

        Lause on väärin, mutta tämä vihko on kirjoitettu maailmalle, jossa ei ole SR:ää eikä muita mekaniikan lakeja kuin Newton. Siinä inertiaalikoordinaatiston yksi vastakohta on pyörivä koordinaatisto. Sinun esimerkeissäsi ei mainita pyörimistä vaihtoehtona. Lisäksi tämä pyöriminen ei tämän vihon mukaan aiheuta eroja kellonaikoihin, vaikka lait ovat kuulema erilaiset johtuen näennäisvoimien ilmaantumisesta. Itseasiassa vihon mukaan lait eivät ole erilaiset, vaan se sanoo, että on olemassa yksi laki ja on olemassa tämän lain hylkäävä olotila. Eikä sille mitään voi muuta kuin esitellä näennäisvoimat ja olla niiden kanssa laittomana. SR muuten ei pysty olemaan tätä vihkoa parempi, koska SR:ssä koordinaatiston pyöriminen on myös näennäisvoimilta näyttävä. Ainoa kehitysaskel aina olevaksi yhdeksi ja pysyväksi laiksi olisi muodostaa GR-ideoita.

        Laillisuuden tarkempi määritelmä on se, että on olemassa yhtälö esim. kappaleen liikkeelle, ja tämä yhtälö on kovariantti eli samanmuotoinen, jos keksitään eri koordinaatisto-esityksiä kuvata samaa kappaletta. Mistä tahansa laista voi tehdä GR-muotoisen. Pyöriessä nähty vastaavuus on yleensä triviaali, mutta sitä voidaan kokeilla tämän ns. lain oikeaksi muodoksi, myös kun se viedään gravitaation äärirajoille, tai jos mittaustarkkuus riittää.

        "
        "Havaintoympäristöt, jotka liikkuvat toistensa suhteen vakionopeudella, ovat inertiaalikoordinaatistoja."
        https://matematiikkalehtisolmu.fi/2014/SR1_TL.pdf
        "

        Kyseinen lause on kohdasta, missä puhutaan Galilein suhteelllisuudesta. Voiko olla niin, että 'havaintoympäristö' on 1500-luvun kieltä? Galilein esimerkeissä yksi liiikkuva asia voi olla ruotsinlaivan kokoinen. Galilein mekaniikassa on niin vähän ilmiöitä, että materiaalinen ympäristö ei kärsi yhtä paljon siitä, että se liikkuu origossa olevan materiapisteen mukana.

        Se että inertiaalisuutta ei tarkisteta millään, vaan uskotaan siihen, miltä kaukana näyttää, on todella kiellettyä vasta SR-teksteissä, ja GR-teksteissä, missä vapaapudotustarkistus puolestaan ei anna olettaa mitään SR-inertiaalisuudesta.

        "
        "Fysiikan opettajat opettavat siis väärin."
        "

        Jokaisessa opettajassa tähän mennessä olisi ollut parannettavan varaa noissa kohdissa ja noissa konteksteissa, jotta tällaiselta lopputulokselta vältyttäisiin. On kuitenkin nämäkin kaksi ajateltavaa asiaa väärinopettamisesta:

        1.Kun 50 v Einstein opettaa SR:ää, hän opettaa jotain tarkoituksella väärin. Ei SR ole hänestä kauheasti totta.

        2. Fysiikassa voi olla myös fysiikan teksti, jossa lukee, että tätä tekstiä saa käyttää vain, jos on kaksi muoviämpäriä samalla lattialla. Tällöin lukijan pitäisi tietää, että ei kopioi siitä tekstistä mitään toisille, jos näillä ei ole samat ämpärit mielessä. Tai ainakin hänellä pitää itsellään olla jokin todella hyvä tarina antaa syyksi, ja voisi olla periaatteessa niin, että hän kritisoi ensimmäistä tekstiä sen näköalattomuudesta.

        "
        "Suhteellisuusperiaatteen mukaisesti luonnonlait ovat samanlaisina voimassa kaikissa toistensa suhteen inertiaalisissa koordinaatistoissa."
        https://users.aalto.fi/~thunebe1/courses/monjst.pdf
        "

        Siinä ei ole kirjoitettu tuolla tavalla, vaan oikea lainaus on

        'Suhteellisuusperiaatteen mukaan kaikkien luonnonlakien taytyy patea samanlaisina kaikissa
        inertiaalikoordinaatistoissa.'

        Toinen lauseeni tästä kohdasta:
        'Väärin. Koordinaatistot eivät ole toistensa suhteen inertiaalisia. Inertiaalinen on aina inertiaalinen.'
        keskittyy juuri tuohon eroon, joka teidän välillänne on. Oli hyvin oleellista mainita siitä, että intertiaalisia koordinaatistoja voi määritellä inertiaalisiksi yksi kerrallaan, koska kirjoitit viestissäsi monta kappaleta ajatuksia, jotka kaikki perustuvat johonkin ihmeelliseen monikkoon näitä koordinaatistoja, jotka dramaattisesti muuttuvat samuudeksi. Tämä lause minkä sinulta poimin, ei ollut mitenkään täydellinen kohde tähän, koska lause alkaa käsitteellä suhteellisuusperiaate, mikä on useasta koordinaatistosta puhuva asia. Sain kuitenkin onnekkaasti sanottua lausetta myös kokonaisuudessaan ja viestiltään vääräksi, koska siinä ajateltiin vain kapealla tavalla GR:ää edeltäneitä lakeja. Moniste, jonka löysit ei taas sisällä GR:ää. Jos et lue koskaan yhtään GR:n opetusta, niin tämä yritys verrata tekstejä ja opetuksia ei voi onnistua.

        "
        "Yksinkertaisesti tulkiten, jos avauksessa Maahan jäävä atomikello ja ns. avaruuskello eivät ole toistensa suhteen kiihtyvässä liikkeessä, ne ovat samassa inertiaalikoordinaatistossa"
        "

        Et ole maininnut linkkiä, mistä tämä on peräisin? Seuraavassa esityksessä on kuulema mahdollista olla monta koordinaatistoa, mutta ei ole annettu esimerkkiä sellaisesta tapauksesta, missä olisi joskus sama koordinaatisto kyseessä:

        (lainaus Helsingistä)

        1


      • Anonyymi
        Anonyymi kirjoitti:

        "
        "Inertiaalikoordinaatisto on koordinaatisto, jossa Newtonin mekaniikan lait pätevät."
        https://www.jyu.fi/science/fi/fysiikka/opiskelu/tyoosasto/tyot/fysa2020/fysa2020_2_k2015.pdf
        "

        Lause on väärin, mutta tämä vihko on kirjoitettu maailmalle, jossa ei ole SR:ää eikä muita mekaniikan lakeja kuin Newton. Siinä inertiaalikoordinaatiston yksi vastakohta on pyörivä koordinaatisto. Sinun esimerkeissäsi ei mainita pyörimistä vaihtoehtona. Lisäksi tämä pyöriminen ei tämän vihon mukaan aiheuta eroja kellonaikoihin, vaikka lait ovat kuulema erilaiset johtuen näennäisvoimien ilmaantumisesta. Itseasiassa vihon mukaan lait eivät ole erilaiset, vaan se sanoo, että on olemassa yksi laki ja on olemassa tämän lain hylkäävä olotila. Eikä sille mitään voi muuta kuin esitellä näennäisvoimat ja olla niiden kanssa laittomana. SR muuten ei pysty olemaan tätä vihkoa parempi, koska SR:ssä koordinaatiston pyöriminen on myös näennäisvoimilta näyttävä. Ainoa kehitysaskel aina olevaksi yhdeksi ja pysyväksi laiksi olisi muodostaa GR-ideoita.

        Laillisuuden tarkempi määritelmä on se, että on olemassa yhtälö esim. kappaleen liikkeelle, ja tämä yhtälö on kovariantti eli samanmuotoinen, jos keksitään eri koordinaatisto-esityksiä kuvata samaa kappaletta. Mistä tahansa laista voi tehdä GR-muotoisen. Pyöriessä nähty vastaavuus on yleensä triviaali, mutta sitä voidaan kokeilla tämän ns. lain oikeaksi muodoksi, myös kun se viedään gravitaation äärirajoille, tai jos mittaustarkkuus riittää.

        "
        "Havaintoympäristöt, jotka liikkuvat toistensa suhteen vakionopeudella, ovat inertiaalikoordinaatistoja."
        https://matematiikkalehtisolmu.fi/2014/SR1_TL.pdf
        "

        Kyseinen lause on kohdasta, missä puhutaan Galilein suhteelllisuudesta. Voiko olla niin, että 'havaintoympäristö' on 1500-luvun kieltä? Galilein esimerkeissä yksi liiikkuva asia voi olla ruotsinlaivan kokoinen. Galilein mekaniikassa on niin vähän ilmiöitä, että materiaalinen ympäristö ei kärsi yhtä paljon siitä, että se liikkuu origossa olevan materiapisteen mukana.

        Se että inertiaalisuutta ei tarkisteta millään, vaan uskotaan siihen, miltä kaukana näyttää, on todella kiellettyä vasta SR-teksteissä, ja GR-teksteissä, missä vapaapudotustarkistus puolestaan ei anna olettaa mitään SR-inertiaalisuudesta.

        "
        "Fysiikan opettajat opettavat siis väärin."
        "

        Jokaisessa opettajassa tähän mennessä olisi ollut parannettavan varaa noissa kohdissa ja noissa konteksteissa, jotta tällaiselta lopputulokselta vältyttäisiin. On kuitenkin nämäkin kaksi ajateltavaa asiaa väärinopettamisesta:

        1.Kun 50 v Einstein opettaa SR:ää, hän opettaa jotain tarkoituksella väärin. Ei SR ole hänestä kauheasti totta.

        2. Fysiikassa voi olla myös fysiikan teksti, jossa lukee, että tätä tekstiä saa käyttää vain, jos on kaksi muoviämpäriä samalla lattialla. Tällöin lukijan pitäisi tietää, että ei kopioi siitä tekstistä mitään toisille, jos näillä ei ole samat ämpärit mielessä. Tai ainakin hänellä pitää itsellään olla jokin todella hyvä tarina antaa syyksi, ja voisi olla periaatteessa niin, että hän kritisoi ensimmäistä tekstiä sen näköalattomuudesta.

        "
        "Suhteellisuusperiaatteen mukaisesti luonnonlait ovat samanlaisina voimassa kaikissa toistensa suhteen inertiaalisissa koordinaatistoissa."
        https://users.aalto.fi/~thunebe1/courses/monjst.pdf
        "

        Siinä ei ole kirjoitettu tuolla tavalla, vaan oikea lainaus on

        'Suhteellisuusperiaatteen mukaan kaikkien luonnonlakien taytyy patea samanlaisina kaikissa
        inertiaalikoordinaatistoissa.'

        Toinen lauseeni tästä kohdasta:
        'Väärin. Koordinaatistot eivät ole toistensa suhteen inertiaalisia. Inertiaalinen on aina inertiaalinen.'
        keskittyy juuri tuohon eroon, joka teidän välillänne on. Oli hyvin oleellista mainita siitä, että intertiaalisia koordinaatistoja voi määritellä inertiaalisiksi yksi kerrallaan, koska kirjoitit viestissäsi monta kappaleta ajatuksia, jotka kaikki perustuvat johonkin ihmeelliseen monikkoon näitä koordinaatistoja, jotka dramaattisesti muuttuvat samuudeksi. Tämä lause minkä sinulta poimin, ei ollut mitenkään täydellinen kohde tähän, koska lause alkaa käsitteellä suhteellisuusperiaate, mikä on useasta koordinaatistosta puhuva asia. Sain kuitenkin onnekkaasti sanottua lausetta myös kokonaisuudessaan ja viestiltään vääräksi, koska siinä ajateltiin vain kapealla tavalla GR:ää edeltäneitä lakeja. Moniste, jonka löysit ei taas sisällä GR:ää. Jos et lue koskaan yhtään GR:n opetusta, niin tämä yritys verrata tekstejä ja opetuksia ei voi onnistua.

        "
        "Yksinkertaisesti tulkiten, jos avauksessa Maahan jäävä atomikello ja ns. avaruuskello eivät ole toistensa suhteen kiihtyvässä liikkeessä, ne ovat samassa inertiaalikoordinaatistossa"
        "

        Et ole maininnut linkkiä, mistä tämä on peräisin? Seuraavassa esityksessä on kuulema mahdollista olla monta koordinaatistoa, mutta ei ole annettu esimerkkiä sellaisesta tapauksesta, missä olisi joskus sama koordinaatisto kyseessä:

        (lainaus Helsingistä)

        1

        "
        "Suppea suhteellisuusteoria: fysiikka sama kaikissa inertiaalikoordinaatistoissa. Yleinen suhteellisuusteoria: fysiikka sama kaikissa koordinaatistoissa. Vakionopeudella liikkuvat koordinaatistot ovat erityisasemassa eli Lorentz-muunnos tehdään samalla tavalla joka paikassa aika-avaruudessa. AINE MÄÄRÄÄ AIKA-AVARUUDEN GEOMETRIAN."
        http://www.courses.physics.helsinki.fi/teor/stp/stp_2014_2.pdf
        "

        Aika-avaruuden geometria ei ole sama asia kuin koordinaatisto. Joten yksi aine ei ole määrännyt äskeisessä kohdassa kelloja samaan inertiaalikoordinaatistoon tai mitään sellaista.

        "
        Vakionopeudella liikkuvat koordinaatistot ovat erityisasemassa eli Lorentz-muunnos tehdään samalla tavalla joka paikassa aika-avaruudessa.
        "

        Lorentz-muunnoksen tekeminen tässä kertoo jo, että inertiaalikoordinaatistoa muutetaan. Halian sanoa, että lauseena tämä on todella huonosti mietitty. Jos erityisasema poistetaan, tarkoitetaan olevan myös jotain ei-erityistä. Tässä tapauksessa tämä ei saisi olla vielä GR, koska jos lauseessa ajatellaan GR:n olevan olemassa, niin silloin vakionopeudella liikkuvat asiat eivät ole enää missään Lorentz-muunnoksen päässä toisistaan (tarkemmin esitetty yllä). Siten tämä on vain historiallinen tai jopa pedagoginen viittaus siihen, että jo ennen GR:ää on olemassa SR kiihtyvillä koordinaatistoilla. Kirjoittajan tarkoitus on sanoa, että GR on näihin kiihtyiin koordinaatteihin liittyvä ajatuksen jatke, tai GR:ssä (GR:n avaruuksia koskevissa ideoissa) voidaan esittää ne omalaatuisella tavalla, mikä selittää asiaa ns. lisää. Tämä on saanut esitelmänpitäjän kirjoittamaan tietyt sanat pötköön tällä tavalla, mutta se ei tarkoita, että pötkössä on sisällä jokin looginen sääntö, jota voidaan alkaa soveltamaan ja tehdä siitä kellojen sääntöjä SR:ssä tai GR:ssä. Jos esitelmänpitäjällä on ollut tämä SR-kiihtyvyydestä GR:ään pääaiheenaan, esityksen katsomisella voisi ehkä olla vaikutuksena, että joku sisäistää asian.

        Äskeisessä voi olla jotain mieltä lauseen alkua varten. Mutta lauseen loppuosa taas syökseen sen melkein suoraan GR:ään. Ja siten lauseessa on esiintyy ehkä se virhe, että se ajattelee Lorentzin muunnosten lokaalisen version GR:ssä muuttuvan erikoistaoaukseksi eli globaaliksi SR-versioksi sillä perusteella, että on vakionopeuskoordinaatistoja. Tosiasiassa siinä pitää ensin tehdä aika-avaruudesta, joka on invariantti suure, täysin litteä.

        Kiihtyminen SR:ssä on voinut olla jokin sivun 1 juttu, josta mennän nopeasti ohi myös esityksessä.Seuraava sivu näyttää olevan hyvin samasta aiheesta, ja siinä tehdään korjausliike siirtää lokaalien Lorentz-muunnosten keskustelu litteään avaruuteen (mikä myös sitten hautaa kaiken odottamani kiihtyvyyskeskustelun). Esitys on kuitenkin vähän pihalla, kun se sanoo saatavan vastauksia miksi-kysymyksiin, kuten juuri 'miksi suppean suhteellisuusteorian avaruus on homogeeninen' pelkästään sillä, kun GR:n tyhjiöksi on valittu homogeenisyys.

        Kysymys ' miksi Lorentz muunnoksen kertoimet ovat samat kaikkialla' on myös niin tyhmä kysymys, että siihen pitää tehdä tyhmä vastaus. Jos joku kysyy, miksi Lorentz-muunnos tehdään samalla tavalla joka paikassa, perusvastauksen pitäisi olla, 'koska se ei ole koko Poincare-muunnos'. Koko Poincare-muunnos sisältää origon translaatiot, ja sen voi esittää 5x5-matriisina, jossa kertoimet riippuvat siitä, mihin haluat origon siirtää. Poincare-muunnos koskee vain SR:ää. Mutta kun vertaa Lorentzin muunnosta Poincare-muunnokseen, huomaa myös että siinä pysytään yhdessä pisteessä, ja että GR:ssä on koko ajan käytössä sellainen lokaali Lorentz-muunnos,jossa annetaan piste, mistä juuri sitten ei tehdä translaatiota yhtään mihinkään. Kyseessä on siis yhden pisteen sisällä olevat eri nopeuksisten havaitsijoiden vertailut, jotka ovat tangenttiavaruudessa aika-avaruuden sijasta. Tästä saa toisen hienon miksi-vastauksen, kun sanoo, että Minkowskin avaruudessa tämä tangenttiavaruus on koko aika-avaruuden päällä ja vastaa sitä yksi-yhteen, mikä sitten globalisoi yhdessä pisteessä tehdyt muunnokset koskemaan kaikkia nyt idempotenttisesti tähän pisteeseen liittyviä muita tangenttiavaruuksia.

        "
        "Väärin. Toinen on yhdessä inertiaalikoordinaatistossa, ja toinen kello on toisessa. Inertiaalikoordinaatiston yksi määrittelevä muuttuja on kellon nopeusvektori."
        Myös Helsingin yliopisto opettaa siis väärin.
        "

        Helsingissä ei sanottu mitään näiden kellojen kahta koordinaatistoa vastaan. Tämä on ainoa lause, missä käytän sellaista huonoa notaatiota, että 'A on koordinaatistossa, ja, koordinaatisto missä A on, on inertiaalinen' olisi sama kuin 'A:lla on lepokoordinaatisto, A on inertiaalinen'.

        2


      • Anonyymi
        Anonyymi kirjoitti:

        "
        "Suppea suhteellisuusteoria: fysiikka sama kaikissa inertiaalikoordinaatistoissa. Yleinen suhteellisuusteoria: fysiikka sama kaikissa koordinaatistoissa. Vakionopeudella liikkuvat koordinaatistot ovat erityisasemassa eli Lorentz-muunnos tehdään samalla tavalla joka paikassa aika-avaruudessa. AINE MÄÄRÄÄ AIKA-AVARUUDEN GEOMETRIAN."
        http://www.courses.physics.helsinki.fi/teor/stp/stp_2014_2.pdf
        "

        Aika-avaruuden geometria ei ole sama asia kuin koordinaatisto. Joten yksi aine ei ole määrännyt äskeisessä kohdassa kelloja samaan inertiaalikoordinaatistoon tai mitään sellaista.

        "
        Vakionopeudella liikkuvat koordinaatistot ovat erityisasemassa eli Lorentz-muunnos tehdään samalla tavalla joka paikassa aika-avaruudessa.
        "

        Lorentz-muunnoksen tekeminen tässä kertoo jo, että inertiaalikoordinaatistoa muutetaan. Halian sanoa, että lauseena tämä on todella huonosti mietitty. Jos erityisasema poistetaan, tarkoitetaan olevan myös jotain ei-erityistä. Tässä tapauksessa tämä ei saisi olla vielä GR, koska jos lauseessa ajatellaan GR:n olevan olemassa, niin silloin vakionopeudella liikkuvat asiat eivät ole enää missään Lorentz-muunnoksen päässä toisistaan (tarkemmin esitetty yllä). Siten tämä on vain historiallinen tai jopa pedagoginen viittaus siihen, että jo ennen GR:ää on olemassa SR kiihtyvillä koordinaatistoilla. Kirjoittajan tarkoitus on sanoa, että GR on näihin kiihtyiin koordinaatteihin liittyvä ajatuksen jatke, tai GR:ssä (GR:n avaruuksia koskevissa ideoissa) voidaan esittää ne omalaatuisella tavalla, mikä selittää asiaa ns. lisää. Tämä on saanut esitelmänpitäjän kirjoittamaan tietyt sanat pötköön tällä tavalla, mutta se ei tarkoita, että pötkössä on sisällä jokin looginen sääntö, jota voidaan alkaa soveltamaan ja tehdä siitä kellojen sääntöjä SR:ssä tai GR:ssä. Jos esitelmänpitäjällä on ollut tämä SR-kiihtyvyydestä GR:ään pääaiheenaan, esityksen katsomisella voisi ehkä olla vaikutuksena, että joku sisäistää asian.

        Äskeisessä voi olla jotain mieltä lauseen alkua varten. Mutta lauseen loppuosa taas syökseen sen melkein suoraan GR:ään. Ja siten lauseessa on esiintyy ehkä se virhe, että se ajattelee Lorentzin muunnosten lokaalisen version GR:ssä muuttuvan erikoistaoaukseksi eli globaaliksi SR-versioksi sillä perusteella, että on vakionopeuskoordinaatistoja. Tosiasiassa siinä pitää ensin tehdä aika-avaruudesta, joka on invariantti suure, täysin litteä.

        Kiihtyminen SR:ssä on voinut olla jokin sivun 1 juttu, josta mennän nopeasti ohi myös esityksessä.Seuraava sivu näyttää olevan hyvin samasta aiheesta, ja siinä tehdään korjausliike siirtää lokaalien Lorentz-muunnosten keskustelu litteään avaruuteen (mikä myös sitten hautaa kaiken odottamani kiihtyvyyskeskustelun). Esitys on kuitenkin vähän pihalla, kun se sanoo saatavan vastauksia miksi-kysymyksiin, kuten juuri 'miksi suppean suhteellisuusteorian avaruus on homogeeninen' pelkästään sillä, kun GR:n tyhjiöksi on valittu homogeenisyys.

        Kysymys ' miksi Lorentz muunnoksen kertoimet ovat samat kaikkialla' on myös niin tyhmä kysymys, että siihen pitää tehdä tyhmä vastaus. Jos joku kysyy, miksi Lorentz-muunnos tehdään samalla tavalla joka paikassa, perusvastauksen pitäisi olla, 'koska se ei ole koko Poincare-muunnos'. Koko Poincare-muunnos sisältää origon translaatiot, ja sen voi esittää 5x5-matriisina, jossa kertoimet riippuvat siitä, mihin haluat origon siirtää. Poincare-muunnos koskee vain SR:ää. Mutta kun vertaa Lorentzin muunnosta Poincare-muunnokseen, huomaa myös että siinä pysytään yhdessä pisteessä, ja että GR:ssä on koko ajan käytössä sellainen lokaali Lorentz-muunnos,jossa annetaan piste, mistä juuri sitten ei tehdä translaatiota yhtään mihinkään. Kyseessä on siis yhden pisteen sisällä olevat eri nopeuksisten havaitsijoiden vertailut, jotka ovat tangenttiavaruudessa aika-avaruuden sijasta. Tästä saa toisen hienon miksi-vastauksen, kun sanoo, että Minkowskin avaruudessa tämä tangenttiavaruus on koko aika-avaruuden päällä ja vastaa sitä yksi-yhteen, mikä sitten globalisoi yhdessä pisteessä tehdyt muunnokset koskemaan kaikkia nyt idempotenttisesti tähän pisteeseen liittyviä muita tangenttiavaruuksia.

        "
        "Väärin. Toinen on yhdessä inertiaalikoordinaatistossa, ja toinen kello on toisessa. Inertiaalikoordinaatiston yksi määrittelevä muuttuja on kellon nopeusvektori."
        Myös Helsingin yliopisto opettaa siis väärin.
        "

        Helsingissä ei sanottu mitään näiden kellojen kahta koordinaatistoa vastaan. Tämä on ainoa lause, missä käytän sellaista huonoa notaatiota, että 'A on koordinaatistossa, ja, koordinaatisto missä A on, on inertiaalinen' olisi sama kuin 'A:lla on lepokoordinaatisto, A on inertiaalinen'.

        2

        "
        Onneksi Wikipedia osaa vastata asiaan täysin yksiselitteisesti ja kiertelemättä:
        "Täysin muuttumattomalla nopeudella liikkuvan kappaleen sanotaan olevan inertiaalikoordinaatistossa."
        "

        Wikipedia käyttää koordinaatisto-sanaa väärin, ja se käyttää inertiaalisuus-sanaa väärin jo esim. Newtonin näkökulmasta, missä on kaikkein helpointa kiihtyä yhtä nopeasti kuin muut ja pitää muut täysin muuttumattomassa nopeudessa itseensä nähden. Tieto siitä, että on kappale, mihin ei vaikuta voima, pitää tulla jostakin valmiina ennenkuin Newtonin mukaan voi sanoa, että se joka näkee tämän kappaleen liikkuvan vakionopeudella, on itse inertiaalinen havaitsija. Samalla vaivalla jokainen, johon ei vaikuta voima, olisi inertiaalinen havaitsija. Tämä on fyysinen mittaus, jonka voi tehdä itselleen. Samalla kun ensimmäinen tapaus, jossa saisi valita kohteen myös toisesta galaksista asti, on fiktiota, jota kerrotaan.

        "
        Jos vakionopeuksisia kappaleita on useita ja ne liikkuvat täysin samaan suuntaan, ovat ne samassa inertiaalikoordinaatistossa, vaikka kappaleiden välinen etäisyys olisi ääretön.
        "

        Yhtä väärin kuin äskeinen.

        Minä ajattelen, että tässä lukee näkymättömällä sana, että ne ovat saman suuruisia nopeuksia. Inertiaalikoordinaatisto ei voi olla olemassa ilman, että kerrotaan, missä sen origo käytännössä on. Joko sama koordinaatisto riittää tai sitten kaikkiin kappaleiden tutkimisen vaiheisiin ei edes tarvitse sellaista koordinaatistoa yhtään, joka toteuttaisi kaikkia koordinaatiston tehtäviä?

        Et käyttänyt minusta koskaan tällaisia toisistaan katsottuna paikallaan pysyviä kappaleita esimerkkinä, jotta niitä olisi pitänyt kommentoida.

        "
        Siten inertiaalikoordinaatistotkin ovat äärettömiä.
        "

        Mikä on inertiaalikoordinaatiston vastinpari? Ja miten sen äärettömyys näkyi?

        "
        Inertiaalikoordinaatistot ovat ihanteellisia eli kuvitteellisia, eikä niitä oikeasti ole. Tämä johtuu siitä, että kaikkialla kappaleiden liikkeeseen vaikuttaa kaukaistenkin kappaleiden painovoima, jolloin minkään todellisen kappaleen nopeus ei voi olla täysin muuttumaton.
        "

        Tämä koskee vielä tähänkin asti vain Newtonin gravitaatiota. Kukaan ei menisi tällaiseen keskusteluun GR:n kanssa.

        Tästä kohdasta voi saada monta väärää mielikuvaa siitä, milloin esim. Newtonimainen fysiikka on lähellä inertiaalisuutta. Yksi niistä on se, kun kosketaan Maan pintaa eikä nähdä, kun maa pyörii. Toinen se kun pudotaan suorassa linjassa kohti maanpintaa, ja sanotaan että se on Maan vika, että se liikkuu meitä kohti.

        "
        "Jos kappaleet ovat kuvitteellisesti samassa inertiaalikoordinaatistossa, niissä vaikuttava aika on aina sama."
        "

        Wikipediassa ei otettu mitään aikaa tai ajan muutoksia huomioon, joten et voi perustaa tätä siihen, mitä siellä sanottiin. Kesken lauseen ei pidä tehdä mistään kuvitteellista, koska se koskee kaikkea siinä lauseessa. Eli onko tarpeeksi selvitetty asiaa, jos 'aika on kuvitteellisesti sama'? Suppeassa suhteellisuusteoriassa näkee, miten moni ajan asia on sama, jos kappaleet ovat Lorentzin muunnoksen päässä toisistaan, kun niiden välinen etäisyys v = 0 ja muunnoksessa kerrotaan toisen aikakoordinaatti (joka on erikseen toisena aikakoordinaattina) ykkösellä. Silti silloin esim. Einstein päätti, että hän tekee vielä Einsteinin synkronisaation.

        3


      • Anonyymi
        Anonyymi kirjoitti:

        "
        Onneksi Wikipedia osaa vastata asiaan täysin yksiselitteisesti ja kiertelemättä:
        "Täysin muuttumattomalla nopeudella liikkuvan kappaleen sanotaan olevan inertiaalikoordinaatistossa."
        "

        Wikipedia käyttää koordinaatisto-sanaa väärin, ja se käyttää inertiaalisuus-sanaa väärin jo esim. Newtonin näkökulmasta, missä on kaikkein helpointa kiihtyä yhtä nopeasti kuin muut ja pitää muut täysin muuttumattomassa nopeudessa itseensä nähden. Tieto siitä, että on kappale, mihin ei vaikuta voima, pitää tulla jostakin valmiina ennenkuin Newtonin mukaan voi sanoa, että se joka näkee tämän kappaleen liikkuvan vakionopeudella, on itse inertiaalinen havaitsija. Samalla vaivalla jokainen, johon ei vaikuta voima, olisi inertiaalinen havaitsija. Tämä on fyysinen mittaus, jonka voi tehdä itselleen. Samalla kun ensimmäinen tapaus, jossa saisi valita kohteen myös toisesta galaksista asti, on fiktiota, jota kerrotaan.

        "
        Jos vakionopeuksisia kappaleita on useita ja ne liikkuvat täysin samaan suuntaan, ovat ne samassa inertiaalikoordinaatistossa, vaikka kappaleiden välinen etäisyys olisi ääretön.
        "

        Yhtä väärin kuin äskeinen.

        Minä ajattelen, että tässä lukee näkymättömällä sana, että ne ovat saman suuruisia nopeuksia. Inertiaalikoordinaatisto ei voi olla olemassa ilman, että kerrotaan, missä sen origo käytännössä on. Joko sama koordinaatisto riittää tai sitten kaikkiin kappaleiden tutkimisen vaiheisiin ei edes tarvitse sellaista koordinaatistoa yhtään, joka toteuttaisi kaikkia koordinaatiston tehtäviä?

        Et käyttänyt minusta koskaan tällaisia toisistaan katsottuna paikallaan pysyviä kappaleita esimerkkinä, jotta niitä olisi pitänyt kommentoida.

        "
        Siten inertiaalikoordinaatistotkin ovat äärettömiä.
        "

        Mikä on inertiaalikoordinaatiston vastinpari? Ja miten sen äärettömyys näkyi?

        "
        Inertiaalikoordinaatistot ovat ihanteellisia eli kuvitteellisia, eikä niitä oikeasti ole. Tämä johtuu siitä, että kaikkialla kappaleiden liikkeeseen vaikuttaa kaukaistenkin kappaleiden painovoima, jolloin minkään todellisen kappaleen nopeus ei voi olla täysin muuttumaton.
        "

        Tämä koskee vielä tähänkin asti vain Newtonin gravitaatiota. Kukaan ei menisi tällaiseen keskusteluun GR:n kanssa.

        Tästä kohdasta voi saada monta väärää mielikuvaa siitä, milloin esim. Newtonimainen fysiikka on lähellä inertiaalisuutta. Yksi niistä on se, kun kosketaan Maan pintaa eikä nähdä, kun maa pyörii. Toinen se kun pudotaan suorassa linjassa kohti maanpintaa, ja sanotaan että se on Maan vika, että se liikkuu meitä kohti.

        "
        "Jos kappaleet ovat kuvitteellisesti samassa inertiaalikoordinaatistossa, niissä vaikuttava aika on aina sama."
        "

        Wikipediassa ei otettu mitään aikaa tai ajan muutoksia huomioon, joten et voi perustaa tätä siihen, mitä siellä sanottiin. Kesken lauseen ei pidä tehdä mistään kuvitteellista, koska se koskee kaikkea siinä lauseessa. Eli onko tarpeeksi selvitetty asiaa, jos 'aika on kuvitteellisesti sama'? Suppeassa suhteellisuusteoriassa näkee, miten moni ajan asia on sama, jos kappaleet ovat Lorentzin muunnoksen päässä toisistaan, kun niiden välinen etäisyys v = 0 ja muunnoksessa kerrotaan toisen aikakoordinaatti (joka on erikseen toisena aikakoordinaattina) ykkösellä. Silti silloin esim. Einstein päätti, että hän tekee vielä Einsteinin synkronisaation.

        3

        " kun niiden välinen etäisyys v = 0"

        v on niiden välinen nopeus.


    • Anonyymi

      Uskon mieluummin yliopiston juttuihin ja Wikipediaan kuin väärinväärinväärinväärin-mieheen.

    • Anonyymi

      Omassa ranteessasi oleva kello kertoo miten aika kulkee omassa koordinaatiossasi. Oman kellosi sekunti on siis meidän tutun sekuntimme pituinen.
      Verrattuna muiden koordinaatistojen aikaan, aika muuttuu nopeuden ja gravitaation mukaan. Oma aikasi on silti se vanha tuttu.

      • Anonyymi

        Kun puhutaan koordinaatistoista, onko tuollainen kello tarkka fysiikassa arvioidulla tavalla? Kalibroimaton mekaaninen rannekello saattaa usein jätättää tai edistää jopa minuutin tai pari vuorokaudessa.

        Useilla nuorilla on nykyisin käytössään vain kännykän näytön kello, ei taskukello, rannekelloa tai seinäkelloa. Aktiivisuusmittari toimittaa myös kellon virkaa.


    • Anonyymi

      > Uskon mieluummin yliopiston juttuihin ja Wikipediaan kuin väärinväärinväärinväärin-mieheen.

      Se on luonnollista.

      > Verrattuna muiden koordinaatistojen aikaan, aika muuttuu nopeuden ja gravitaation mukaan.

      Samassa intertiaalikoordinaatistossa, olipa etäisyys miten pitkä tahansa, aika ja sen kulku on sama. Inertiaalikoordinaatisto on kuvitteellinen eikä sillä ole rajoja, vaan se voi olla ääretön.

      > Luulisi että noilla hörhöillä alkakaisi hälytyskellot soida, kun ei ole missään esitetty edes valokuvaa empiirisestä kokeesta, josta näkisi kellojen olevan eri ajassa.

      Miten se valokuva voisi todistaa asiaa mitenkään todeksi? Ei edes (halpa) suurnopeuskamera pysyisi mukana asiassa, ja valokuvien manipulointi on maailman toiseksi helpoin tehtävä.

      > Koska aika on sidottu atomikellon värähtelyyn, mutta ko. avaruuden olosuhteissa aika kuluu hitaammin, atomikellon värähtelytaajuuden on myös muututtava, tai Maassa määritelty kerroin on väärin.

      The Dynamic Universe (DU) eli dynaamisen universuminen mallissa kelloilla (atomeilla) on eri värähtelytaajuudet eri energiatiloissa; sen takia kellot näyttävät eri aikaa. (TT Suntola)

      Ei siis ole täyttä yksimielisyyttä siitä, mihin aikaero perustuu eli onko värähtelytaajuus muuttuva tekijä. Maan koordinaatistossa ja kaikissa samanlaisissa inertiaalikoordinaatistoissa atomien (atomikellojen) värähtelytaajuus on tietenkin yksi ja sama. Sekunti on aina sekunti samassa inertiaalikoordinaatistossa.

      Aikadilaation olemassaolosta ei fyysikoiden tai insinöörien kesken ole kuitenkaan lainkaan epäselvyyttä: "aika kuluu" eri tavoin eri nopeuksissa ja eri painovoimakentissä. "Tosiaika" on aina paikallista. Muissa paikoissa eli muissa koordinaatistoissa kellojen käyntinopeus vaihtelee sen mukaan, millainen gravitaatiopotentiaali, nopeus, kiihtyvys ja etäisyys niillä on käyntinopeutta havainnoivan suhteen. Aika on havainnoija-riippuvainen.

      • Anonyymi

        "Miten se valokuva voisi todistaa asiaa mitenkään todeksi? Ei edes (halpa) suurnopeuskamera pysyisi mukana asiassa, ja valokuvien manipulointi on maailman toiseksi helpoin tehtävä."

        Kalifornian teknillisen yliopiston tutkija Lihong Wang työryhmineen raportoi nyt kehittäneensä suurnopeuskameran, joka ottaa 70 biljoonaa valokuvaa sekunnissa. Kameran kuvataajuus on niin suuri, että sillä on mahdollista kuvata esimerkiksi valon etenemistä, atomiytimien fuusioitumista tai radioaktiivista hajoamista.

        https://tekniikanmaailma.fi/maailman-nopein-kamera-ottaa-70-biljoonaa-kuvaa-sekunnissa-tutkijoiden-esittelema-kamera-pystyy-kuvaamaan-valon-etenemista-tai-atomiytimien-fuusioitumista/


      • Anonyymi
        Anonyymi kirjoitti:

        "Miten se valokuva voisi todistaa asiaa mitenkään todeksi? Ei edes (halpa) suurnopeuskamera pysyisi mukana asiassa, ja valokuvien manipulointi on maailman toiseksi helpoin tehtävä."

        Kalifornian teknillisen yliopiston tutkija Lihong Wang työryhmineen raportoi nyt kehittäneensä suurnopeuskameran, joka ottaa 70 biljoonaa valokuvaa sekunnissa. Kameran kuvataajuus on niin suuri, että sillä on mahdollista kuvata esimerkiksi valon etenemistä, atomiytimien fuusioitumista tai radioaktiivista hajoamista.

        https://tekniikanmaailma.fi/maailman-nopein-kamera-ottaa-70-biljoonaa-kuvaa-sekunnissa-tutkijoiden-esittelema-kamera-pystyy-kuvaamaan-valon-etenemista-tai-atomiytimien-fuusioitumista/

        Vähän suppean suhteellisuusteorian ytimestä.

        Kuten olemme huomanneet kirjoituksista SR on havaitsijakeskeinen,
        kuitenkin liike sellaisenaan ei ole havaitsijakeskeinen, puhumattakaan
        mitään ajasta, joka ei välitä havaitsijoista mitään.

        Tietysti Einstein joutui määrittelemään ajan käsitteen kellon käsitteellä,
        koska "meillä ei ole mitään parempaa", kuten hän sanoi.

        Ihminen tuo mukanaan vertailukoordinaatiston, josta hän kellon avulla
        havainnoi liikkeitä ja aikaa. Ennen ihmistä ja ennen Einsteinia oli tapahtumia
        jotka eivät olleet havaitsijakeskeisiä, Einsteinin jälkeen koordinaatistokäsitteen
        kautta aika muuttuikin havaitsijakeskeiseksi.

        Nyt tähän on sanottava, että Einstein teki erittäin huonon valinnan tieteen
        historiassa, hän nimesi vakiintuneen sekuntitermin, myös oman teoriansa
        sekunniksi. Nyt onkin niin, että on kaksi sekunnin määritelmää, alkuperäinen
        ja einsteinsekunti (e-sekunti) jotka käyttäytyvät eritavoin. Paljon huomiota
        saaneet atomikellot tuottavat e-sekunteja ja einsteinaikaa (e-aika).

        Suppeaa suhteellisuusteoriaa on tutkittu tiivisti ajan kuluessa, tuloksista voi sanoa
        seuraavaa.

        Fyysikot: pitävät siitä, ainakin einsteinilaiset kovastikin.

        Matemaatikot: Nieleskelevät hiukan,eivät pidä postulaateista, mutta laskevat teorian mukaan. Ihmettelevät hieman.

        Loogikot: menevät syvälle pinnan alle, karsastavat postulaatteja. Tutkimuksissa
        joissa käsitellään SR:n ja logiikan suhteita on tullut kovaakin kommenttia.
        Kun otetaan huomioon logiikan välttämättömyys tieteelle, niin aika outoa
        suhteellisuusteorialle.

        "Therefore, the relativity of time (according to SR) invalidates the
        universal application of the fundamental laws of logic to all reference
        frames: it would only be applicable to proper references frames. And
        considering the basic necessity of those laws to build arguments, we
        could only be sure of the ones we build in our proper reference frame.
        The Lorentz Transformation not only deforms space and time but also
        deforms the universality of the laws of logic."


    • Anonyymi

      "Samassa intertiaalikoordinaatistossa, olipa etäisyys miten pitkä tahansa, aika ja sen kulku on sama. Inertiaalikoordinaatisto on kuvitteellinen eikä sillä ole rajoja, vaan se voi olla ääretön."

      Aivan. Aika-avaruus on koko universumin ydin.

    • Anonyymi

      Aikadilaatio on kellon näyttämänä totta, mutta se on käytännössä mahdollista todeta myös muulla tavaoin.

      Suuri osa voimakkaasta kosmisesta säteilystä, jos se pääsee aurinkotuulen läpi, hajoaa viimeistään Maan ilmakehässä myoneiksi. Myonien nopeus on äärimmäisen lähellä valon nopeutta, ja niiden elinikä on todella lyhyt. Maan eli meidän ihmisten koordinaatistossa (inertiaalikoordinaatistossa) myonien ei pitäisi elinikänsä puitteissa, ihmisten inertiaalikoordinaatistossa 2.2 mikrosekuntia, saavuttaa Maan pintaa vaikkapa merenpinnan tasolla. Kyllä silti saavuttavat.

      Avaruudesta saapuva kosminen säteily ja siten ko. muodostuvat myonit elävät omassa inertiaalikoordinaatistossaan, joka ei välitä meidän mielipiteistämme tai meidän ajastamme. Koska ko. myonien nopeus on lähellä valonnopeutta, myonien elinikä omassa koordinaatistossaan on noin 20 kertaa pidempi kuin meidän: myonit saavuttavat helposti Maan pinnan ajassa 20 x 2.2 = 44 mikrosekuntia.

      Einstein oli kova kaveri - osasi ajatella sen, mitä muut pystyivät vasta myöhemmin mittaamaan.

    • Anonyymi

      Kyllä on turhaa lässytystä koko ketju, muutamaa poikkeusta lukuunottamatta.
      Tässä ketjussa "sokea rampaa taluttaa".

      • Anonyymi

        No, valaise meitä todella hyvillä vastauksilla, ettei vaan jäisi lässytyksen
        makua omasta kommentistasi.


      • Anonyymi
        Anonyymi kirjoitti:

        No, valaise meitä todella hyvillä vastauksilla, ettei vaan jäisi lässytyksen
        makua omasta kommentistasi.

        Ei tuollaisia kirjoittajia kannata yrittää "valaista", Jatkatte vaan jankutustanne valaisi teitä miten hyvänsä, kommentit menevät teiltä ohi kuin vesi hanhen selästä.


      • Anonyymi
        Anonyymi kirjoitti:

        Ei tuollaisia kirjoittajia kannata yrittää "valaista", Jatkatte vaan jankutustanne valaisi teitä miten hyvänsä, kommentit menevät teiltä ohi kuin vesi hanhen selästä.

        "Ei tuollaisia kirjoittajia kannata yrittää "valaista","

        Onko tuo parasta mitä saat aikaiseksi ??


    • Anonyymi

      "Hitainta" aika on avaruuden painottomuudessa. Ero Maahan verrattuna on kuitenkin mitättömän pieni.
      Paikallinen sekunti on aina saman mittainen paikalliselle havaitsijalle, nopeudesta tai gravitaatiosta riippumatta.

      • Anonyymi

        Nii-in - Kyllä just näin Kuten kirjoitit:"Paikallinen sekunti on aina saman mittainen paikalliselle havaitsijalle, nopeudesta tai gravitaatiosta riippumatta".

        - Eikös tässä nimen omaan yritetä selvittää keskustelemalla ajan venyvää käsitettä ja selventää itse kullekkin.

        Aloittajan ensimmäiset rivit otsikon jälkeen: "> Mikä on nopein mahdollinen nopeus jonka ajan kulku voi saavuttaa?

        Eli jos tähän saisi vastausta.


      • Anonyymi
        Anonyymi kirjoitti:

        Nii-in - Kyllä just näin Kuten kirjoitit:"Paikallinen sekunti on aina saman mittainen paikalliselle havaitsijalle, nopeudesta tai gravitaatiosta riippumatta".

        - Eikös tässä nimen omaan yritetä selvittää keskustelemalla ajan venyvää käsitettä ja selventää itse kullekkin.

        Aloittajan ensimmäiset rivit otsikon jälkeen: "> Mikä on nopein mahdollinen nopeus jonka ajan kulku voi saavuttaa?

        Eli jos tähän saisi vastausta.

        Aika kulkee sekunnin kerrallaan Toisinaan jopa nanosekunnin kerrallaan.
        Kun kerran vastaus on jo annettu niin tuosta on aivan turha ruveta inttämään että pitäisi saada vastaus.


      • Anonyymi
        Anonyymi kirjoitti:

        Nii-in - Kyllä just näin Kuten kirjoitit:"Paikallinen sekunti on aina saman mittainen paikalliselle havaitsijalle, nopeudesta tai gravitaatiosta riippumatta".

        - Eikös tässä nimen omaan yritetä selvittää keskustelemalla ajan venyvää käsitettä ja selventää itse kullekkin.

        Aloittajan ensimmäiset rivit otsikon jälkeen: "> Mikä on nopein mahdollinen nopeus jonka ajan kulku voi saavuttaa?

        Eli jos tähän saisi vastausta.

        "Aloittajan ensimmäiset rivit otsikon jälkeen: "> Mikä on nopein mahdollinen nopeus jonka ajan kulku voi saavuttaa? "
        Nopein mahdollinen ajan nopeus olisi se, missä 99,999...% valon nopeudesta liikkuva tarkkailija seuraisi paikallaan olevaa kelloa.
        Mutta kuitenkin, paikallinen sekunti on aina saman mittainen paikalliselle havaitsijalle, nopeudesta tai gravitaatiosta riippumatta. "Oma" sekunti on aina tavallisen sekunnin mittainen, muiden ajankulun nopeus muuttuu omaasi verrattuna.


    Ketjusta on poistettu 1 sääntöjenvastaista viestiä.

    Luetuimmat keskustelut

    1. Monenko kanssa olet harrastanut seksiä

      tänä aikana kun olet kaivattuasi kaipaillut?
      Ikävä
      119
      2651
    2. Timo Soini tyrmää Tynkkysen selitykset Venäjän putinistileiristä

      "Soini toimi ulkoministerinä ja puolueen puheenjohtajana vuonna 2016, jolloin silloinen perussuomalaisten varapuheenjoht
      Maailman menoa
      255
      1148
    3. Melkein lähetin viestin.

      Onneksi tulin järkiini. Mukavaa kesää
      Ikävä
      86
      1104
    4. Taas kuoli kuortaneella

      Mitä tapahtui kuhinoilla kun auton alle jäi ja kuoli 66.
      Kuortane
      8
      1064
    5. Nainen voi rakastaa

      Ujoakin miestä, mutta jos miestä pelottaa näkeminenkin, niin aika vaikeaa on. Semmoista ei varmaan voi rakastaa. Miehelt
      Ikävä
      79
      1011
    6. Sulla on nainen muuten näkyvät viiksikarvat naamassa jotka pitää poistaa

      Kannattaa katsoa peilistä lasien kanssa, ettet saa ihmisiltä ikäviä kommentteja.
      Ikävä
      63
      933
    7. Kalateltta fiasko

      Onko Tamperelaisyrittäjälle iskenyt ahneus vai mistä johtuu että tänä vuonna ruuat on surkeita aikaisempiin vuosiin verr
      Kuhmo
      12
      930
    8. Rakastan sinua

      Olen tiennyt sen pitkään mutta nyt ymmärsin että se ei menekään ohi
      Ikävä
      30
      896
    9. IS Viikonloppu 20.-21.7.2024

      Tällä kertaa Toni Pitkälä esittelee piirrostaitojansa nuorten pimujen, musiikkibändien ja Raamatun Edenin kertomusten ku
      Sanaristikot
      41
      832
    10. Ikävöimäsi henkilön ikä

      Minkä ikäinen kaipauksen kohteenne on? Onko tämä vain plus 50 palsta vai kaivataanko kolme-neljäkymppisiä? Oma kohde mie
      Ikävä
      37
      799
    Aihe